■ちょっとした疑問はここに書いてね30■

このエントリーをはてなブックマークに追加
1ご冗談でしょう?名無しさん
前スレ
■ちょっとした疑問はここに書いてね29■
http://science2.2ch.net/test/read.cgi/sci/1076324052/

【質問する前に】
教科書や参考書をよく読もう。
http://www.yahoo.co.jp/ とか http://www.google.com/ を利用し自分で調べること。
宿題を聞くときは、どこまでやってみてどこが分からないのかを書くこと。
丸投げはダメだからね。
(丸投げ君は完全無視。答えるだけ無駄。)

質問に対する返答には、何かしらの返答ちょうだいね。

★過去スレ、関連スレはここ>>2よ。
★それから、書き込む前に>>3の注意事項を読んでね。
★数式の書き方(参考)はこちら>>4
(予備リンク:>>2-10
荒らし厳禁、煽りは黙殺、忘れないうちに定期age。
単発質問スレを発見したらこのスレへの誘導をよろしくね。>ALL
2ご冗談でしょう?名無しさん:04/03/24 15:52 ID:???
過去スレ
Part1:http://cheese.2ch.net/sci/kako/968/968116983.html
Part2:http://cheese.2ch.net/sci/kako/981/981661589.html
Part3:http://cheese.2ch.net/sci/kako/991/991491349.html
Part4:http://cheese.2ch.net/sci/kako/997/997037229.html
Part5:http://cheese.2ch.net/sci/kako/1003/10030/1003082106.html
Part6:http://cheese.2ch.net/sci/kako/1006/10064/1006423654.html
Part7:http://cheese.2ch.net/sci/kako/1011/10111/1011199123.html
Part8:http://cheese.2ch.net/sci/kako/1014/10149/1014955593.html
Part9:http://science.2ch.net/test/read.cgi/sci/1019557050/
Part10:http://science.2ch.net/test/read.cgi/sci/1024330084/
Part11:http://science.2ch.net/test/read.cgi/sci/1025840212/
Part12:http://science.2ch.net/test/read.cgi/sci/1027694658/
Part13:http://science.2ch.net/test/read.cgi/sci/1029946423/
Part14:http://science.2ch.net/test/read.cgi/sci/1032629984/
Part15:http://science.2ch.net/test/read.cgi/sci/1035913464/
Part16:http://science.2ch.net/test/read.cgi/sci/1038224562/
Part17:http://science.2ch.net/test/read.cgi/sci/1041622629/
Part18:http://science.2ch.net/test/read.cgi/sci/1043668297/
Part19:http://science.2ch.net/test/read.cgi/sci/1045448430/
Part20:http://science.2ch.net/test/read.cgi/sci/1048741188/
Part21:http://science.2ch.net/test/read.cgi/sci/1052820825/
Part22:http://science.2ch.net/test/read.cgi/sci/1055317668/
Part23:http://science.2ch.net/test/read.cgi/sci/1057733284/
Part24:http://science2.2ch.net/test/read.cgi/sci/1060960040/
Part25:http://science2.2ch.net/test/read.cgi/sci/1067352339/
Part26:http://science2.2ch.net/test/read.cgi/sci/1068020889/
Part27:http://science2.2ch.net/test/read.cgi/sci/1071424750/
Part28:http://science2.2ch.net/test/read.cgi/sci/1074075716/
3ご冗談でしょう?名無しさん:04/03/24 15:55 ID:???
書き込む際の注意

1.)
板の性格上、UNIX端末からの閲覧も多いと推察されます。
機種依存文字(ローマ数字、丸数字等)は避けて下さい。

2.)
以下のような質問に物理板住人は飽き飽きしているので、たぶん無視されます。
しないで下さい。
「相対性理論は間違っています」「量子力学は間違っています」
「宇宙論は間違っています」「シュレディンガーの猫は変です」
「永久機関を作りました」「タイムマシンについて教えて」
「どうして〜?」関連(→「どのようにして〜?」と質問すること)
「なぜ〜?」関連(物理学の対象ではないため)

「どうして・なぜ」:http://academy.2ch.net/philo/
(哲学板・雑談板のほうがむいている場合が多いです。)
雑談は雑談スレで:http://science.2ch.net/test/read.cgi/sci/973536997/

3.)
宿題を聞くときは、どこまでやってみてどこが分からないのかを書くこと。
丸投げはダメよ。丸投げに答えるのもダメよ。
せめてポインタを示す程度に留めましょう。

4.)
厨房を放置できない人も厨房です
4ご冗談でしょう?名無しさん:04/03/24 15:56 ID:???
掲示板での数学記号の書き方例(数学板より拝借)】
●スカラー:a,b,c,...,z, A,B,C,...,Z, α,β,γ,...,ω, Α,Β,Γ,...,Ω, ...
●ベクトル:x=[x[1],x[2],...], |x>, x↑, vector(x) (← 混同しない場合はスカラーと同じ記号でいい.通常は(成分を横で書いても)縦ベクトルとして扱う.)
●行列(1成分表示):A[i,j], I[i,j]=δ_(ij)
●行列(全成分表示):A=[[A[1,1],A[2,1],...],[A[1,2],A[2,2],...],...]=[a1,a2,a3,...], I=[[1,0,0,...],[0,1,0,...],[0,0,1,...],...] (←ここでは列ごとに表示(縦ベクトルを横に並べる).行ごとに表示しても構わないが,統一して使わないと混同するので注意.)
●関数:f(x), f[x]
●数列:a(n), a[n], a_n
●足し算:a+b
●引き算:a-b
●掛け算:a*b, ab (← 通常"*"を使い,"x"は使わない.)
●割り算・分数:a/b, a/(b+c), a/(bc) (← 通常"/"を使い,"÷"は使わない.)
●複号:a±b=a士b, a干b (← "±"は「きごう」で変換可.他に漢字の"士""干"なども利用できる.)
●平方根:√(a+b)=(a+b)^(1/2) (← "√"は「るーと」で変換可.)
●指数・指数関数:a^b, x^(n+1), exp(x+y)=e^(x+y) (← "^"を使う."exp"はeの指数.)
●対数・対数関数:log_{a}(b), log(x/2)=log_{10}(x/2), ln(x/2)=log_{e}(x/2) (← 底を省略する場合,"log"は常用対数,"ln"は自然対数.)
5ご冗談でしょう?名無しさん:04/03/24 15:56 ID:???
●三角比・三角関数:sin(a), cos(x+y), tan(x/2)
●内積・外積・スカラー3重積:a・b=(a,b), axb=[a,b], a・(bxc)=(axb)・c=[a,b,c]=det([a,b,c])
●行列式・トレース:|A|=det(A), tr(A)
●絶対値:|x|
●ガウス記号:[x] (← 関数の変数表示などと混同しないように注意.)
●共役複素数:z~
●階乗:n!=n*(n-1)*(n-2)*...*2*1, n!!=n*(n-2)*(n-4)*...
●順列・組合せ:P[n,k]=nPk, C[n.k]=nCk, Π[n,k]=nΠk, H[n,k]=nHk (← "Π"は「ぱい」で変換可.)
●微分・偏微分:y', dy/dx, ∂y/∂x (← "∂"は「きごう」で変換可.)
●ベクトル微分:∇f=grad(f), ∇・A=div(A),∇xA=rot(A), (∇^2)f=Δf (← "∇"は「きごう」,"Δ"は「でるた」で
変換可.)
●積分:∫[0,1]f(x)dx=F(x)|_[x=0,1], ∫[y=0,x]f(x,y)dy, ∬_[D]f(x,y)dxdy (← "∫"は「いんてぐらる」,"∬"は「きご
う」で変換可.)
●数列和・数列積:Σ[k=1,n]a(k), Π[k=1,n]a(k) (← "Σ"は「しぐま」,"Π"は「ぱい」で変換可.)
●極限:lim[x→∞]f(x) (← "∞"は「むげんだい」で変換可.)
●図形:"△"は「さんかく」,"∠"は「かく」,"⊥"は「すいちょく」で変換可.

●その他
・関数等の変数表示や式の括弧は,括弧()だけでなく[]{}を適当に組み合わせると見やすい場合がある.
・ギリシャ文字はその読み方で変換可.
・上記のほとんどの数学記号や上記以外の数学記号"⇒∀≠≧≒∈±≡∩∽"などは「きごう」で順次変換できる.
6ご冗談でしょう?名無しさん:04/03/24 18:37 ID:???
>>1
7ご冗談でしょう?名無しさん:04/03/24 20:37 ID:???
452 :名無しさん@4周年 :04/03/24 10:23 ID:KljJAAEC
>>440
すこしでも物理を知っている人間なら
大槻先生の偉大さを知っているんだけどね

http://news5.2ch.net/test/read.cgi/newsplus/1080011924/l50

お前ら少しも物理を知らないだろ
8ご冗談でしょう?名無しさん:04/03/24 22:57 ID:lnaZMPKq
空間て何ですか?
9ご冗談でしょう?名無しさん:04/03/24 23:30 ID:EY1obdZm
http://cgi.2chan.net/m/src/1080138173194.jpg
最下点(h2=0)での速度を求めたいんですが
自分なりの解き方で解いてみると
v^2-vo^2=2ax   (voは初速度)
↑この公式をつかって
a=gsinθ-μgcosθなので
v=√(2g(h2(1-μ)+h1)
となりました。
しかし答えを見てみると
v=√(2g(h2(1-μ)+2h1)
となってました
一体どこが違ってるんでしょうか?
どなたか教えてください
10ご冗談でしょう?名無しさん:04/03/25 01:02 ID:???
おお、過去ログが。すばらしい。
11ご冗談でしょう?名無しさん:04/03/25 01:08 ID:???
単なる計算間違いでしょ。
も一回チェックしてみたら?
129:04/03/25 01:11 ID:j2DDOn+n
>>11
何度もチェックしたんですが
計算違いじゃないみたいです
もしかして教科書のミスでしょうか?
13ご冗談でしょう?名無しさん:04/03/25 01:15 ID:???
>>9
あってるよ。答えがミスプリ。
そんな変な公式なんか使うから、自分の答えに自信がなくなるのでは?
位置エネルギー+初めの運動エネルギー−摩擦で失ったエネルギー=最後の運動エネルギー
って当たり前の式をたてて、計算すれば最後まで物理的な意味を見失わずに
計算できるよ。
14ご冗談でしょう?名無しさん:04/03/25 01:59 ID:???
変な公式使うなという>>13に賛成

あと、おかしいなと思ったら特殊なケースでやってみるとか。
今の場合だと摩擦なし(μ=0)でやってみる。この場合は
エネルギー保存の式もわかりやすいだろ。すると自分の答えに
自信が持てるから、教科書の答えのほうが間違ってると確信できる

159:04/03/25 02:35 ID:???
やっぱりそうなんですね
安心しました
ありがとうございました
16ご冗談でしょう?名無しさん:04/03/25 02:36 ID:???
>>14
摩擦なしって
μ=0じゃなくてμ=1じゃなかったっけ?
17ご冗談でしょう?名無しさん:04/03/25 02:45 ID:???
んなわきゃない
18四月から大学生(not物理関係):04/03/25 09:44 ID:q3p0wlzz
えっと、温度が高い、低いという事について一人で悶々と妄想してみたのですが

比熱が大きい物質A1gと比熱が小さい物質B1gでは、Aの方が温度が低くても
総熱エネルギー量が大きいという事が当然あり得るわけですよね。

エネルギー総量が大きいのになぜ、温度が低いという事になるのか。
それは、温度が高い→他者に熱エネルギーを渡しやすい状態、という定義になる、
ということなのですか?

Aはエネルギー総量が大きくても、他者にエネルギーを渡しにくいから「熱が低い」
状態でBはエネルギー総量が小さくても、他者にエネルギーを渡しやすいから「熱
が高い」と。

ということは、分子間力を無視するとすると、気体の比熱とか熱伝導率と分子量は
密接な関係を持つ、という訳ですかね?
19ご冗談でしょう?名無しさん:04/03/25 12:29 ID:YP3TLGV2
地上から数メートルのところに、地球を一周する鋼鉄の輪があり、
支えるものを一度にはずすと、どうなりますか
20ご冗談でしょう?名無しさん:04/03/25 12:42 ID:FQJO6bF9
その鋼鉄が地球のどの位置に配置されているかにもよりますが、輪がものすごく硬く、そして地球表面が真球に近い、と仮定しますと、運が良ければフラフープのように回る・・と思います。
現実的には輪の一部分が地表にめり込み、そのまま停止すると思います。
21ご冗談でしょう?名無しさん:04/03/25 12:47 ID:???
温度拡散率(熱拡散率)で検索すると理解の助けになるかも。
>>18の内容については、だいたい合ってると思いますよ。goodです。
温度の定義についても調べれば、完璧です。
熱力学(的)温度、統計力学的温度など。特に前者について。
22ご冗談でしょう?名無しさん:04/03/25 12:49 ID:???
>>19
どこかに過負荷がかかって折れます。
いや、どこかというよりいたるところが折れると思います。
23ご冗談でしょう?名無しさん:04/03/25 12:55 ID:???
剛体の時ですが
全質量をM 密度をρ(r) rを位置ベクトルとしたとき 積分区間をVとしたとき
M=∫_[V]ρ(r)d^3r
と書きますが
この
「d^3r」
とは何でしょうか?
d^3r/dt^3の上側だということはなんとなくわかるのですが。。。
24ご冗談でしょう?名無しさん:04/03/25 12:58 ID:???
鉄材って思ったより強くないんだよね。
それでなんど泣いたことか。
25ご冗談でしょう?名無しさん:04/03/25 13:27 ID:???
>>23
d^3r=dV=dxdydz
26ご冗談でしょう?名無しさん:04/03/25 13:31 ID:???
d^3r/dt^3の上側ではありません。
27ご冗談でしょう?名無しさん:04/03/25 13:31 ID:???
>>25
それは定義でいいんですよね

どうもありがとうございました
28ご冗談でしょう?名無しさん:04/03/25 18:16 ID:???
糸の一端を天井に固定し他端につるした質量mの小球を
水平面内で等速円運動させるとき。
小球に働くすべての力を、床の上に静止した観測者から見た場合と、
小球と一緒に回転する観測者から見た場合について図示せよという問題で

床に静止した場合は小球に働くちからは張力と重力だけで
遠心力は書かなくてもいいみたいですけど
これは一体なんでなんでしょうか?
29ご冗談でしょう?名無しさん:04/03/25 18:43 ID:bXS/gHfs
>>28
感覚的にわかりやすい説明をすると、遠心力まで書いちゃうと小球に働いている力は
すべて釣り合ってしまって、小球が円運動をしているという事実とつじつまが合わなくなっちゃうでしょ。
(力がすべて釣り合っている場合は、静止しているか等速直線運動しているかのどちらか)

上記を理解したものとしてさらに説明すると、等速円運動は加速度運動だから、
運動を続けさせるためには常に何らかの力が加わっていないといけない。
等速円運動の場合、この力は円の中心方向に向いていて、これを向心力という。
今の場合は、小球に働く重力と糸の張力との合力が向心力となって、小球は等速円運動を続けている。
小球に働いている力は、この「重力+張力」(=「向心力」)だけだ。

30つづき:04/03/25 18:44 ID:bXS/gHfs
小球と一緒に回転する観測者から見た場合、小球は静止して見える。
ということは小球に働いている力の合力は釣り合っていなければいけないはずだが、
実際には重力と張力だけであり、これら2つの力は釣り合っていない。
これはニュートンの運動の法則に反しているように見えるが、
これは観測者が小球と一緒に運動しているせいである。
ニュートンの運動の法則は静止もしくは等速直線運動をしている観測者についてのみ正しい。
いまの観測者は小球と一緒に等速円運動(=加速度運動)をしているから、
運動の法則が成り立たないのは当然のことだ。
ところが、加速度運動をしている観測者についても、その加速度運動を起こさせる力と
ちょうど相殺するような力を「仮想的に」付け加えてやれば、運動の法則は成り立つということがわかっている。
この、加速度運動している観測者にとっても運動の法則が成り立つように
「仮想的に」付け加えた力のことを「見かけの力」などという。
今の場合、等速円運動を起こさせるのに必要な力は、上で述べた向心力であった。
これとちょうど相殺する力は、向心力と大きさが同じで、向きが反対、すなわち円の外側に向く力だ。
実際、向心力は重力と張力の合力だったわけだから、これと反対向きで同じ大きさの力が
小球に加わっていると考えれば、これはちょうど重力と張力の合力と釣り合い、小球に働く力は釣り合っているように見え、
小球が静止しているという観測とつじつまがあう。

このような、等速円運動をしている観測者にとっての「見かけの力」のことを「遠心力」という。
遠心力を考えるのは、加速度運動している観測者についても運動の法則が成り立つようにするためであって、
静止している観測者についてはそのような力を考える必要はない。

31四月から大学生(not物理関係):04/03/25 23:02 ID:q3p0wlzz
>>21
ご指導ありがとうございます。早速お教え頂いたキーワードで検索してみることにします。

物事の「何故?」を考えて、それを説明してみようとするのってやっぱり楽しいですね。
物理は専門ではないのですが、物理自体は好きなので、これからも勉強していこう
かと思っています。

ありがとうございました〜
32ご冗談でしょう?名無しさん:04/03/25 23:14 ID:???
>>29
>>30
ご丁寧なご解説大変助かりました
どうもありがとうございました
33ご冗談でしょう?名無しさん:04/03/26 10:49 ID:54NT/K6G
すんごい初歩的な質問です。

SPIでよく「AからBへ行くときの平均時速をx、BからAのときをyと
すると、往復の平均時速は

1/z = 1/2 * (1/x + 1/y)

と書いてありますが、 z = (x + y) / 2
ではダメなんでしょうか?DQNな質問ですが、
解説お願いします、エロイひとm(__)m
多分、流水算とか仕事算と同じような考え方
がベースになってるとは思うのですが・・・。
34ご冗談でしょう?名無しさん:04/03/26 11:19 ID:???
>>33
速さの定義からちゃんと考えてごらん。
往復だから、行きと帰りで移動距離は同じだけど…
(あとは自分で計算してみよう)
35ご冗談でしょう?名無しさん:04/03/26 11:23 ID:???
>>33
板違いだけど
平均時速について考えていて訳分からなくなったら,常に 時速×時間=距離の式に戻って考える。

往復の平均時速は 往復距離/往復時間 になる。
片道の距離を d とおくと,行きの時間は d/x ,帰りの時間は d/y だから,
往復時間は d/x+d/y だ。
往復距離は 2d だから,
往復の平均時速は z = 2d/(d/x+d/y) = 2/(1/x+1/y)
よって 1/z = 1/2 * (1/x+1/y) だ。

平均といったらいつでも相加平均 (x+y)/2 で良いわけではなくて,
この例のように調和平均 1/2*(1/x+1/y) を使うこともあるし,
平均成長率なんかは相乗平均 √(xy) を使う。
36ご冗談でしょう?名無しさん:04/03/26 11:29 ID:ho5bIzPn
お願いっす。

下文中の(  )内に入れる数値として、正しいものは(1)〜(5)のうちど
れか。
 
 「事務室において、在室人員が20人、外気の二酸化炭素(炭酸ガス、CO2)
濃度が0.03%であるとき、この事務室の必要換気量は約(  )m3/hであ
る。
 ただし、計算には下式を用い、式中の室内CO2基準濃度は0.1%、呼気中
のCO2濃度は4%、1人当りの呼気量は毎分10リットルとする。

              在室者の1時間当りの呼出CO2量(m3/h)
 必要換気量(m3/h) = ────────────────────
             (室内CO2基準濃度)-(外気のCO2濃度)     」
  
(1) 343
(2) 549
(3) 686
(4) 857
(5)1143
37ご冗談でしょう?名無しさん:04/03/26 12:28 ID:ZmqV6WRM
質問です。
今日道を歩いていると「光速に近づくにつれ、物体の質量は増加する」という声が天から降ってきました。
にわかにはその意味がわかりませんでした。
しかしふと、オリンピックに出るような短距離選手を思い出しました。
短距離選手は体中が筋肉モリモリで体重も重そうです。
僕よりも光速に近くそして質量も重いです。
つまりこういう事でしょうか。
38ご冗談でしょう?名無しさん:04/03/26 12:59 ID:???
そういうことです。
39ご冗談でしょう?名無しさん:04/03/26 14:00 ID:???
>>36
物理の問題じゃない、ただの算数だろ。

20[人]×10[g/(人・分)]×60[分/h]×4×0.01×1/1000[m^3/g]
--------------------------------------------------------
0.1/100 - 0.03/100

40ご冗談でしょう?名無しさん:04/03/26 15:28 ID:???
質量mで高さhにある物質に働く位置エネルギーがmgh
ってことは地球から無限遠方にある物質の位置エネルギーは
無限大ってことですか?
41ご冗談でしょう?名無しさん:04/03/26 15:36 ID:???
mgh、それは近似
hが地球半径に比べてかなり小さくないと誤差が無視できなくなります
4237:04/03/26 16:41 ID:ZmqV6WRM
ではいきなり僕はアインシュタインの域に達したのですか?
43ご冗談でしょう?名無しさん:04/03/26 16:43 ID:???
そういう事です
44ご冗談でしょう?名無しさん:04/03/26 17:01 ID:Wq976VOt
今度の学会で発表しる
45ご冗談でしょう?名無しさん:04/03/26 17:14 ID:OU5feUvJ
質問いいですか?

今TVのニュースで火事を報道してたんですが。
「バケツリレーをして消化活動をした」といっていました。

前から疑問に思ってたんですがバケツリレーって効率いいんでしょうか?
学識のない自分にはどうしても、同じ人数である場所からある場所へ水をバケツで移動させる場合、
バケツリレーより、それぞれが水を汲んで移動するほうが効率がよいようにおもんですが。どうでしょうか?
46ご冗談でしょう?名無しさん:04/03/26 17:36 ID:???
でも疲れてくるだろ。
47ご冗談でしょう?名無しさん:04/03/26 17:37 ID:???
>>45
物理って言うより人間の行動様式の問題

取水口が少なくばらばらに水を汲んで移動させた場合
よっぽど訓練された集団で無いと水を汲む時間のロスが増える
水道のまえで団子になっている状態とか
また長時間、人間が走って往復し続けると疲労するので効率が悪くなる

もちろん、すでに全てのバケツに水が汲んであって一人一個ずつ持って移動するだけなら
いっせいに動き出した方が速い
48>>45:04/03/26 17:42 ID:???
>>47
なるほど、解決しました。
水場で人が溜まるってのは自分も考えてたんですが、それより往復することでの疲労ってのに気が付きませんでした。
これはかなり重要ですよね。
機械じゃなく人間がやるんだもんねえ。
ありがとうございます。
49ご冗談でしょう?名無しさん:04/03/26 20:53 ID:???
トンネル効果を理論的に解説できる方法を考えたのだけれども、こういう場合、どこで話すべき?

2ちゃんねるで書いて、(まあ、本来は無用の心配だけれども)万が一、本当にできることだったら、やばくないか?
いい方法ありますかね?
50ご冗談でしょう?名無しさん:04/03/26 20:56 ID:???
ネタは嫌いでつ
51ご冗談でしょう?名無しさん:04/03/26 20:58 ID:???
>>49
本気で発表したいなら、論文をお書きなさい
レビュアー付きのジャーナル(PRLとかPLxとか)がbestだが、
とりあえずe-printアーカイブに送っとけば証拠にはなるだろう


ていうか、トンネル効果が理論的に解説されていなかったとでも?
52ご冗談でしょう?名無しさん:04/03/26 21:06 ID:???
いや、トンネル効果を調べてみたものの、自分の言うのとは少し違うんだが・・・

ついでに言うと、論文に表すほど行数を必要としない、単純な解説で、かつ明確にいえるものなのだが・・・

だからどうしようかと。
53ご冗談でしょう?名無しさん:04/03/26 21:46 ID:???
>>52
何が違う?起こる現象が違うのかい
じゃ、明確に間違ってるんだよ(鼻くそほじりながら)

PRLの頁数を知ってるかい?たった6Pだ。
どうしてもペーパー書きたくないんだったら、秋の学会ででも発表したら?
「と」仲間の会員が二人いれば、君も立派な物理学会会員だw
54ご冗談でしょう?名無しさん:04/03/26 21:54 ID:???
オーケイ。
ま、名前を売る気もないから、ここで合っているかどうか質問させてもらうかな。

その前に1題。

-1の指数関数は、どんな図形を描くか。

これに答えられないようなレベルでは質問する意味がないので。
55ご冗談でしょう?名無しさん:04/03/26 22:07 ID:???
構って欲しいなら、そう言えばいいのにw
てか"-1の指数関数"って何よ? (-1)^e? e^(-1)?
56ご冗談でしょう?名無しさん:04/03/26 22:10 ID:???
まさかと思うが(-1)^xの事か?
わざわざこれを質問する意図がさっぱり不明なのだが…
57エセ物理師  ◆QjlwIfyAts :04/03/26 22:12 ID:???
(ー1)^xのことだが・・・eって何だよ。
まあ、それを答えてからだな。

どんな図形を描くだろうか?
58ご冗談でしょう?名無しさん:04/03/26 22:29 ID:???
>>57
eも知らずに指数関数とほざいてたのかよ
59ご冗談でしょう?名無しさん:04/03/26 22:33 ID:???
>(ー1)^xのことだが・・・
ネタ決定だな
60エセ物理師  ◆QjlwIfyAts :04/03/26 22:34 ID:???
いや、自然対数。。。というか、ネピア数が、なぜここで出てくるのかな、と。

それはともかく、(-1)^xの描く軌跡は何か?

意味はあるんだがな。
61みきちゃん:04/03/26 22:49 ID:???
くけいはー
62ご冗談でしょう?名無しさん:04/03/26 22:57 ID:???
>>60
意味があると思うんなら説明してみろよ
63ご冗談でしょう?名無しさん:04/03/26 23:08 ID:54NT/K6G
>>34
>>35
平均速度について質問した者です。
お答えありがとうございます!凄い分かりやすい
解説でした。。自分はSPIとか中学入試でありそうな
流水算や仕事量の問題っぽく、

(1/x) = 速度xあたりに1単位の距離を進むときの時間

と考え、片道にかかった平均時間の等式として

1/z = (1/2) * (1/x + 1/y)

が成立すると自分に納得させていました。考えてみたら
距離を具体的にdとおいてもこれは成立する筈ですよね・・・。
速度の定義からの考えで理解できて、すっきりしました。
ありがとうございました(__)m
64ご冗談でしょう?名無しさん:04/03/26 23:11 ID:???
>>60
答えてやらん事もないが、その前に
ln(-1)を定義してもらえまいか
65エセ物理師  ◆QjlwIfyAts :04/03/26 23:17 ID:???
y=(−1)^x のグラフは、x軸とy軸に垂直な方向へ虚軸をとると、
x軸を中心とした螺旋の図形を描く。
もし、ここに於いて、y軸に実線を引くならば、正の数が底の指数関数は、
これに触れることなくグラフを書くのは不可能であるよ。
しかし、y=(−1)^x+0.5 のグラフは、螺旋運動をしながら
これに触れることなくすり抜けることが可能であると。
この理論より、シュレディンガーの公式にも、虚数関数があるように、
z軸の代わりに虚軸を用いて考えると、(実際には、x、y、z軸に対して垂直に引くことができればいいのだが)
x、y平面上の三次元の物質とは違って、虚軸方向にも運動できる電子は、
x−y平面上(本当は、x−y−z立体上)の物質を飛び越えることが可能であると。
要は、トンネル効果というのは、三次元の障害物を、電子が虚軸方向からよけて通るという現象である、とも。
ついでにじゃ。
こういう風に考えれば、光より早く通過するのではなく、入る前の電子と別の電子が観測されたと考えることができると。
66エセ物理師  ◆QjlwIfyAts :04/03/26 23:18 ID:???
で、この理論だが、どう思う?
67エセ物理師  ◆QjlwIfyAts :04/03/26 23:20 ID:???
この理論が正しいのなら、超伝導の仕組みも明確に解説することができるのだが・・・いかがなものか。
68ご冗談でしょう?名無しさん:04/03/26 23:20 ID:???
69ご冗談でしょう?名無しさん:04/03/26 23:26 ID:???
また春厨の季節か
70ご冗談でしょう?名無しさん:04/03/27 03:07 ID:???
コンパクトな空間の宇宙を初期条件として用意して、
相対論的に時間発展させることで非コンパクトな空間の宇宙になる解は
ありますか?
71ご冗談でしょう?名無しさん:04/03/27 07:15 ID:???
>>65
くだらんネタでスレを消費したくないので
あとは自分一人で数学のお勉強でもしておけ
http://akademeia.info/main/math_lecturez/math_log.htm
72エセ物理師  ◆QjlwIfyAts :04/03/27 09:02 ID:???
くだらないネタ?
示したリンク先が何の当てにもならない高校レベルの指数対数関数のお勉強で、
何の意味があるのやら。

ものすごい単純なことなのに、どこが間違っているか解説もできず、
合っているかどうかも判断できない。

所詮、俺よりもひどいエセ知識を振りかざして満足に浸るだけの、独りよがりのスレッドだったって訳か。
いや、ほんま。
73ご冗談でしょう?名無しさん:04/03/27 09:06 ID:???
早く続きが見たい。
74ご冗談でしょう?名無しさん:04/03/27 10:34 ID:???
もし地球の自転が一瞬で止まったら人間はどうなるんですか?
75エセ物理師  ◆QjlwIfyAts :04/03/27 10:45 ID:???
>>74

慣性の法則により、あらゆるものが吹っ飛びます。
76ご冗談でしょう?名無しさん:04/03/27 11:05 ID:???
荒らしが住み着いたようですね
77ご冗談でしょう?名無しさん:04/03/27 11:58 ID:???
>>75
そのまま宇宙にふっとびますか?
78ご冗談でしょう?名無しさん:04/03/27 13:09 ID:???
>>65
>y=(−1)^x+0.5
 これだとy軸にぶつかるなw y=(−1)^(x+0.5)だろ?

 細かいつっこみはさておき、実数しか見ない、つーのはせいぜい工房までで許される甘えだ。
ちゃんと複素数全体にそれなりの意味を見出せなけりゃ理論としての体をなしてないわな。

 まぁ妄想にふけるのもほどほどのところにしておいて、ちょいとまじめに勉強しておくんだね。
79ご冗談でしょう?名無しさん:04/03/27 13:37 ID:???
スピンハミルトニアンで教科書を見ると
H=-J (S1・S2) とH=-2J(S1・S2)
の二通りの表示があるのですが, これはJの定義が違うだけ
でしょうか?これが違うとシングレット・トリプレットの
エネルギー差もJと2Jと異なってしまいます。
80ご冗談でしょう?名無しさん:04/03/27 14:37 ID:8MnqBCM/
>>78
Jの定義ではなくSの定義とみた。
Sって時々-1,+1の値をとるとされてるときアルでしょ。
そうするとパウリのスピン行列をそのまま使えたり、積がやはり-1,+1だったりいろいろ便利なんだよ。
単に比例係数の違いなんだけど。
で、そういうのはちゃんと教科書のどこか、その式がでてくる前に書いてあると思うから探してみ。
81車屋:04/03/27 15:05 ID:lOplkFvH
分子の原子間の距離を知りたいのですが、
関数6-31++Gについて何かヒントか良い本など
知っていたら教えてください。
82ご冗談でしょう?名無しさん:04/03/27 15:21 ID:???
>>52
>トンネル効果を調べてみたものの、自分の言うのとは少し違うんだが・・・
つまりおまいのいうトンネル効果はいわゆるトンネル効果とは違うってことだろ

>>72
>ものすごい単純なことなのに、どこが間違っているか解説もできず
おまいの脳内トンネル効果がどんなものかわからん以上、どこが間違ってるかなんて
解説しようがないやんけ

83ご冗談でしょう?名無しさん:04/03/27 15:22 ID:E88+zSYi
http://cgi.2chan.net/up2/src/f51778.jpg
参考書に書いてあることがよくわかりません。
問題は解けますが「参考」の部分の意味がわかりません。
お願いします。
8479:04/03/27 15:52 ID:???
>>80
ありがとうございます。Sの定義に着目してみると,
固体物理の一般的な教科書ではちゃんと書かれてな
かったりして困りました。まあ拾い読みではダメで
ちゃんと式を追えってことですけど。イバッハの
固体物理学ではsingle-tripletのエネルギー差をJ
として交換定数を導入してスピンハミルトニアン
をH=-2J(sigma1・sigma2)とパウリ行列で表現してました。
なのでやはりSの定義によるようですね。
ありがとうございました。
85ご冗談でしょう?名無しさん:04/03/27 15:59 ID:???
>>83
Idw/dt=Nがどの座標系のものなのかはっきりしろということだ。
時計の針のように棒が回転する運動は固定点のまわり。
その問題の頭の3式の一番下の式は円板の重心のまわり。
最後に書いてあるのは、重心ではなく円板と斜面の接触点のまわりを選んだ場合。
86ご冗談でしょう?名無しさん:04/03/27 17:00 ID:a23nGawT
バックトゥザフューチャーの第一作で、ドクはマーティの手紙を破いたのに、
なんで未来では破けてなかったんですか?
8783:04/03/27 17:41 ID:E88+zSYi
http://xtp0001.s3.x-beat.com/cgi-bin/up/source/Sonata_7689.jpg
なんか消えてしまったので
>>85
ありがとうございます
それではなぜ接触点まわりだと
Mgl*sinθ=(1/2)Iω^2
としてよいのでしょうか?
88ご冗談でしょう?名無しさん:04/03/27 17:48 ID:joN0h9+O
電界と磁界の関係を抽象的にいうならば何でしょうか?(難しい数式は苦手なので)
磁界は電子の回転と関係あります?
89ご冗談でしょう?名無しさん:04/03/27 18:43 ID:iArc1zox
「点」って何次元にあたるのですか?
それとも全ての1次元、2次元、3次元において、全ての
0を意味するものなのですか?
90ご冗談でしょう?名無しさん:04/03/27 20:23 ID:???
>>86
未来ではセロテープで補修してる。

>>88
コインの裏表

>>89
点は0次元
91ご冗談でしょう?名無しさん:04/03/27 22:06 ID:???
>>89
次元の意味考えろよ、わかるだろ
92ご冗談でしょう?名無しさん:04/03/27 23:12 ID:UHXyMQki
「物理数学の直観的方法」の第2集が出たんですが
さて、物理の専門の方々の評価はどんなもんなんでしょうか?
なんか、たくさん読まれているというハナシなんですが。。。
93ご冗談でしょう?名無しさん:04/03/28 01:14 ID:rgaMPJBo
ここのページのカモの動きってどんな物理式??

http://www5e.biglobe.ne.jp/~hirohide/
94ご冗談でしょう?名無しさん:04/03/28 01:48 ID:???
>>93
javascriptだからソースを見ろ
95ご冗談でしょう?名無しさん:04/03/28 03:08 ID:70OR7HYe
>>90
「セロテープ」はニチバンの登録商標だろが!
 あれはメリケンの物語であるからして、補修してあるのは「セロテープ」
ではないのだよ。
 じゃあ何だよっっって聞かれても困るけどな...
96ご冗談でしょう?名無しさん:04/03/28 04:28 ID:pA/s+fWh
別スレで尋ねてみたのですが、返答されそうにないので、ここにも書きます。

(問題)半径60cm、抵抗10^6Ωの一巻きの円形コイルの軸上に、
コイルの中心O(オー)から80cmの距離Pに、強さ100Ωの点磁極がある。
このコイルに向って磁極を近づけ、点Oから軸上45cmの所に移すとき、
コイルに流れる全電気量は何クーロンか?

これは北杜夫「どくとるマンボウ青春期」中公文庫 に出てくる物理の
問題ですが、式と正解を教えてください。
ちなみに、この問題に対して、当時旧制松本高校生だった斎藤宗吉(北杜夫の
本名)が書いた答案は。。

(答)電磁感応ニヨリ、こいるニ電気が流レルガ、コレハ最モヨク知ラレタ公式
ニヨツテ式ヲ立テ(物理学ノ本ヲ見ラレタイ)、上記ノ数値ヲ代入シ、多少数学的ニ
コレヲヒネクリマハスコトニヨリ、答ヲ得ルハ容易ナルコトデアル。
先生自ラコレヲ試ミラレタイ。

です。
97ご冗談でしょう?名無しさん:04/03/28 11:16 ID:???
ホントくだらない疑問です
ここには教科書を書いている教授方もいらっしゃるでしょうが
いったい印税はどれくらい頂いているのですか?
額じゃなくて既婚男性の妻からもらうおこずかいとかそんな感じの比喩でお願いします
98ご冗談でしょう?名無しさん:04/03/28 11:25 ID:???
>>97 クイズ$ミリオネアで10問正解した際にもらえる金額くらい。
99ご冗談でしょう?名無しさん:04/03/28 11:55 ID:I5NBE6RZ
過去は存在してるんですか?
過去は確かにあったけど今現在に過去は過去と言う形で存在してるんですか?
100DQn:04/03/28 12:05 ID:???
>>99
因果律を認めて、因果に掛かる定量を基準に考えれば、
距離が決まるので、それを時間と呼んでもいいですよ。
101ご冗談でしょう?名無しさん:04/03/28 12:32 ID:???
>>96
磁荷を近づける過程で、途中磁荷の速度がどう変化しても
コイルに流れる総電気量は変わらないことを踏まえて、
距離が80cmの時と45cmの時のコイルを貫く磁束の
差を求めれ。
102ご冗談でしょう?名無しさん:04/03/28 13:38 ID:ZOOhJC80
>>88
なぜ地球には磁界が発生するのでしょうか?また、地球には電界も発生しているということでしょうか?(さらに磁界と電界ではどちらが先に発生するのでしょうか?)
103ご冗談でしょう?名無しさん:04/03/28 17:38 ID:sqSo9RHh
今私はこの瞬間どの方向にどのぐらいの速度で動いているんですか?
基本的知識のない文系金融向けソフトエンジニアです。
PCの前で静止しているが、地球は自転してるし、公転してるし、銀河はまわるし、宇宙は広がるし.
またこれを計測するすべはあるんですか?
104ご冗談でしょう?名無しさん:04/03/28 17:41 ID:???
>>103はいつでも静止している。自己中。
105ご冗談でしょう?名無しさん:04/03/28 18:08 ID:???
速度っていうのは相対的なものです
好きな基準を決めて測ってください
異なる基準で測った速度同士を変換して比較するときは
相対性理論を参考にすると正しく変換されるので安心です
106ご冗談でしょう?名無しさん:04/03/28 22:03 ID:RvtaqQOE
公務員試験の問題なのですが。ご教授ねがいます。

腕を一回転させると高さがhだけ上昇するネジ式ジャッキがあり、
質量mの物体が載せてある。
ジャッキの中心からの距離がrである腕のA点およびB点に力を加え、
物体が下がらないよう保持したい。
A点およびB点の力の大きさが同じであるとき、
1箇所あたりどれだけの力を加えればよいか。
ただし、ジョッキの可動部の質量と摩擦は無視できるとする。

107ご冗談でしょう?名無しさん:04/03/28 22:14 ID:???
バカが公務員を目指す世の中なんとかならねーのか?
108ご冗談でしょう?名無しさん:04/03/28 22:31 ID:???
>>106
滑車の原理で、一周(2πr)回してh上がるから、その比率で力も
分散するわけで、結局 mg≦2F・2πr/h を解けば良い。
違ったらごめん
109106:04/03/28 23:15 ID:RvtaqQOE
>>108 さん ありがとうございます。なるほど分かりました。

ところで、
私は力のつりあいを考えようとして頓挫したのですが、
力のつりあいでは解けないものでしょうか?
110ご冗談でしょう?名無しさん:04/03/28 23:54 ID:wxjw9Pnv
ニムペーパーとは何ですか?
111ご冗談でしょう?名無しさん:04/03/29 00:07 ID:???
>>109
 解けない。ジャッキを支えている地面からの力が出ないから。
112ご冗談でしょう?名無しさん:04/03/29 00:30 ID:???
>>102
もしかすると、「マグマの対流による摩擦によって静電気が発生し、そのため電界が発生することになるので磁界が発生する」ということでしょうか?
113ご冗談でしょう?名無しさん:04/03/29 01:11 ID:???
>>102 >>112
仮説ではダイナモ理論が有名。
114ご冗談でしょう?名無しさん:04/03/29 02:05 ID:???
xyz空間で円柱x^2+y^2=4,xy平面、放物面z=x^2+y^2で囲まれた領域をDとして、
Dの境界をSとする。F=y・i+xy・j-z・kとする。(i,j,kはx軸y軸z軸方向の単位ベクトル)
2)発散定理を用いて、ベクトル場Fの曲面Sを貫く外向きの流束(flux)を求めよ。

この問題の求め方がわかりません
どなたか教えてください
115ご冗談でしょう?名無しさん:04/03/29 09:11 ID:???
発散定理を用いれば、流束は div F の体積積分として求められる。
積分の計算は、円柱座標を用いると簡単になるだろう。
116ご冗談でしょう?名無しさん:04/03/29 19:04 ID:???
円形のディッシュに液体(水など)をある程度の量満たして、
なかに小さな粒子を入れ、ディッシュを回転運動させた後に停止させると、
粒子はディッシュのどの位置にとどまるのでしょうか?
流体力学の専門家の方、よろしくお願いします。
117ご冗談でしょう?名無しさん:04/03/29 19:17 ID:???
>>116
そんなの液体と粒子の性質によるのでは?
そもそも、なぜ容器とか皿とか言わずにディッシュ?
一瞬何かと思ったよ。
118ご冗談でしょう?名無しさん:04/03/29 19:26 ID:???
>>117
生物の実験で、細胞を継代して撒くときに、
ディッシュに円を描く運動をさせると細胞が中央に固まってしまうので
前後左右に動かすようにしろ、といわれています。実際そうなのですが、
なぜそうなるのかがよくわからないのでここで聞いてみたのですが。。
119ご冗談でしょう?名無しさん:04/03/29 20:54 ID:???
>>115
まだよくわからないんですが
Fの発散を求めて
それと円柱の体積と掛け合わせればいいってことですか?
つまり
∫[θ;0〜2π]∫[r;0〜2]r^3・divF drdθ
これであってます?
120ご冗談でしょう?名無しさん:04/03/29 21:03 ID:???
電圧をポテンシャルというのはおかしいよね。ポテンシャルといえばエネルギーっしょ。mghのような。
121ご冗談でしょう?名無しさん:04/03/29 21:25 ID:???
>>120
おかしくないよ。
122ご冗談でしょう?名無しさん:04/03/29 21:32 ID:???
>>121
だって、仕事はVIじゃん。Iは抵抗で決まるわけでしょ。でもmghは他の何にも依存せずずそれだけでエネルギーじゃん。電圧がポテンシャルっちゅうのはねぇ。
123ご冗談でしょう?名無しさん:04/03/29 21:35 ID:???
丁半ばくちでずっと例えば50回、丁が続いたとします。
すると次の回、丁が出る確率はずっと減ってるんでしょうか。
やっぱり2分の一なんでしょうか?
124ご冗談でしょう?名無しさん:04/03/29 21:42 ID:???
>>123
もちろん減ります。ナイチンゲールの法則(ばくちは最後までやり続ければ必ず勝てる法則)を勉強してみて。もしも変わらないなら、パチンコやギャンブルで負けているときに、「次こそは」と言いつつ深みに嵌るような人はでないじゃんか。
125121:04/03/29 21:42 ID:???
>>122
厳密には電圧というより電位がポテンシャル。
それからポテンシャルの定義をきちんと理解した方がいいよ。
126ご冗談でしょう?名無しさん:04/03/29 21:44 ID:???
ポテンシャルの定義って何?
127ご冗談でしょう?名無しさん:04/03/29 22:15 ID:???
>>123
 減りません、逆に増えます。なぜならそのさいころがいかさまである可能性が高まるからです。
128ご冗談でしょう?名無しさん:04/03/29 22:18 ID:???
謎は、解けた!
129ご冗談でしょう?名無しさん:04/03/29 22:43 ID:???
>>126
教科書嫁
130ご冗談でしょう?名無しさん:04/03/29 22:44 ID:???
>>120
ポテンシャルエネルギーなら間違いなくエネルギーだけど
単にポテンシャルと言った場合はエネルギーとは限らん
131119:04/03/29 22:47 ID:???
だれか〜〜〜
132121:04/03/29 23:01 ID:???
>>131
119は間違っている。
ベクトル解析と重積分をやりなおせ。
133119:04/03/29 23:13 ID:???
>>132
すいません
どの変がまちがってるのか
おしえてください
134ご冗談でしょう?名無しさん:04/03/29 23:18 ID:4vjb5QAJ
よく保存力の判定としてつかわれてる
∂Fx/∂y=∂Fy/∂x→F保存力
(Fx:力のx成分、Fy:力のy成分)
ってどうしていえるんですか?
逆がいえるのはわかるんですが。
135真面目:04/03/29 23:21 ID:rjK8q3CS

   ワープ理論的に可能ですか?
    最新のお願いします。




136ご冗談でしょう?名無しさん:04/03/29 23:31 ID:???
>>116-118
重いものが外に、軽いものが中に
回転運動したら分離しちゃうだろう
137ご冗談でしょう?名無しさん:04/03/29 23:44 ID:???
>>133
∫∬divF・rdθdzdr で計算する。
放物面z=x^2+y^2がz=r^2と表されることを考えればzの積分範囲はわかるだろう。
138真面目:04/03/30 00:05 ID:???
139ご冗談でしょう?名無しさん:04/03/30 02:02 ID:???
>>134
ベクトルFについてrotF=0ならF=−gradUとなるような
スカラーUが存在するという定理があるのよ。証明はベクトル
解析の教科書でも読んでちょ。
(ちなみに>>134の∂Fx/∂y=∂Fy/∂xはrotF=0のz成分ね)

一方、力FがスカラーUを使ってF=−gradUと書けるというのは
Fが保存力であるという定義そのものだから、FがrotF=0を
満たせばFは保存力ってことになる。
140ご冗談でしょう?名無しさん:04/03/30 07:48 ID:???
ポテンシャルは何か仕事をしそうな雰囲気ってこと。
141134:04/03/30 11:25 ID:???
>>139
そんな定理があるんですか。調べてみます。
どうもありがとうございました
142ご冗談でしょう?名無しさん:04/03/30 11:50 ID:???
143ご冗談でしょう?名無しさん:04/03/30 17:12 ID:VR27ZN5F
全く板に不似合いな質問で悪いですが、
宗教板で0≒∞と言われました。
http://etc.2ch.net/test/read.cgi/psy/1071414850/l50#tag757
もしそうなら死ぬのも怖くないなぁと。
0≒∞ですか?

144ご冗談でしょう?名無しさん:04/03/30 17:40 ID:???
>>143
どういう意味で≒を使っているかによる
145ご冗談でしょう?名無しさん:04/03/30 18:15 ID:???
どだげって言われても返答に困る
146ご冗談でしょう?名無しさん:04/03/30 21:11 ID:???
俺的には8≒∞
147ご冗談でしょう?名無しさん:04/03/31 01:26 ID:???
何でLC回路って勝手に電流が振動するんですか?
なぜ振動するのかなかなかイメージできないんですけど
どう考えればいいですか?
148ご冗談でしょう?名無しさん:04/03/31 01:45 ID:???
>>120
オームの法則は直感的に分かりにくいので、重力場に置き換えるとわかってくる。
質量と電荷量を置き換えて考えると以下の通りとなる。
電場で、電荷量qに働く力はF=qEとあらわせる。重力場では質量mに働く力はF=mgとあらわせるので、
電界Eは電場での重力加速度と考えることができる。
電界はV/h(hは距離)であり、g=V/hと考えることでV=ghとなる。
つまり、電圧は重力加速度に高さを掛けたものである。
mghが仕事であることを考えれば、
gh(=V)は単位電荷当たりのエネルギーであり、これは(単位電荷当たりの)ポテンシャルで良いですよね。
また、仕事WはW=VItなので、VIt=mgh→It=m→I=m/tとなるので電流を重力場で考えると時間当たりの質量の変化となり訳がわからなくなってしまう。
ちなみに抵抗はV=IR→mg=m/t×R→R=gtとなり、重力加速度に時間を掛けたものであることが分かる。
つまり、重力に従って落下した際の速度を意味する。
う〜ん、いまいちでしたか?
149ご冗談でしょう?名無しさん:04/03/31 11:07 ID:???
単振動のイメージ
150ご冗談でしょう?名無しさん:04/03/31 11:33 ID:???
>>148
前半はまあいいだろうが、後半は?だ。
電界を重力場に置き換えて考えるなら、電流に相当するものは連続的な
物体の移動、たとえばパイプの中の水の流れなどだ。
電流の強さは単位時間当たりに移動する電気量で表されるから、重力場の
場合に電流の強さに相当するものは、単位時間当たりに移動する物質の
質量たとえば水の流量などだ。つまりI=m/tは時間当たりの質量の変化
というよりも時間当たりの移動質量と考えるべきだ。
また、V=IR→mg=m/t×R→R=gtは間違いだ。
V=IR→gh=m/t×R→R=gth/mが正しい。
しかし、この式のままではRの意味がわかりにくい。
むしろm/t=gh/Rとしたほうがわかりやすい。
水の場合、左辺は単位時間当たりの流量だから、それがRに反比例することになる。
つまりRは水の流れにくさを表している。
たとえばパイプが細くなるほど水が流れにくくなるのでRが大きくなる。
さらに流量がghに比例していることは次のように解釈できる。
たとえばパイプの傾きを大きくするとghが大きくなり、パイプの両端の圧力差
が大きくなって流量が増加する。これは電圧が高くなると電流が増加することに相当する。
151ご冗談でしょう?名無しさん:04/03/31 18:19 ID:RC8YzQiv
火星にメタンや石油があると聞いた事があるんですが、
化石燃料である石油などの資源が、生命も無しに存在
しうるものでしょうか?
152ご冗談でしょう?名無しさん:04/03/31 18:21 ID:FytKvH5V
硝子の表面や鏡面化されたアルミの表面を電子顕微鏡で見たのですが
ぼやけてしまって?よく見えませんでした.なんででs?
153ご冗談でしょう?名無しさん:04/03/31 18:25 ID:???
>>151
石油があるなんて誰が言ってるんだ??
154ご冗談でしょう?名無しさん:04/03/31 18:26 ID:???
>>151
どこで聞いた? そんなヨタ話
155ご冗談でしょう?名無しさん:04/03/31 18:29 ID:???
>>151
石油・石炭・天然ガスなどの資源は化石燃料では無く、本当は宇宙起源
の炭化水素で、これらは地球生成時に地中深くに閉じ込められた物が、
プルームテクトニクスにより地殻まで押し上げられる際、生物起源の物質
をほんのちょっぴり溶かし込んだために、生物起源というように認識され
てしまったが、本来は地球に限らず他の惑星でも、石油・石炭・天然ガス
などの資源は炭化水素として地中深に閉じ込められている可能性は高い。
156ご冗談でしょう?名無しさん:04/03/31 18:37 ID:???
>>155
オイオイ、4月1日はまだだよ
157ご冗談でしょう?名無しさん:04/03/31 18:52 ID:RC8YzQiv
>>153-154
以前、『諸君!』という論壇誌を立ち読みしていた時に、
学びの最先端という特集に書いてありました。

>>155
そうですか、化石燃料という認識は完全に正しいとも言えないんですね。
たしか、トーマス・ゴールド氏の地下深層ガス説とか言うので聞いた事があります。
どうもありがとうございました。

158ご冗談でしょう?名無しさん:04/03/31 20:28 ID:YcIJtjgk
こんな問題出たのですが解りますか?

ある一定の速度nkm/hで移動する重さ300gの物質がある。この物質は一秒間に365日進んでいる。
この物質の速度を求めよ。

お願いします
159ご冗談でしょう?名無しさん:04/03/31 20:29 ID:3MsezmwY
4月から、電磁放射線(ガンマ線)を使用してる仕事に就くのですが、人体に及ぼす影響はどのくらいのものですか?
160ご冗談でしょう?名無しさん:04/03/31 20:52 ID:???
>>158
>この物質は一秒間に365日進んでいる
この部分の意味がわからん
161ご冗談でしょう?名無しさん:04/03/31 20:54 ID:???
>>159
放射線従事者になるなら教育訓練が法律で義務づけられているから、
そのときにいやでも習うよ
162ご冗談でしょう?名無しさん:04/03/31 23:05 ID:dkvw4YHn
看板は誰が考えた物なのですか?
あの看板 すきです。
163ご冗談でしょう?名無しさん:04/04/01 04:42 ID:???
>>158
1秒間に365日ってことは
365*24=8760
1秒間に8760時間進んでいる。
これが1時間だと8760*60*60=31536000
すなわちこの物質は1時間に31536000時間すすんでいるといことは
時速31536000*n(km) で進んでいるということだ。
そんな馬鹿な。
164ご冗談でしょう?名無しさん:04/04/01 10:50 ID:???
この物質の固有時間で1秒の間に、外部の基準系の時間が365日進んでいるということかな?
165ご冗談でしょう?名無しさん:04/04/01 11:17 ID:???
http://private.rocketbeach.com/~mms1617/2ch/upload/dat/0052.jpg
Dフリップフロップ2つが直列に接続されている場合
下のタイムチャートを完成させよという問題をやってみたら
http://private.rocketbeach.com/~mms1617/2ch/upload/dat/0051.jpg


http://private.rocketbeach.com/~mms1617/2ch/upload/dat/0053.jpg
↑こうなったんですが
これってあってますか?
自信が無いので誰かおしえてください
166ご冗談でしょう?名無しさん:04/04/01 13:55 ID:uBRUp0hD
角運動量保存をLagrangianの空間等方性から出そうとしました。

空間等方性はq -> q + δφ×qの変換について不変と表現されるので、
L(q + δφ×q, q', t) - L(q, q', t) = 0
(∂L/∂q)・(δφ×q) = 0
内積と外積の順序を変えて
δφ・(q×(∂L/∂q)) = 0
Lagrange方程式より
δφ・(q×d/dt(∂L/∂q')) = 0

で、後は d/dt をくくりだせば示せるのですが、そのためには q' × (∂L/∂q') = 0 を
示す必要があります。これはどうやって示したらよいのでしょう?
167ご冗談でしょう?名無しさん:04/04/01 14:43 ID:???
ん? ネーターの定理そのままじゃあかんの?
168ご冗談でしょう?名無しさん:04/04/01 14:52 ID:???
ネーターの定理は詳しくは知らないのですが、とりあえず題意が
「q -> q + δφ×q について不変」 なので、こういうやり方になってます。
169ご冗談でしょう?名無しさん:04/04/01 15:54 ID:Fw3GDwxr
よく歴史に「IF」はない。
というような事を言っている人がいるけど本当に無いんですか?
170ご冗談でしょう?名無しさん:04/04/01 17:07 ID:???
>>169
あるよ。
それは、光が粒子と波の性質を併せ持っており、1つの光子が同時に2つの
スリットをすり抜ける事からも明らか。つまり、一瞬一瞬で同時に複数の歴史
が作られているという事になる。そして、分岐された歴史同士は互いに干渉
する事はないので、結果としてその存在は認識できず、歴史は1つであると
言うように認識してしまうのだ。
171ご冗談でしょう?名無しさん:04/04/01 17:10 ID:2nh/6o6R
>ネーターの定理
その議論を q -> q +ε・S(q)として一般化したのがネーターの定理。
で166のまちがいはq->q+δφ×qの変換のq'への寄与を考慮してないと言うこと。
だから正しくは
δL = (∂L/∂q)δq + (∂L/∂q')δq'
δq = δφ×q
δq' = d/dt(δq) = δφ×q'
などという風になる。
172ご冗談でしょう?名無しさん:04/04/01 17:16 ID:???
>>170
169程度の奴に多世界解釈の概略なんか与えたらトンデモない方に走り出すと思うが。
第一169の質問からは「よく歴史に「IF」はない。」という言葉にどういう含意を感じているかがわからない。
「よく歴史に「IF」はない。」に対する一般的な解釈を採用するならばその解凍はあまりふさわしくないと思われ。
173論理学者:04/04/01 18:19 ID:1S9B1dku
ゴビュウが多いのは許してやれ
174ご冗談でしょう?名無しさん:04/04/01 19:09 ID:???
>>171 納得しました。せっかくなのでネーターの定理も調べてみます。ありがとうございました。
175ご冗談でしょう?名無しさん:04/04/01 19:38 ID:Igc+uypi
すんません。ベッセル関数って有限項の表現ありましたっけ?
176ご冗談でしょう?名無しさん:04/04/01 20:27 ID:???
>>175 無いと思われ 適当な範囲で漸近展開汁.
177ご冗談でしょう?名無しさん:04/04/01 21:22 ID:+SNkq+8x
ちょっとした疑問なんですが。

上空から物を落とすと落ちる程に速さが増しますが、空気抵抗でそのうち一定に
なりますよね?
では、その限界の速さを超えるスピードで上空から下にぶん投げた場合、物体は
減速して一定の速さになるのでしょうか?
178ご冗談でしょう?名無しさん:04/04/01 21:24 ID:???
そうです。
179ご冗談でしょう?名無しさん:04/04/01 21:32 ID:???

冷やかし・冗談 抜きでここは本当に良い板ですね。
180177:04/04/01 21:36 ID:???
>>178
ありがとうございます。これでゆっくり寝られます。
181ご冗談でしょう?名無しさん:04/04/02 06:15 ID:rFgxPiGY
抵抗はなんで変化するのか?金属結合や共有結合等によって電子が陽子から離れる(電子を陽子から引き離すための)ために必要なエネルギーが違うからか。
仮にそうだとすると、なぜ必要なエネルギーが異なるのか?また、電流が流れるときに発生する熱はなぜ生じるのか?
ご回答があればください。あるいはこんなことが分かるホームページはないかい?(マルチご容赦ください。終りそうなスレだったので)
182ご冗談でしょう?名無しさん:04/04/02 06:22 ID:???
抵抗が何で変わるかっていうのは、「変える条件」によって様々。
簡単に言えば、空いてる道と混んでる道とじゃ、流れやすさが違うでしょ?
熱が生じるのは電子などが物理的に運動して、その一部が熱エネルギーに変わるから
(超電導状態では伝導損失が無いので、熱も発生しない)
183ご冗談でしょう?名無しさん:04/04/02 06:39 ID:rFgxPiGY
>>182
空いてる道というのは、陽子間が広いか狭いかということ?「電子などと」は陽子も中性子も含むの?
また、運動した後の停止時に熱エネルギーが発生するの(地面から持ち上げたボールを落とすと、熱エネルギーが地面とボールに発生するように)?
そして、電子を動かすため(電流を流すため)に電圧を加えるという行為は言い換えれば、クーロン力を与えるということ?
(つまりどこかの、いくつかに対してクーロン力で電子を引っ張るか押し出すかを行うことで、電子と陽子のバランスを崩すことで電流は流れる?)
184ご冗談でしょう?名無しさん:04/04/02 13:37 ID:MHsVtlBJ
波動関数Ψと状態ベクトル|Ψ>の違いってなんですか?(用途に関してのメリットとかも
知りたいです)後、<p|Ψ>と言ったらどういう事を表しているんですか?
185ご冗談でしょう?名無しさん:04/04/02 14:08 ID:???
>>183
自由電子の意味わかってるか?
186KingMathematician ◆5lHaaEvFNc :04/04/02 15:13 ID:1Q+4hbD2
Re:>>184 通常違いは無いものと思われるが、|>を付けた方が、線型空間の元というニュアンスが強くなると思う。
<p|Ψ>と云ったら、pとΨの「内積」のことを云う。その「内積」は、たぶん積分で定義されているだろう。
187ご冗談でしょう?名無しさん:04/04/02 15:23 ID:???
>>184

|ψ> = Σψ(a) |a> (和はaについて)

<a|ψ> = ψ(a)

状態ベクトル|ψ>を基底|a>で表示したのが波動関数ψ(a)
188ご冗談でしょう?名無しさん:04/04/02 16:29 ID:MHsVtlBJ
>>187
あ、なんかよく分かりました。すいません、あともう一つで
状態ベクトルとか使うメリットって何かあるんですか?(ブラケットってよく
本に載ってますけど)
189ご冗談でしょう?名無しさん:04/04/02 16:37 ID:rCUxE/sP
ゲーデルの不完全性定理は実際どの程度影響を及ぼしているのですか?
デリダ達の脱構築は物理学者から見てどのような印象を及ぼしますか?
190ご冗談でしょう?名無しさん:04/04/02 16:38 ID:???
>>188
http://homepage2.nifty.com/eman/quantum/contents.html
の第二部を読め。っつーか教科書ちゃんと読め。
191188:04/04/02 17:50 ID:MHsVtlBJ
>>190
どうもです。(講義は受けたんですけど理解できなかったもので)
192ご冗談でしょう?名無しさん:04/04/02 19:49 ID:bjzBpLYI
>>185
自由電子は陽子の周りや陽子間を飛び回っている電子。ですよね。それは分かるんですけど。。。
193ご冗談でしょう?名無しさん:04/04/02 20:21 ID:???
いやもちょっと正確には、なんらかの外力に束縛されることなく運動する
電子のことしか自由電子とは言わないですね普通。
実際の金属中では、電子の性質はわりと自由電子に近いということになっていて
それで自由電子近似でバンドを議論したりするのが金属電子論の初歩でもあります
194ご冗談でしょう?名無しさん:04/04/02 20:26 ID:bjzBpLYI
>>193
ということは自由電子を考えた場合は抵抗(外力)を受けないということですか?
そして、そこへ電界というクーロン力が働くために、電流が流れるということですか?
しかし、なんで自由電子は動き回るのに、そのことを電流が流れたとはいわないんでしょうか?
195ご冗談でしょう?名無しさん:04/04/02 20:31 ID:???
密室の中でも空気分子は高速で飛び回ってるのに、風が吹いてるとは言わないのと一緒だろ。
196ご冗談でしょう?名無しさん:04/04/02 20:45 ID:bjzBpLYI
>>195
空気分子が光速で飛び回るとエネルギーを放出するので、いってみれば電流が流れているのと同じ事では?
197ご冗談でしょう?名無しさん:04/04/02 20:45 ID:???
金属中の電子を自由電子で記述したときは、電子の運動を制限するものがないので
普通に結晶格子の対称性に基づいた量子化エネルギーを持つだけですよね。
そこに電場を印加したときは、電子はF=qEだけの力を受けて運動します
量子力学的には電流は運動量と同じように空間の1階微分で計算するので、
電子が電場に従って運動していれば、そこに電流が流れていることになります
このように電場をかけ続ければ、電子はF=qEに従ってどんどん加速し続ける
ことになります(=電流が増大しつづける)が、実際はそうはなりませんよね
それが抵抗と呼ばれているもので、例えば結晶中に不純物があった場合には
電子は不純物に衝突して散乱され、ある一定以上の速度にならないことになります
こういう電子の運動を阻害する要因は不純物以外にもたくさんあり、
またそのうち主要に働く原因が物質ごと(あるいは系のスケールごと、測定系ごと)に
よって違うので、それぞれの物質によって抵抗の値が異なるというのが簡単な理解です
198ご冗談でしょう?名無しさん:04/04/02 20:54 ID:bjzBpLYI
>>197
電界で引っ張られる電子の速度が一定になるということは、重力場で考えれば物が落ちるときに空気抵抗で速度が一定になるのと同じということですね。
不純物の割合以外に抵抗を変化させるのは、陽子間の距離や陽子の電荷が物質によって違うからでしょうか?
また、通常は電子が進む速度は毎秒数cmと聞いたことがありますが、超伝導では抵抗がなく無限に加速するので電子は光速で移動するんでしょうか?
(おまけとして、例えば電池に導線を接続して短絡させると、最初の瞬間は電池と導線が接している個所のほんの少しの電子だけが電界を受けて徐々に導線全体に電界が伝わるのでしょうか?
それとも同時に導線全体に電界が伝わるのでしょうか?)
199ご冗談でしょう?名無しさん:04/04/02 21:07 ID:???
>>192
>>197
自由電子模型ってドゥルーデ模型やゾンマーフェルト模型のことだろ。
自由電子は格子から「自由」なんだから、自由電子が格子の対称性を反映するのはおかしい。
それとも空格子近似やほぼ自由な電子の近似のことを言っているのか?

不純物やランダムポテンシャルによる並進対称性の破れや、フォノン散乱で抵抗は発生するんだろ。
200199:04/04/02 21:11 ID:???
他にもちろん電子間のクーロン反発力やバンド構造も影響するな
201ご冗談でしょう?名無しさん:04/04/02 21:18 ID:???
宇宙空間における代表的な金属の劣化現象を教えてください
202ご冗談でしょう?名無しさん:04/04/02 22:09 ID:???
そうそう、前のは空格子近似のことを意識して書きました。補足助かります

重力で落ちていく物体が空気抵抗で速度一定という1行目の理解で十分だと思います
あと、陽子間の距離って言うと原子核内部の話かな?って思っちゃうので
固体物理の範囲では原子間距離(=結晶格子の格子点間距離)という言葉を
使うのが一般的ですね。同様に陽子の電荷はいつも+e程度であるので、
原子核の電荷が物質によって異なる、というほうが正確です

物質による原子核の違いは、やっぱり抵抗値にも影響を与えています
電子を散乱する大きな要因として、原子核自体の熱振動(フォノン)が
考えられます。系の温度が高いときは原子核自体が本来あるべき結晶格子点
を中心に振動していて、それが電子の運動を乱す大きな要因となっているためです
フォノンの運動は原子核の持つ電荷というよりはむしろ、系の温度と原子核自体の重さで
決められていて、その違いが物質ごとに抵抗の値を変える原因にもなっています

あと結晶中の電子は原子核が作る周期的なポテンシャルに散乱を受けながら
運動しています。原子核の違いで結晶構造が大きく変わってくることを考えると、
電子は物質ごとに異なる周期ポテンシャルを感じて運動するので、電子は
物質ごとに異なる動きづらさ(=有効質量)を持っていることになりますよね

電界の伝わる速さは実際にあまり問題になることがなかったのでよくワカリマセン
また超伝導のことも専門外なのでワカリマセン
やっぱまだまだ勉強たりないなあ漏れ
203ご冗談でしょう?名無しさん:04/04/03 00:54 ID:???
あの〜流れを無視します、そしてアホです
このことを前提として質問です
超能力ってあるじゃないですか、世間的な
針を立てて紙をバランスよく乗せて、透明なケースで囲って
外から超能力で回すってやつ、あれ物理的にはどうなんですか?
どう解釈するんですか?ヤラセってことになるんですか?
204ご冗談でしょう?名無しさん:04/04/03 00:58 ID:???
さあ?

というのが正しい物理的態度
205ご冗談でしょう?名無しさん:04/04/03 01:06 ID:+cSNybHG
シャボン玉・・・
赤く見えるときの膜厚みはどれくらいかな・・・
青く見えるときも・・・・
数式でしりたいです・・・お願いします。
206203:04/04/03 01:10 ID:???
じゃあヤラセかどうかとかはどうでもいいんで
あれが可能なのか不可能なのかお願いします
207ご冗談でしょう?名無しさん:04/04/03 01:16 ID:???
超能力があれば可能なんじゃないのお?(鼻糞ほじりながら)
208ご冗談でしょう?名無しさん:04/04/03 02:28 ID:???
>>207
なんかラブやん調だなw

>>206
いい方法思いつかないけど、固有振動とか
使えないかな?
209ご冗談でしょう?名無しさん:04/04/03 03:48 ID:???
>206
俺には手品にしか見えんがな。
210ご冗談でしょう?名無しさん:04/04/03 05:12 ID:???
>>203
テレビでやってる超能力番組ってのは
手品を超能力と言ってお金を稼ごうとしてるインチキ手品師と
超能力特番と言って馬鹿な視聴者をだまして
視聴率を稼ごうとしてるなまくらプロデューサーとの
小ざかしい息の合った目論見の賜物です。

本当に超能力が使えるんならテレビなんか出ないって。
211ご冗談でしょう?名無しさん:04/04/03 12:11 ID:???
それらを否定することをウリにする物理学者も一枚噛んでるかもな。
212ご冗談でしょう?名無しさん:04/04/03 12:47 ID:7U6tm4O1
>>202
さんくす。
自由電子は物質によって密度が違うんですね。
自由電子以外の電子が移動することによって電流が流れることを絶縁破壊と言うんですよね?
(そして、このためには相当なクーロン力が必要となると。)半導体は比較手的小さいクーロン力で絶縁破壊を起こすので
壊れやすいのですよね(ホールやキャリアが期待した状態でなくなってしまう)?
213ご冗談でしょう?名無しさん:04/04/03 14:00 ID:???
>>211
大槻教授のことを悪くゆうなー
214ご冗談でしょう?名無しさん:04/04/03 14:42 ID:vL+s6R/l
昔、別の板で出てた問題なんだが。
---
(1)電気量Qで一様に帯電した半径aの導体球を真空中に固定する。
導体球の中心から距離r(>a)の位置に、質量m、電気量qの自由粒子を静かに置くとき、
自由粒子がクーロン力を受けて運動する軌道を求めよ。

(2)導体球が固定された回転軸の周りに一定角速度ωで自転しているとする。
ここに(1)と同じ自由粒子を置くとき、クーロン力とローレンツ力によって
自由粒子が運動する軌道を求め、(1)の場合と比較せよ。

(3)次の主張が正しければ証明し、正しくなければ反証を与えよ。
「(2)の条件において軸の周りの回転座標系をとるとき、
静磁場は見かけ上消失する。したがって、(2)におけるローレンツ力は
回転座標系の慣性力に同じである。」
---

(1) は動径方向の直線に決まってるんだが、(2) から先が手も足も出ない。
定性的には、回転導体球の周りに出来てる磁場に乗って
荷電粒子が渦を描いて飛ぶことはわかるんだが、その軌道を出す式の立て方がさっぱりわからん。
215KingMathematician ◆5lHaaEvFNc :04/04/03 15:09 ID:cv4IZ445
Re:>>214 自由粒子がどんな力を受けるかを考え、運動方程式を立てるのだ。
216ご冗談でしょう?名無しさん:04/04/03 18:51 ID:???
どうして誘電率テンソルは対称テンソルなんですか?
対称テンソルであることには系のどのようなミクロな対称性が反映されているのですか?
217ご冗談でしょう?名無しさん:04/04/03 19:01 ID:pqyAgBrc
大気圧には月の重さもプラスされているんですか?
218ご冗談でしょう?名無しさん:04/04/03 19:15 ID:???
>>217
地球の大気は月の存在する場所まで続いてはいないしので、プラスされていない。
もしされていたとしても、観測不可能なくらい小さい。
219ご冗談でしょう?名無しさん:04/04/03 21:14 ID:dbYRh0rf
砂川「理論電磁気学」のp.66に、次のような式があるのですが、どうも意味がわからないです。

P↑(x↑) = ∫[δV]n(x↑-r↑)p↑(r↑)d^3r/nδV

P↑はベクトルP、[δV]は積分領域、最後のnは、上にバーがついてます。
理論電磁気学を読んでこの式理解できた方、この式の読み方を教えて!
220216:04/04/03 22:36 ID:???
あげ
221ご冗談でしょう?名無しさん:04/04/03 23:38 ID:???
>>219
点xに影響を及ぼす全てのp↑を足し合わせた結果がx点でのP↑ということ。
222ご冗談でしょう?名無しさん:04/04/04 01:05 ID:???
地球を破壊するために必要なエネルギーって何ジュールくらいですか?
223ご冗談でしょう?名無しさん:04/04/04 04:49 ID:???
ポンジュースくらい
224ご冗談でしょう?名無しさん:04/04/04 04:56 ID:2cdKooxL
156 名前:132人目の素数さん[] 投稿日:02/03/28 01:25
リーマン予想は応用が無い数学の例にはならないぞ。
量子カオスの分野で、リーマン・ゼータの零点分布が
エネルギー準位の統計と異常に一致するとか言って前から騒いでる。
もしリーマン予想が証明されたら、シュレディンガー方程式を解かなくても
量子カオスの研究ができるようになるらしい。

ラマヌジャンの保型形式も超弦理論と関係するつってるし。
あの手の純粋数学は却ってどんな応用が出るか油断できないんじゃないか?



数学板のレスですけど、シュレディンガー方程式を解かなくても量子カオスの
研究ができるということは、どのような意味があるのでしょうか?
225ご冗談でしょう?名無しさん:04/04/04 05:30 ID:???
電子はスピンによって磁界を作るそうですが、ということはどんな物質であれ磁界を
持っていて、それが単に打ち消しあっているから、磁界を感じないだけなの?
226ご冗談でしょう?名無しさん:04/04/04 05:38 ID:???
みんなばらばらだからね。
227ご冗談でしょう?名無しさん:04/04/04 07:06 ID:???
http://science2.2ch.net/test/read.cgi/wild/1079687059/181 
 
181 名前:名無虫さん[sage] 投稿日:04/04/04(日) 05:30 ID:???  
人間のホネの中で1番かたいのが頭蓋骨。 
次にかたいのがひざの骨。3番目にかたいのがヒジのホネ。 
1番かたい頭蓋骨をこぶしで破壊することは物理的に不可能。 
頼むから格オタはもうちょっと常識を学習してくれよ。   
228219:04/04/04 07:43 ID:73Urn2Uw
>>221

レスthx!221のヒントをみてもう一回考えてみたんだけど、まだよくわかりません。。。

そもそも、r↑ってどうやってとったんでしょうか?
x↑の原点からとったのか、それともx↑を始点にしてるのか、どっち?

もし原点からとったのなら、n(x↑-r↑)が解釈不能。
x↑-r↑があらぬ方向を向いてしまい、どうもうまくいかない。

もしx↑を始点にしているのなら、n(x↑+r↑)になるはず。

わたしが何を勘違ってるのか、教えてください。
229ご冗談でしょう?名無しさん:04/04/04 07:59 ID:???
>>216
では、逆に
対称テンソルで表現される物理量の他の例を挙げてみな

>>219
nは何?
俺の本は旧版なんでページ違ってわからん。
章のどの辺の内容で前後がわからないと・・
230ご冗談でしょう?名無しさん:04/04/04 08:31 ID:TGL99DCY
>>218
ありがとございました。熱力でひっかかってましたので。
よく考えると、空気の分子と速度が違うことに気がつきました。
231228:04/04/04 08:41 ID:73Urn2Uw
第3章で、分極ベクトルを定義しているところです。

第3版では式(1.9)と(1.10)の間にあります。

232216:04/04/04 18:08 ID:???
>>229
対称テンソルは他に、計量テンソル・慣性テンソル・歪みテンソル等があります。
うーん、でもこれらを挙げてみても共通点が見出せません…
導電率等は対称じゃありませんし
233ご冗談でしょう?名無しさん:04/04/04 19:02 ID:???
>>219
x↑がδVの位置を表す代表点。
r↑がδV内の任意の点で始点はx↑と同じでしょう。
いずれにしても、その本はnやr↑についてちょっと説明不足。
234ご冗談でしょう?名無しさん:04/04/04 20:56 ID:???
age
235ご冗談でしょう?名無しさん:04/04/04 21:19 ID:udZ8n1HH
厨房的な質問でスマソなんですけど
「電荷Qが半径aの球面上で一様分布しているときの電場を求めよ」と問題なんですが、
ガウスの定理を使って、E(r)がrの関数として
 (4πr^2)E(r)ε。=Q (Q:電荷)
まではわかったんですが、Qが求まりません。
どうすればよいでしょうか。ご教授願います。
236235:04/04/04 21:21 ID:udZ8n1HH
文章が不適切でした、すいません
「電荷Qが半径aの球面上に一様に分布しているとき、これのつくる電場はどうなるか」でした
237ご冗談でしょう?名無しさん:04/04/04 21:24 ID:???
Qは与えられているんだから求める必要はないんじゃネーノ?
238ご冗談でしょう?名無しさん:04/04/04 21:30 ID:udZ8n1HH
問題の答えを見ると

 E(r)=0 (r<aの時)
 E(r)=Q/(4πε。r^2) (r>a)

って書いてありますが
239ご冗談でしょう?名無しさん:04/04/04 21:32 ID:???
だから、Qはそのまま使ってるでしょ?
240ご冗談でしょう?名無しさん:04/04/04 21:35 ID:udZ8n1HH
何故、r<aの時、E(r)=0になるんでしょうか
241ご冗談でしょう?名無しさん:04/04/04 21:36 ID:???
>>235>>238
この問題はQやaやrを使って電場Eを求める問題であって、Qを求める問題ではない。
242ご冗談でしょう?名無しさん:04/04/04 21:37 ID:???
だって閉曲面内に電荷ないでしょ
243ご冗談でしょう?名無しさん:04/04/04 21:38 ID:???
ガウスの法則を理解できていない模様。

r<aのときは、その閉曲面内の電荷がゼロ、つまりQ=0になるでしょ?
244ご冗談でしょう?名無しさん:04/04/04 21:39 ID:???
>>240
球体の対称性とガウスの法則を使えば分かる。
対称性から、電場は放射状、あるいはその逆、そしてゼロのいずれかしか許されない。
245ご冗談でしょう?名無しさん:04/04/04 21:39 ID:udZ8n1HH
>>242
閉曲面外にも電荷が無いような気がしますが・・・
246KingMathematician ◆5lHaaEvFNc :04/04/04 21:42 ID:weVjxCa1
Re:>>242 それで説明したつもりなのか?
247ご冗談でしょう?名無しさん:04/04/04 21:44 ID:???
ガウスの法則を初めて習っても、それを即座に問題に応用できないのは普通だと思う。
いくつか問題と回答を眺めて、あ、そーやって適用するのか!っていうショックを味わう必要がある。
248ご冗談でしょう?名無しさん:04/04/04 21:47 ID:???
KingMathematicianって数学スレ限定の固定じゃないの?
249ご冗談でしょう?名無しさん:04/04/04 21:55 ID:???
確かに、電荷があるのは閉曲面内部と閉曲面外部の境目だけなのに
内部は0で、外部がQなのは疑問だ
250ご冗談でしょう?名無しさん:04/04/04 21:57 ID:???
>>249
それの何が疑問なのかが疑問だ。
251ご冗談でしょう?名無しさん:04/04/04 21:58 ID:???
>>250
電荷は閉曲面の内部にも外部にも存在しないのに
内部だけ0になるのが疑問
252KingMathematician ◆5lHaaEvFNc :04/04/04 21:59 ID:weVjxCa1
Re:>>248 別に物理板に来てもいいだろう。
Re:>>249 内部ではあらゆる方向に電荷が分布していることに注意して欲しい。
253ご冗談でしょう?名無しさん:04/04/04 21:59 ID:???
>>249
>内部ではあらゆる方向に電荷が分布していることに注意して欲しい。
もう少し噛み砕いて下さい
254KingMathematician ◆5lHaaEvFNc :04/04/04 22:00 ID:weVjxCa1
あらゆる方向に電荷が分布しているからと云って、必ずしも、そこにある電場が0になるわけではないが。
255ご冗談でしょう?名無しさん:04/04/04 22:00 ID:q565Kcht
auger過程・Auger Electron Spectroscopyなどの
Auger氏のフルネームがわかる人いますか?
よろしくお願いします。
256ご冗談でしょう?名無しさん:04/04/04 22:00 ID:???
>>252
今度は是非、化学板にも遊びに来てください
257KingMathematician ◆5lHaaEvFNc :04/04/04 22:01 ID:weVjxCa1
Re:>>253 積分計算で何とか頑張るしかないだろうか?
258ご冗談でしょう?名無しさん:04/04/04 22:01 ID:???
な?
初心者は閉曲面と言えば、即r=aの閉曲面しか思い浮かばなかったりするんだよ。
お前らだって初めは>>251みたいにアホな考え方をしてたんだよ。もうとっくに忘れてしまっただろうが。
259ご冗談でしょう?名無しさん:04/04/04 22:02 ID:???
大学教育に、素人の思考様式を考慮したカリキュラム編成を導入するのは決してムダにはならないハズ。
260KingMathematician ◆5lHaaEvFNc :04/04/04 22:03 ID:weVjxCa1
Re:>>256 化学はなかなか難しい。高校で化学はストップした上、高校で習った化学はかなり忘れてしまったよ。中学までの化学なら大丈夫だが。
261ご冗談でしょう?名無しさん:04/04/04 22:03 ID:???
0<r<aの時 電荷は0
r=aの時 Q
r>aの時 電荷は0

じゃないのでしょうか?
262KingMathematician ◆5lHaaEvFNc :04/04/04 22:05 ID:weVjxCa1
Re:>>261 またわけの分からぬことを。気持ちは分からぬでもないが…。
r=aのときも電荷は0だろう。
電荷密度(?)は0にはならないが。
263ご冗談でしょう?名無しさん:04/04/04 22:06 ID:???
右辺をQと書いちゃうのが問題だな。

右辺は∫ρdv。
264ご冗談でしょう?名無しさん:04/04/04 22:07 ID:???
>>262
r=aでは電荷Qが一様に分布しているのに何故、電荷が0になってしまうのでしょうか
265KingMathematician ◆5lHaaEvFNc :04/04/04 22:07 ID:weVjxCa1
ちょっとした疑問:
電場と電位はどう違うのですか?
266KingMathematician ◆5lHaaEvFNc :04/04/04 22:08 ID:weVjxCa1
Re:>>264 ある一点を採って考えれば0になる。
267ご冗談でしょう?名無しさん:04/04/04 22:08 ID:???
>>265
カエレ。
268ご冗談でしょう?名無しさん:04/04/04 22:09 ID:???
>>265
E=-gradient(V)
269ご冗談でしょう?名無しさん:04/04/04 22:10 ID:???
>>261
r=aの面上は特異的だからr=aを議論するのは無意味。
現実問題、電荷が2次元的に球面上に分布することは無いから。

>>251
球の内部と球の外部(つまり球の補空間)は同等ではない
(数学的には「同相ではない」)のだからちっともおかしくない。
何と何が対等あるいは対称かを見極めることが必要。
270ご冗談でしょう?名無しさん:04/04/04 22:13 ID:???
>>269
内部、外部にも電荷が存在すると言うことでしょうか?
271KingMathematician ◆5lHaaEvFNc :04/04/04 22:14 ID:weVjxCa1
Re:>>270 問題文からでは、それは分からない。
272ご冗談でしょう?名無しさん:04/04/04 22:14 ID:???
KingMathematicianは何歳ですか?趣味は?
273ご冗談でしょう?名無しさん:04/04/04 22:14 ID:???
>>269を補足すると、r=aを議論することは「tan(π/2)」や
「ステップ関数σ(x)のσ(0)」を議論することと同様に無意味。
274ご冗談でしょう?名無しさん:04/04/04 22:16 ID:???
>>271 お前は黙ってろボケ!
問題文からは、内部、外部ともに電荷分布はなく、r=aの球面上に二次元的に分布しておると書いてある。
その仮定の上でなお、特異点(特異面といったほうがいいか)の電荷を議論するのは
このような初歩的な問題では全く意味をなさない、ということをいっておるのだ。
275ご冗談でしょう?名無しさん:04/04/04 22:17 ID:???
>>270
現実問題としては、電子は空間的な分布を持ち(古典的には電子は大きさを持ち、
量子論的には確率の広がりをもつことに相当)、球面の内と外に対して広がって分布するので、
球面の外内で電場はゼロにゆっくりと近づく。しかし十分に内側では電場はやはりゼロ。
276274:04/04/04 22:18 ID:???
×特異面の電荷を議論
○特異面の電場を議論
277KingMathematician ◆5lHaaEvFNc :04/04/04 22:19 ID:weVjxCa1
電場の物理単位って何ですか?
278ご冗談でしょう?名無しさん:04/04/04 22:19 ID:???
KingMathematicianに質問なんだけど
閉曲面Sがあって、その外部に点電荷Qがあるとするとき、
∫E(r)・nds=0
(E(r):電場ベクトル、nは曲面Sの微小面積dsに対する単位法線ベクトル)
であることを数学的証明の仕方を教えて下さい
279275:04/04/04 22:19 ID:???
だから>>270の言うとおり、現実問題では内部・外部にも電荷は存在する。
この問題はあくまで理想化された人工的なモデルであることに注意したほうがよい。
280ご冗談でしょう?名無しさん:04/04/04 22:23 ID:???
>>277-288
微分形式を使って説明おながいします。
281ご冗談でしょう?名無しさん:04/04/04 22:26 ID:Qc+/f4OJ
>>277-278
閉曲面内部をコンパクト多様体としてあつかって、
電場の2-形式を電荷の体積形式にストークスの定理を使って変換すればいいの?
282ご冗談でしょう?名無しさん:04/04/04 22:29 ID:???
>>281
ストークスじゃなくてガウスの発散定理じゃね?
283ご冗談でしょう?名無しさん:04/04/04 22:34 ID:Qc+/f4OJ
>>282
いや、多様体上の積分定理のストークスの定理。面積文を線積分に置き換えるやつとちがう。
∫_∂V ω = ∫_V dω ってやつ。
KingMathematicianの解説に期待。
284KingMathematician ◆5lHaaEvFNc :04/04/04 22:34 ID:weVjxCa1
Re:>>278 ガウスの発散定理って知ってる?
さらにE(r)には電場ポテンシャルUがあって、rot(U)=Eとなるはずだから、
∫Ends=∫divEdxdydz=∫divrotEdxdydz=0 (divrot=0だから)
となる。
p.s. 電場の定義を書いてください。
285ご冗談でしょう?名無しさん:04/04/04 22:35 ID:???
>>284
電界と同じ
286ご冗談でしょう?名無しさん:04/04/04 22:36 ID:???
>さらにE(r)には電場ポテンシャルUがあって、rot(U)=Eとなるはずだから、

だからお前のようなアホは帰れと言っておるだろーが。これ以上恥をかくこともあるまい。
287ご冗談でしょう?名無しさん:04/04/04 22:36 ID:???
>>284
>さらにE(r)には電場ポテンシャルUがあって、rot(U)=Eとなるはずだから、
難しい数学わかんね
288ご冗談でしょう?名無しさん:04/04/04 22:37 ID:???
>さらにE(r)には電場ポテンシャルUがあって、rot(U)=Eとなるはずだから、

電場ポテンシャルUがわからない
何故、rot(U)=Eなるものがあるのかがわからない
289KingMathematician ◆5lHaaEvFNc :04/04/04 22:37 ID:weVjxCa1
誰か誘電率の定義を説明してください。あるいは、物理単位を書いてください。
290ご冗談でしょう?名無しさん:04/04/04 22:38 ID:???
>>283
閉曲面は閉じているからストークスは適応できない予感
291ご冗談でしょう?名無しさん:04/04/04 22:39 ID:Qc+/f4OJ
>>289
>>280-283を教えてくださったら俺がお答えします。
292ご冗談でしょう?名無しさん:04/04/04 22:40 ID:???
>>289
単位は[クーロン]^2*[ニュートン]^(-1)*[メートル]^(-2)
293ご冗談でしょう?名無しさん:04/04/04 22:40 ID:???
>>286
恥をかくことに違和感がなければ、帰る必要もない。
まるで、生きてるのか、死んでるのか、寝ているのか、起きているのか、
わからない世界にとっては、恥などどうでもいい。
294ご冗談でしょう?名無しさん:04/04/04 22:41 ID:Qc+/f4OJ
>>290
閉曲面に囲まれた領域はコンパクトだから、その閉領域に対してストークスの定理が適用できると思う。
でも微分形式の取り方が分からん。
295KingMathematician ◆5lHaaEvFNc :04/04/04 22:43 ID:weVjxCa1
Re:>>291 吾はストークスの定理を二つ知っている。
296ご冗談でしょう?名無しさん:04/04/04 22:44 ID:???
>>295
焦らすのは女のたしなみ。行動力があるのは男の甲斐性。
297ご冗談でしょう?名無しさん:04/04/04 22:44 ID:???
KingMathematician、KingMathematician、>>287-288に答えて
298KingMathematician ◆5lHaaEvFNc :04/04/04 22:51 ID:weVjxCa1
Re:>>297 悪いけど、電場という概念自体がまだよく分からないからパス。
299ご冗談でしょう?名無しさん:04/04/04 22:53 ID:???
KingMathematician、KingMathematician、>>272に答えて
300ご冗談でしょう?名無しさん:04/04/04 23:01 ID:???
KingMathematician、KingMathematician、何か面白いこと言って
301ご冗談でしょう?名無しさん:04/04/04 23:33 ID:???
原子核と電子の間にある空間はいったいどのようなものなのですか?
何も無いと思うのですが、何も無いとはどういうことなのでしょうか。
電車の中で一時間考えても分かりませんでした、どなたか教えてください。
302ご冗談でしょう?名無しさん:04/04/04 23:47 ID:???
誰か>>216 >>232にも答えてください。
303ご冗談でしょう?名無しさん:04/04/05 01:34 ID:NkmX5AkB
中性子は地表において、単独で長期間(例えば年単位)存在しつづけることは
あるでしょうか? 何かの原子核に吸収される感じで落ち着くことになるでしょうか?

中性子線の中性子の運命に興味があります。申し訳ありませんが、よろしくお願い
いたします。
304ご冗談でしょう?名無しさん:04/04/05 01:54 ID:???
>>303
単独ではベータ崩壊して陽子になる。
半減期20分ぐらいだったかな。
単独で長期間存在しつづけることはないはず。
305ご冗談でしょう?名無しさん:04/04/05 02:00 ID:???
>>304
ありがとうございました。
半減期約20分ですか、とても短いのですね。
陽子になった後は近場の適当な物質と化合すると考えてよさそうですね。
大変参考になりました。ありがとうございました。
306ご冗談でしょう?名無しさん:04/04/05 02:49 ID:???
目にはみえない、かみなりさまのビリビリを理解するのには
ファインマンさんの教科書がよい。とおもう。
307ご冗談でしょう?名無しさん:04/04/05 04:34 ID:???
>>305
 中性子線より陽子線(ていうのか?)の方が電離作用もあるだろうから危険のような。
308ご冗談でしょう?名無しさん:04/04/05 13:04 ID:???
伸ばしてシュレーディンガーっていうのか
伸ばさずシュレディンガーっていうのかどっちが一般的?
漏れのIMEではシュレーディンガーは一発変換できるけど
シュレディンガーはシュレとディンガーで分かれる。
309ご冗談でしょう?名無しさん:04/04/05 14:11 ID:???
>>308
どっちも同じぐらい一般的。
ただし言葉のあとに『音頭』 が付くとシュレディンガーの方が一般的。
310ご冗談でしょう?名無しさん:04/04/05 15:05 ID:90LhYHQL
無理だろうけど、鏡の球体の中に光を入れたら
永久に明るいんでつか?
311ご冗談でしょう?名無しさん:04/04/05 16:14 ID:???
>>310
「明るい」かどうかを調べるには何らかの検出器をその球体の中に入れなきゃならんだろ。
検出器で「明るい」ことが検出されたったことは、球内の光子の一部が検出器に吸収されたってことだ。
てことはずーっと明るいってことはずーっと検出器に光子が吸収され続けてるってことで、
だんだん球内の光子が減ってくわけだから、永久に明るいって事はありえない。

検出器を入れなければ(そしてその球体の鏡が反射率100%なら)球内の光子はいつまでも
球内で反射を続けるだろうが、その状態を「明るい」と言うことができるかどうか。
これは「明るい」という言葉の定義の問題になってくるわけだが。
312ご冗談でしょう?名無しさん:04/04/05 16:53 ID:???
>>310
普通に考えると、反射率が100%じゃないからいずれ真っ暗。
313ご冗談でしょう?名無しさん:04/04/05 17:42 ID:???
>>312
じゃあ、これならどう?
光ファイバーをループ状に繋げ、その中に光を入れるってのは。
314ご冗談でしょう?名無しさん:04/04/05 18:07 ID:???
>>313
ファイバーの透過率が(以下同文
315ご冗談でしょう?名無しさん:04/04/05 19:48 ID:???
>>313
光を閉じ込めたいならこれ↓
ttp://www.ceramic.or.jp/ig-nenka/nenkai/1.html
一千万分の一秒も閉じ込められるんだぞ
316ご冗談でしょう?名無しさん:04/04/05 21:47 ID:???
kingとか名乗ったり自意識過剰な言葉使いとかしてる時点で厨っぽさ爆発してるのに
アホとかボケとか言われながらも必死に食いついてくる緑文字がいじらしい
317KingMathematician ◆5lHaaEvFNc :04/04/05 22:01 ID:gxJYLaFz
緑?
318ご冗談でしょう?名無しさん:04/04/05 22:03 ID:???
さっそく食らいついてきますた
319KingMathematician ◆5lHaaEvFNc :04/04/05 22:15 ID:gxJYLaFz
吾はOJでスレ見てるから、名前は全部緑色になるし、自由に色を変えられるよ。
320ご冗談でしょう?名無しさん:04/04/05 23:18 ID:TVZ6BHB9
>>319
おまいにどう見えるかは何の関係もないだろw
321ご冗談でしょう?名無しさん:04/04/06 00:24 ID:QD9OkSdX
322ご冗談でしょう?名無しさん:04/04/06 00:31 ID:???
>>321
ブラクラ
323ご冗談でしょう?名無しさん:04/04/06 00:35 ID:MsdeweWC
↓こんな積分が出て来たのですが
∫[0→π](c/(c-a・cosΘ))dΘ (c,aは定数)
つまってしまいました。
secの積分だったらわかるんですが
やりかたわかりますか?

数学板できこうかと思ったけど砂川の電磁気演習(P121問22)やってたら出てきたので
こっちのほうがいいかなと。
324ご冗談でしょう?名無しさん:04/04/06 01:05 ID:???
>>323
有理式の変数を三角関数で置き換えた関数を積分する方法は
1年の解析でやったはず。教科書の不定積分のあたりを見ろ。
325ご冗談でしょう?名無しさん:04/04/06 01:43 ID:MsdeweWC
>>324
t=tan(Θ/2)っておくやつですか。
326323:04/04/06 01:50 ID:MsdeweWC
あ、できた。ありがとう。
327ご冗談でしょう?名無しさん:04/04/06 06:12 ID:K8hPGUd1
エンタルピーとエントロピーとはどのような物理量を表すのでしょうか。
物理学や物理化学の本を手当たりしだい調べたのですがどれも抽象的で
イマイチイメージが沸きません。
どなたかご教授お願いします。
328ご冗談でしょう?名無しさん:04/04/06 07:08 ID:jlHAKR5j
エンタルピーは、物質が蒸発するのに必要なエネルギー。
エントロピーは、状態の加逆性を表す指標。

気体の状態でのエンタルピーと液体の状態でのそれの差が気化熱となる。

状態Aと状態Bがあったとき、AのエントロピーがBのエントロピーより小さければ、A→Bとできる。

また、状態Aの系と状態Bの系があったとき、それらを相互作用させて状態Cと状態Dにすることができるのは、
S(A)+S(B) ≦ S(C)+S(D)
のとき。

熱力学の本には、これぐらいのことはきっと書いてあるはず。
化学系ならフェルミか佐々真一の本を、物理系の人なら田崎とかを読むべし。
329ご冗談でしょう?名無しさん:04/04/06 07:44 ID:???
エンタルピー・・・基準となるある温度下における系に含まれる熱量
エントロピー・・・系の”乱雑さ”を表す指標


この説明だと、”エントロピーとエネルギーの違いは?”、”乱雑さって?”と聞かれたら困るが・・・
330ご冗談でしょう?名無しさん:04/04/06 14:22 ID:???
エンタルピーってどこにアクセントあるの?
331ご冗談でしょう?名無しさん:04/04/06 17:23 ID:???
そんな事よりよ、ちょいと聞いてくれよ。スレとあんま関係ないけどさ。
このあいだ、ザ掲示板の物理学行ったんです。物理学。
そしたらなんか『錬金術について』ってのがあるんです。
で、よく見たらなんかスレがあって、
『「賢者の石」とか作ってみたいにですが・・・。』
とか書いてあるんです。
もうね、アホかと。馬鹿かと。
お前らな、『鋼の錬金術師』如きで普段来てない物理学に来てんじゃねーよ、ボケが。
賢者の石だよ、賢者の石。
なんか消防とかもいるし。小学生で錬金術師か。おめでてーな。
よーしぼく錬金しちゃうぞー、とか言ってるの。もう見てらんない。
お前らな、漫画やるから物理学から消えろと。
錬金ってのはな、もっとオカルトっぽくなってるべきなんだよ。
宗教の違いでいつ殺戮が始まってもおかしくない、
死すか殺すか、そんな雰囲気がいいんじゃねーか。女子供は、すっこんでろ。
で、やっと落ちついたかと思ったら、他の奴が、
『実際錬金術をするのに錬成陣はいるのですか? 』
とか言ってるんです。
そこでまたぶち切れですよ。
あのな、錬成陣なんて意味ねーんだよ。ボケが。
得意げな顔して何が、錬成陣、だ。
お前は本当に錬金術興味があるのかと問いたい。問い詰めたい。
小1時間問い 詰めたい。
お前、賢者の石って言いたいだけちゃうんかと。

ttp://physics.dot.thebbs.jp/r.exe/1062850418.html
332ご冗談でしょう?名無しさん:04/04/06 18:08 ID:???
     ∧_∧  ∧_∧
ピュ.ー (〆∀〆) ( ´ー` ) <これからも僕たちを応援して下さいね(^^)。
  =〔~∪ ̄ ̄ ̄∪ ̄ ̄〕
  = ◎――――――◎                      ボーア&ハイゼンベルク

これを改良してください。
333ご冗談でしょう?名無しさん:04/04/06 18:12 ID:MsdeweWC
334ご冗談でしょう?名無しさん:04/04/06 20:54 ID:???
18 :あるケミストさん :04/04/06 11:18
私は近眼で両目とも0.3なので、
普段メガネをかけないとテレビが見づらい。
で、あるときふと思いついた。

テレビを鏡に映し、その鏡を至近距離で見れば
メガネなしでもOKじゃないか! と。

しかし、実際に試してみると、
鏡に映るテレビがぼやけて見える。
メガネを掛けて鏡をみるとはっきり見える。
いまだにこのアイデアがどこで間違ったのか分からない。
だれか教えてくり。

化学板では板違いで終わらせられてたんで
こっちで教えてください。俺も知りたい。
335ご冗談でしょう?名無しさん:04/04/06 21:24 ID:???
モニターと鏡の違いを考えろ
モニターと鏡、見る角度を変えた時どうなるか
そこから、眼球内で像を結ぶ光がどのように
やってきてるかに思いをはせつつ星に願いを
336ご冗談でしょう?名無しさん:04/04/06 21:26 ID:???
>>334
発光と反射の違い。

間近で発光してるものならちょっとのずれも気にならないが、
反射してるものだと遠くのちょっとのずれも反射するからずれたのを
目が拾ってしまうから。
337ご冗談でしょう?名無しさん:04/04/06 21:27 ID:???
>>336
バカ発見
338336:04/04/06 21:29 ID:???
あ、書いてる内にレスがあった。
星で思ったけど、>>334の理屈で言うと
鏡に映った太陽に近付くと焼け死んじゃうかもな。
339ご冗談でしょう?名無しさん:04/04/06 21:44 ID:???
>>334
つまりそれだと鏡はハキーリ見えるが
それに写った光はハキーリ見えない







正解?
340ご冗談でしょう?名無しさん:04/04/06 22:09 ID:???
>>339
341ご冗談でしょう?名無しさん:04/04/06 22:31 ID:???
凸球面状に歪曲した鏡なら理論上いける。
だがそのためには、もとの映像を凸球面に収束させる必要がある
そのためには・・・・

なんて考えていくと、結局膨らんだガラスを通すのが一番効率的
→めがねをかけれって話になるんだよな。
342ご冗談でしょう?名無しさん:04/04/07 09:01 ID:???
近視だから、膨らんだガラスじゃなくて凹んだガラスでは?
343ご冗談でしょう?名無しさん:04/04/07 11:57 ID:???
錬金術ってなんですか?
344ご冗談でしょう?名無しさん:04/04/07 11:59 ID:???
>>343
化学板で訊いたほうがいいよ
345ご冗談でしょう?名無しさん:04/04/07 13:50 ID:???
d^2r/dt^2ってなんて読むんですか?
346ご冗談でしょう?名無しさん:04/04/07 13:55 ID:???
書いてから高校生用のスレがあることに気がついた。。。
しかしマルチはいやなのでこっちでいいです
347ご冗談でしょう?名無しさん:04/04/07 14:52 ID:???
>>345-346
読み方だよね。
d squared r over dt squared.

または、
The second differential of r with respect to t.
348ご冗談でしょう?名無しさん:04/04/07 15:11 ID:???
>>347
日本語での読み方聞いてんじゃないの?
おれはそのまま
「でぃーじじょうあーるでぃーてぃーじじょう」
と読むけど。
349ご冗談でしょう?名無しさん:04/04/07 15:47 ID:???
漏れもそうかな。発声する事自体があまりないけど

ところで高校数学って、2階微分もやるんだっけ?
350ご冗談でしょう?名無しさん:04/04/07 16:19 ID:???
おれは「でぃーてぃーじじょうぶんのでぃーじじょうあーる」と読む。
351ご冗談でしょう?名無しさん:04/04/07 16:31 ID:???
overって便利だよな。「分の」だと視点の流れが途切れるし、混乱しやすい
352ご冗談でしょう?名無しさん:04/04/07 18:37 ID:???
SFものの小説を書きたいと思ってます。
恒星間移民なんてものを考えてます。
街を一つ宇宙船にしたとして、
光速までの加減速が可能だとします。
加減速度を重力加速度と同じにして、
加速時と減速時で向きを反対にすれば
中の人は普通の生活が送れるかと。
また、光速での巡航時は中の人の時間は経過しないかと。
ここで、
中の人が9.8m/s2の加速を常に感じているためには、
外部(地球)から見てどのような加速をすれば良いのかという疑問が湧きました。
光速付近になると、中の人の時間経過は遅くなるのだから、
えーっとえーっと・・・・
すいません助けてください。



353ご冗談でしょう?名無しさん:04/04/07 19:46 ID:???
>>352
この物語はフィクションです。
実際の科学考証・物理法則は
一切関係ありません。
354ご冗談でしょう?名無しさん:04/04/07 21:04 ID:???
>>351
「わる」って読めば対等だっ!
355ご冗談でしょう?名無しさん:04/04/07 21:40 ID:???
>>352
わかりづらい。物理法則どうこうより、まず文章力をつけたほうがいい
356ご冗談でしょう?名無しさん:04/04/08 12:39 ID:???
>>352
俺も>>355と同意見だが、恐らく
http://wow.bbspink.com/leaf/kako/1031/10315/1031587915.html
の828番のようなことをしたいのではないかと。
357ご冗談でしょう?名無しさん:04/04/08 13:33 ID:???
>>352
事実に対して不都合な事は無視する、それが物語というもの。

隣の恒星アルファケンタウリに移住可能な惑星があるとは限らないので、移住可能な恒星系まで何十年何百年かかるかわからないし、
それと、物質をどうやって光速度まで加速させるか、という問題を無視して、なぜ「加速」という事に拘るかわからない。
358ご冗談でしょう?名無しさん:04/04/08 13:51 ID:???
>>357
物語のためには科学考証(らしきもの)を全否定せよって見方は
ちょっと一面的にすぎるな。
科学的裏付けがあるかのように見せるってのも物語の演出のうちさ。
銀英伝で氷塊ぶつけるのに相対性理論を持ち出したみたいに。

>>352に忠告。
最初に光速でないものを、光速に加速することは決して出来ない。
 光速まで加速→光速近くまで加速
 中の人の時間は経過しない→中の人の時間経過はいくらでもゼロに近くなる
と言うべし。
359ご冗談でしょう?名無しさん:04/04/08 14:01 ID:???
>>345
おれは "the second derivative of r with respect to t" 派なんで
「rのtについての2階導関数」か「rのt2階微分」って読むな。
ディーアールわるディーティーとか読むのは
dr/dt = (dr/dθ)(dθ/dt)
みたいな演算をやってるときか、よっぽど慌ててるときか。
>>349
やるよ。3次・4次関数の変曲点の考察とか。
360ご冗談でしょう?名無しさん:04/04/08 18:26 ID:pbx/GjjX
くだらないこと聞いてすみません。
ゼータ関数と量子論ってどういう関係があるんですか?
361ご冗談でしょう?名無しさん:04/04/08 18:55 ID:???
数式をいちいち読み下す必要がある場面なんて漏れはほとんどないんだけど。
362ご冗談でしょう?名無しさん:04/04/08 18:58 ID:???
>>352 は何をやってもダメな子
363ご冗談でしょう?名無しさん:04/04/08 19:05 ID:???
速度に比例する空気抵抗を受けながら、重力によってまっすぐに落下する
物体の運動を自由落下という、自由落下を表現する微分方程式は

mdv/dt=mg−kv

と書ける。vは物体の落下速度、gは重力加速度、mは落下する物体の質量
kは抵抗係数である、それをmで割ると

v'+kv/m=g

となる。ここでa=k/mとおくと

v'+av=g

となる。
で合っていますでしょうか?
364ご冗談でしょう?名無しさん:04/04/08 19:13 ID:???
>>363
ハァ?
両辺定数で割って、文字の置き換えしただけじゃねーか
どこに間違える余地があるんだ?
んなこといちいち聞きにくんな
365ご冗談でしょう?名無しさん:04/04/08 19:54 ID:???
>>363
合っています。たまに(とくに春は)>>364のような虫が湧いてきますが、
なにか質問があれば遠慮なくまたお越しください。
366ご冗談でしょう?名無しさん:04/04/08 20:47 ID:???
||v||
367ご冗談でしょう?名無しさん:04/04/08 21:49 ID:???
>>365
 >>364の言うことはもっともだと思うがな。

 それと>>363よ、自由落下は空気抵抗はないと考えたものだろ。
368ご冗談でしょう?名無しさん:04/04/08 23:02 ID:tD3/CNwx
コーラをエアダスターで冷やすと…というおバカな話。
今までにパソコンに笑ってふいちゃった食べ物飲み物
http://pc3.2ch.net/test/read.cgi/pcnews/1079997049/54-55
同補足
http://pc3.2ch.net/test/read.cgi/pcnews/1079997049/74


原理を詳しく解説してください。
369ご冗談でしょう?名無しさん:04/04/08 23:25 ID:???
炭酸の飲み物を振ってから開けると爆発するだろうが
370ご冗談でしょう?名無しさん:04/04/08 23:38 ID:5pJE5/fD
>>369
ちょっと違う気がする。
371ご冗談でしょう?名無しさん:04/04/09 00:07 ID:???
大量のガスを吹き込んだんだろ?
過飽和状態の二酸化炭素が吹き込まれたガスを種にして
一気に気化した。振動で種ができるのと同じことだな
372ご冗談でしょう?名無しさん:04/04/09 00:43 ID:???
過飽和状態の二酸化炭素が、吹き込まれたガスを種にして

紛らわしいので読点打っとく
373ご冗談でしょう?名無しさん:04/04/09 01:45 ID:wTxNEHq9
基本粒子よりもさらに小さな構成要素ってありますか?
もしくは、可能性ってあるのでしょうか?
374ご冗談でしょう?名無しさん:04/04/09 03:44 ID:???
>>373
あればそれを基本粒子と呼びなおすだろ
375ご冗談でしょう?名無しさん:04/04/09 06:31 ID:???
ニュートンの運動の法則のことなんですが

f=Ma
って重力場では使えないですよね。
三法則が成立するのは慣性系だけど重力場は慣性系ではないから
376ご冗談でしょう?名無しさん:04/04/09 06:46 ID:???
>>375
ニュートン力学で言う慣性系と一般相対論で言う(局所)慣性系は意味が違う。
「三法則が成立するのは慣性系だけど」の慣性系は前者だし
「重力場は慣性系ではないから」の慣性系は後者。


377ご冗談でしょう?名無しさん:04/04/09 10:09 ID:???
>>376
レス、ありがとうございます。
えっ?そうなんですか??よくわからないです。

例えば「相対性理論」内山龍雄 岩波
1pなんかみると

Newton力学は3つの法則をもとに出来上がってる・・中略
ある座標系からながめたとき、慣性の法則が成立してる場合、慣性系とよぶ
厳密なことをいえば外力が全く作用していないような物体を用意することは
不可能である。したがって慣性系がこの世に存在するか否かは明らかでない・・

とあります。
僕の疑問は、するとニュートン力学は重力場では使えないことになりませんか?
378ご冗談でしょう?名無しさん:04/04/09 10:51 ID:???
>>367
確かに、空気抵抗があったら「自由」じゃないなw

>>373
一時期、プレオンとか仮定されて研究されてたが、
クォークやレプトンよりも小さな構成要素は無さそう、
という結論だった。

>>377
ニュートンの思想では、まず先に「絶対空間」という
大局慣性系ありき、その中に重力場がある、ってこと
だと思う。だから、F=maはまず成り立ってて、
Fが重力やその他の力であることになる。
379ご冗談でしょう?名無しさん:04/04/09 11:18 ID:???
>>361
あんたになくても、俺にはあるぜ
380ご冗談でしょう?名無しさん:04/04/09 11:35 ID:???
>>377
慣性の法則は、いくつかの力がはたらいていても、それらの力がつり合っていれば成り立つ。
ニュートン力学では重力は力だから、たとえば摩擦のない水平面上の運動では重力と垂直抗力がつり合うので慣性の法則が成り立つ。
つまり、重力場でもニュートン力学は使える。
381ご冗談でしょう?名無しさん:04/04/09 12:03 ID:???
>>379
ふ〜ん。大変だねぇ
382ご冗談でしょう?名無しさん:04/04/09 12:11 ID:???
>>381
教室で物理を教えた経験がおありで?
383ご冗談でしょう?名無しさん:04/04/09 12:29 ID:???
>>382
ありますが、白板に式を書いて、口では「こうなります」と言えば事足りてました。
いちいち式を読み下してたらかえってわかりづらく、時間も全然足りなくなっていたに
違いないです。
384ご冗談でしょう?名無しさん:04/04/09 13:27 ID:???
>>383
そりゃそうだ。すべての式を朗読できるわけじゃないし、その必要もないさ。
しかし、だからって「こうなります」ですべてが事足りたわけでもない。
「ディーティーわるディーシータとみて置換すると…」なんて台詞も必要だろ。たまには。
385ご冗談でしょう?名無しさん:04/04/09 13:57 ID:???
>>384
「こう置換すると」で通しちゃってたなぁ。そもそも板書しながら
しゃべらないもん。白板に向かって喋ることになるから聞き取り
にくいだけだろうから。説明に必要な分だけ書き切ってから
おもむろに説明を始める

386ご冗談でしょう?名無しさん:04/04/09 21:20 ID:???
>>368
追試実験とともに他の様々な刺激を炭酸飲料に加えて実験しろ
吹き込むガスの量を変えたりガスでなくて違うものを入れたり
別の方法で何らかのガスを吹き込んだり
387ご冗談でしょう?名無しさん:04/04/09 23:10 ID:???
動滑車と定滑車ってどう違うのでしょうか?教えてください
388ご冗談でしょう?名無しさん:04/04/09 23:20 ID:???
>>387
動くか動かないか
389ご冗談でしょう?名無しさん:04/04/09 23:23 ID:ug/5V1aq
滑車におもりをつるし、両端を糸で固定する。一方の糸を加速度aで動かしたときおもりの加速度はいくらで運動するか? 
教えてください、お願いします
390ご冗談でしょう?名無しさん:04/04/09 23:29 ID:???
慣性の法則にレス下さったみなさん
ありがとうございました。>>380さんの説明で納得しました。
391ご冗談でしょう?名無しさん:04/04/09 23:51 ID:???
>>389
読めば読むほど意味のわからない問題文だな
392389:04/04/09 23:53 ID:ug/5V1aq
>>391 すいません。具体的にはどこがおかしいでしょうか?訂正しますので
393ご冗談でしょう?名無しさん:04/04/10 00:06 ID:???
>>392
何の両端をどういう風に糸で固定するのか。
固定した糸を動かせるのか。
そしておもりの加速度というものは運動するのか。
最後になぜ問題丸投げなのか。
このへんを訂正してください。
394389:04/04/10 00:13 ID:7aOfGrq8
滑車におもりをつるし、一方の端を糸で吊るし、天井に固定する。もう一方の端は指で持ち、 
その糸を加速度aで動かしたときおもりの加速度はいくらになるか? 
すいませんでした、ミスばっかでしたね。 
これは丸投げでなく、問題の途中でこれがわからないと先に進めないんです。どうぞ宜しくお願いします。 
395ご冗談でしょう?名無しさん:04/04/10 00:17 ID:???
そもそも固定した糸が動かせるってのが
おかしいだろ。問題文として。
問題丸投げだったらせめて、本文丸写しに汁
1京歩譲っても他端の糸を手で動かすのかと
小一時間(ry
396ご冗談でしょう?名無しさん:04/04/10 00:20 ID:???
粘着する気はないが、その文からしたら動滑車やろ
397389:04/04/10 00:26 ID:7aOfGrq8
>>395 一方の方は手で持ってるので動かせるんじゃありません?
398ご冗談でしょう?名無しさん:04/04/10 00:28 ID:7aOfGrq8
>>396 はい、動滑車です
399ご冗談でしょう?名無しさん:04/04/10 00:39 ID:cwHKe8oN
恒星を含まず惑星のみで構成された天体系は存在しないのでしょうか?
又ひとつの天体系に恒星はひとつだけと決まっているのでしょうか?恒星が
恒星の周りを回る天体系というのもありうるのでしょうか?
以上、簡単であれば理由も含めて教えて欲しいです。宜しく御願いします。
400ご冗談でしょう?名無しさん:04/04/10 01:02 ID:???
>>399
惑星の定義書いてみ。

んで後半は連星という。
401ご冗談でしょう?名無しさん:04/04/10 01:08 ID:cwHKe8oN
>>400
(定義)恒星の周囲を主に恒星の重力の影響を受けて公転し、自らは発光しない天体。

発光しない天体のみの天体系が存在しない(?)理由が分からないんですけど。
質量が小さいから?
402ご冗談でしょう?名無しさん:04/04/10 01:15 ID:???
>>401
存在しない理由は見あたらないが、何も光ってないんだから
見つけようがないだろw

木星級とかじゃ重力レンズ効果もほとんどなさそうだし。

見つけられないものに分類を与えてもしょうがないってことでは?
403ご冗談でしょう?名無しさん:04/04/10 01:20 ID:???
木星ってあとどのくらい重ければ恒星になれたのだろう?


あと重力で星間ガスが集まって星を形成するのにはガスのマスに下限があるような気がするんだけどどうなんでしょう?
こういうのって重力多体計算の人とかが専門なのかな?
404ご冗談でしょう?名無しさん:04/04/10 01:25 ID:???
あれね。
妄想なんだけど宇宙空間で水爆を使って強力な電磁波パルスを
発生させて、パラボラ状反射体でどこかの宇宙空間目指して
電磁波パルスを発射すればそのうち反射が帰ってくるかもね。
そしたら光ってない天体だけで出来た系を見つけられる可能性があるかも。
アクティブ電波天文みたいなw
405401:04/04/10 01:28 ID:cwHKe8oN
>>402
定義に恒星の周りを回るというのがあるので、惑星のみで存在しない理由が
あるのかと思ったのですが・・・。
406ご冗談でしょう?名無しさん:04/04/10 01:40 ID:???
>>405
そういうのはあっても惑星と呼ばれないだけだ。
そもそも「惑星」について語ってる時点で「惑星の定義に合致するもの」について語っている。
407401:04/04/10 01:52 ID:cwHKe8oN
>>406
発光しない天体のみの天体系

惑星と呼ばれてないだけで、存在はするんですか?
408ご冗談でしょう?名無しさん:04/04/10 01:53 ID:???
そういえばダークマターとかの話では光ってない天体だけで出来た系
ってのは考えてんのかね…
観測できないのにどうやって考慮に入れるのだろうか?
409ご冗談でしょう?名無しさん:04/04/10 01:54 ID:???
>>407
無い理由はないけど、光ってないから見たことある人が居ないって言ってるでしょ。
410ご冗談でしょう?名無しさん:04/04/10 01:58 ID:???
たとえばそこらじゅうを飛んでる彗星さんたちは恒星の周りは回ってない罠。
恒星の重力で軌道がかえられるにしても。
411ご冗談でしょう?名無しさん:04/04/10 02:11 ID:7aOfGrq8
滑車におもりをつるし、一方の端を糸で吊るし、天井に固定する。もう一方の端は指で持ち、 
その糸を加速度aで動かしたときおもりの加速度はいくらになるか? 
まじでお願いします
412ご冗談でしょう?名無しさん:04/04/10 02:21 ID:???
>>411
授業中寝るなよw
413ご冗談でしょう?名無しさん:04/04/10 02:22 ID:7aOfGrq8
>>412 寝ません。寝ないためにも教えてくださいお願いしますよ
414ご冗談でしょう?名無しさん:04/04/10 02:56 ID:???
>>408
21世紀の今では、観測手段は光学望遠鏡だけではないのですよ
415ご冗談でしょう?名無しさん:04/04/10 02:59 ID:???
だっておまえ長さが3:1のシーソーで1の方をx動かしたら
3の方はいくら動くよ?2xだろうが。
んで速度はdx/dtなんだから1の動かし方がX(t)というtの関数だった場合
1の速度はdX(t)/dtで、3の動かし方は3X(t)だから3の速度は簡単でしょ。
同じこともう一回すれば加速度の比も出るわけ。
何回やってもこの比がかわんないことは「微分の線形性」からきてる。

これと同じ事を滑車で動く距離の比がどうなるか考えてからやってみな。
416ご冗談でしょう?名無しさん:04/04/10 03:00 ID:???
おっと、「3の方はいくら動くよ?3xだろうが」ね。
417ご冗談でしょう?名無しさん:04/04/10 03:02 ID:???
>>414
木星みたいな星はどんな電磁波もほとんど出していないわけだが。
418ご冗談でしょう?名無しさん:04/04/10 03:03 ID:???
>>408
褐色矮星なんかは重力レンズ効果で数が見積もられてたりするだろ
419ご冗談でしょう?名無しさん:04/04/10 03:11 ID:???
>>418
質量が違いすぎw

木星みたいなのだけが数百個集まってる星系があったらどうする?
そういう可能性を潰していっているようには見えないのだが。
420ご冗談でしょう?名無しさん:04/04/10 03:35 ID:7aOfGrq8
>>415 X(t)=3Xですか?
421ご冗談でしょう?名無しさん:04/04/10 04:23 ID:???
>>419
どうする?って一体何が。
つーか通常のバリオン物質の量は宇宙の元素生成比から
だいたいこれくらいでないといけない、と見積もられてるだろ
422ご冗談でしょう?名無しさん:04/04/10 09:49 ID:tvEZFmOE
面指数(100)と(200)の違いがよくわからないのですが
物理的にどうちがうのですか?
423ご冗談でしょう?名無しさん:04/04/10 12:34 ID:WFXSpXk8
チェビシェフ・タウ法ってどういうものなんですか?
近いうちに人に説明しないといけない機会があるんですが
もし知ってる方がいれば一言(2,3行ぐらい?)でいいので説明
の例文を考えてください。おねがいします。
424ご冗談でしょう?名無しさん:04/04/10 13:30 ID:7/Zt/xtL
風呂沸かしていて思ったんですが、温かいのは上に冷たいのは下にいきますよね。

これは温度だけが移動してるんでしょうか?それとも水(分子?)自体が移動してるんでしょうか?
例えば、静かに水風呂の底の部分から
熱湯をいれていくと、その熱湯(熱い水分子)が移動して上にいくのか、
熱湯の熱(水分子の振動?)の部分だけが冷水に伝達されていって、
元々は熱かった水分子は冷えて上に元々あった冷水の部分が熱くなっていくのか。

分かりにくくてすいません。どなたか分かりますか?
425ご冗談でしょう?名無しさん:04/04/10 13:49 ID:???
>>424
温度によって密度が違うから。
426ご冗談でしょう?名無しさん:04/04/10 14:03 ID:???
>>424
熱は伝導と対流(と放射)で伝わる。
>例えば、静かに水風呂の底の部分から
>熱湯をいれていくと、その熱湯(熱い水分子)が移動して上にいくのか、
これが対流。
>熱湯の熱(水分子の振動?)の部分だけが冷水に伝達されていって、
>元々は熱かった水分子は冷えて上に元々あった冷水の部分が熱くなっていくのか。
これが伝導。
427ご冗談でしょう?名無しさん:04/04/10 14:28 ID:???
>>424
>>426
んでもって対流と伝導のどっちの寄与が大きいかってのは
条件によるってことも補足しようね。

コンピューターで数値計算しないと何とも言えないわな。
428ご冗談でしょう?名無しさん:04/04/10 19:16 ID:???
固体、液体、気体だと
右に行くほど対流の寄与が大きくなり
左に行くほど伝導の効果が大きくなるな
429ご冗談でしょう?名無しさん:04/04/10 19:22 ID:8rNr8BRT
光速より速い光はあるのでしょうか
430ご冗談でしょう?名無しさん:04/04/10 19:26 ID:???
物質中での特定の挙動に関する速度なら
431ままやら:04/04/10 19:27 ID:gw3F90x6
鏡に映っている文字が裏返しなのは、どうしてですか
それをきちんと答えられる学者は世界でも2,3人しかいないとは本当ですか
432ご冗談でしょう?名無しさん:04/04/10 19:35 ID:???
光の反射を考えれば当たり前。
433ご冗談でしょう?名無しさん:04/04/10 21:28 ID:JJqn7R7N
マイケルソン・モーレーの実験についてお聞きします。
東西南北どこへ光を照射しようとも古典物理学的に同じ慣性系の中にあるので、
速度合成の結果、誤差は発生しない、とすることはなぜできないのでしょうか?

相対論を否定するとかそういうわけではなく、単純に頭が悪く理解できなかったのでご教授ください。
グーグルで検索を一応かけてみましたが、光速度不変を示す実験としての紹介か「と」な否定論しかなく、
わたしの疑問に答えるところを見つけられませんでした。
申し訳ありませんが、文系で数式に自信がありませんので、
できれば平易な言葉で説明をいただければと思います。
434ご冗談でしょう?名無しさん:04/04/10 21:35 ID:???
マクスウェル方程式より
435ご冗談でしょう?名無しさん:04/04/10 21:47 ID:???
>>433
実験室がエーテル(光波の媒質)に対して運動しているならば、
方向によって光の速さは違うはずである。
マイケルソンとモーリーは方向による光速の違いを検出することで
エーテルの存在を実証しようとしたが、検出できなかった。
結果的に、この実験はエーテルの不存在の傍証となった。

という話の筋は、理解しているか?
436ご冗談でしょう?名無しさん:04/04/10 21:59 ID:JJqn7R7N
>>434
早速のレスありがとうございます。現在、マクスウェル方程式も含めて再検索をしてみました。
・・・理解するにはだいぶ時間がかかりそうですが。
>>435
はい。そこまではとりあえず把握しております。
当時の天文観察によって、(あるならば)エーテルは宇宙に対して静止しているとされていたので、
公転により東西方向へエーテルの風が吹いているという前提で行われたんですよね。
437ご冗談でしょう?名無しさん:04/04/10 22:05 ID:???
>>433
マイケルソン・モーレーの実験は、光の媒質たるエーテルの検出をもくろんだ実験。
エーテルが地球静止系に対して運動しているなら、行きと帰りで速度は違うはず。
エーテルが地球静止系に対して静止しているなら、おっしゃる通りの事になるわけですが
それは要するに天動説に他なりません。
地球の太陽に対する公転、銀河中心に対する公転等の全てが、実は
地球を中心とした動きである事になります。
438ご冗談でしょう?名無しさん:04/04/10 22:06 ID:???
○| ̄|_出遅れた
439ご冗談でしょう?名無しさん:04/04/10 22:27 ID:???
>>436
それを理解しているなら何も問題はないはずだが……。
「古典物理学的に同じ慣性系の中にあるので、速度合成の結果、誤差は発生しない」
という文が何を訊きたいのか解らなくなってきたぞ。
変に古典的だの何だの専門用語を使おうとせず、自分の言葉で訊いてくれ。
440ご冗談でしょう?名無しさん:04/04/10 22:42 ID:JJqn7R7N
>>439
MMの実験は絶対静止エーテルの存在を否定した。というだけなのか。
それとも、この実験のみで光速度不変の原理が実証されたのか。

もし、これが光速度不変を表しているのなら、
なぜ光を粒子ととらえて絶対静止エーテルの有無にかかわらず、
地球の公転という慣性系の中での運動と見なさなかったのか。

主に後半部分が疑問となったのです。
言いたいことがうまく伝わるかな。
きっと理解されている方には意味のわからないことを言っているのでしょう・・・
441ご冗談でしょう?名無しさん:04/04/10 22:59 ID:???
>もし、これが光速度不変を表しているのなら、
>なぜ光を粒子ととらえて絶対静止エーテルの有無にかかわらず、
>地球の公転という慣性系の中での運動と見なさなかったのか。

意味不明だぞ
442ご冗談でしょう?名無しさん:04/04/10 23:04 ID:???
MM実験は「失敗」の実験。エーテル流の速度を検出できなかったから。
そしてアインシュタインが光速度不変を提唱するまで
誰もこの実験を適切に解釈できなかった。

後半に関しては、幾ら粒子的に見ようとしても、観測事実としての波動的性質は
無視できないでしょう。
そもそもこの時点では、波動説の方が優勢だった。
光量子仮説、特殊相対論、ブラウン運動は全て1905年の発表。
443ご冗談でしょう?名無しさん:04/04/10 23:07 ID:???
>>440
>MMの実験は絶対静止エーテルの存在を否定した。というだけなのか。
そうです。
>それとも、この実験のみで光速度不変の原理が実証されたのか。
ちがいます。

光速不変の原理とは、「光の伝わる速度は光源の運動に無関係」ということだ。
検証したければ、観測者に近づく光源と観測者から遠ざかる光源とを用意して
2つの光速を比べる実験をするべきだ。
(連星から来る光の速さを調べるとか、π中間子の崩壊で出た2光子の速さを測るとか)
マイケルソンとモーリーの実験装置は、そうはなっていないだろう。
444ご冗談でしょう?名無しさん:04/04/10 23:08 ID:???
おいおい
実験に於いて否定的結果を実証するのと失敗するのとでは雲泥の差があるぞ。
445442:04/04/10 23:12 ID:???
>>444
んあ、センセーショナルに書き過ぎたか。一応「」付きにはしたが。
446ご冗談でしょう?名無しさん:04/04/10 23:14 ID:WFXSpXk8
テンソル解析を勉強したいんですが
初心者でもわかりやすい本ないですか?
演習問題とその答えがわかりやすく解説されてたりする
本とかがほしいんですがそんなの無いですよね?
447ご冗談でしょう?名無しさん:04/04/10 23:19 ID:???
>>446
・分野は? 相対論か、連続体力学か、それとも?
・日本語か英語か
448ご冗談でしょう?名無しさん:04/04/10 23:22 ID:JJqn7R7N
>>441
ひぃ(汗)

電車の中で投げるボール(進行方向とそれに垂直方向へ同じ速度で)と
この実験とでどのように違うのか、ですね。

もちろん、エーテルを想定して行った実験ですので、
エーテルという媒質はあり得ない
(捕捉:≒光が絶対静止している慣性系を伝わる波ではない)
ということが証明されることはわかりました。

が、この実験がそのまま「光は速度が合成されない」という結論に結びつく過程がわからないのです。
この実験とは別に、
(*)我々から見て運動をしている物体から、進行方向とその逆方向へ光を照射し、
その速度を我々が観測しなければ光の速度が一定であるとは言えない、
とわたしには思えるのです。

* もちろん、我々から見て静止している場所から放った光との比較でもかまいません。
449ご冗談でしょう?名無しさん:04/04/10 23:25 ID:???
>>443
うぉぉぉ! だらだらと書き込んでいる間に、一番待っていた答えが!!
なるほど、これ自体には光速度不変の原理を示す要素は含まれていないのですね。
特殊相対性理論の「きっかけ」として紹介されるだけに、
これがそのまま光速度不変の原理を表していると勘違いしていました。
回答ありがとうございます。

また、わたしの舌足らず、勉強不足な質問に親切につきあってくださった方々、
皆さん、ありがとうございました。
450ままやら:04/04/10 23:30 ID:5ke9r+yg
鏡に光りが反射するならどうして上下も反射しないのですか
451ご冗談でしょう?名無しさん:04/04/10 23:31 ID:WFXSpXk8
>>447
相対論でおねがいします
452ご冗談でしょう?名無しさん:04/04/10 23:32 ID:WFXSpXk8
>>447
あ、あと日本語でおねがいします
453ご冗談でしょう?名無しさん:04/04/10 23:40 ID:???
>>449
包まれてないといえば嘘になるが、まあそんなとこだろ。
>>451-452
共立出版物理数学OnePoint「物理とテンソル」(中村純)あたりでどうだ。
思いっきり薄いがとっつきやすいとは思うぞ。
454ご冗談でしょう?名無しさん:04/04/10 23:40 ID:YGCYUIlu
しょうがっこう 3ねんせいのミキティーです みんなよろしくネ




原子も星もとにかく まわりまくってる この宇宙なんですが

すっごい おおきい人が 宇宙中のまわっている星を ぜんぶ いっっぺんに

ビタッ ダメッ! って手で止めたらどうなるんですか?

べつに なんにも おきないのかなぁ




おじさんたちが ミキティーにおしえてネ

 
455ご冗談でしょう?名無しさん:04/04/10 23:41 ID:???
包まれて→含まれて
456ご冗談でしょう?名無しさん:04/04/11 00:04 ID:+PLjgabV
>>453
情報ありがとうございました
でも調べてみるとそれって99ページしかないんですね
なんかちょっと頼りない感じもするんですが
どうなんでしょう?
わかりやすいに越した事はないんですが
しっかりと内容を十分に理解できる本なんでしょうか?
457ご冗談でしょう?名無しさん:04/04/11 00:20 ID:???
>>456
初心者でもわかりやすい本って注文だったからな。値段相応の薄さと内容だよ。
入門書のレベルでこれ一冊あれば何もかも安心なんて本があるわきゃない。
頼りないと思うんならそれまでだ、おとなしくSchaum's Outline of Tensor Calculusでも買っとけ
458ご冗談でしょう?名無しさん:04/04/11 00:48 ID:+PLjgabV
>>457
あの、初心者といっても
テンソル解析についての初心者ということであって
数学初心者というわけではないですのです
一応線形代数は一通り勉強しております

ということで
そんな自分にお勧めな本をもう1,2冊ぐらい
教えていただきたいです。おねがいします。
459ご冗談でしょう?名無しさん:04/04/11 03:48 ID:???
>>449
>これがそのまま光速度不変の原理を表していると勘違いしていました。

あらわしていると言っても別におかしくない
460ご冗談でしょう?名無しさん:04/04/11 14:55 ID:???
>>458
「テンソル解析」(田代嘉宏、裳華房)
「テンソル解析入門」(伊理正夫、教育出版)
あとは佐藤の「相対性理論」等の相対性理論の本を読めば、テンソルの説明がされている。
461ご冗談でしょう?名無しさん:04/04/11 17:20 ID:+PLjgabV
>>460
ありがとうございました
佐藤さんの相対性理論は持ってるんで
残りの2つとも買ってみます
462ご冗談でしょう?名無しさん:04/04/11 19:54 ID:ZFdGomu1
質量がそれぞれm,Mの物体A,Bがある。Aが速度uでBに衝突した後、AはBを突き抜け、Aは衝突前と同じ向きに速度u/2で進んだ。このとき、mとMのの間にはどのような関係がなければいけないか。
よろしくお願いします
463ご冗談でしょう?名無しさん:04/04/11 20:36 ID:???
>>1
宿題を聞くときは、どこまでやってみてどこが分からないのかを書くこと。
丸投げはダメだからね。
(丸投げ君は完全無視。答えるだけ無駄。)
464462:04/04/11 21:01 ID:ZFdGomu1
>>463 運動量保存からBの衝突後の速度V=(mu)/(2M)までわかりましたが、その後が…
465ご冗談でしょう?名無しさん:04/04/11 21:13 ID:???
問題文の条件からその速度の可能な範囲を求めれば良いんで無いの?
466ご冗談でしょう?名無しさん:04/04/11 21:31 ID:???
理想気体のEntropyの式ってどんなでしたっけ。

S = N ln(eV/N) + N cv ln T + ?

って感じだった気がするんですが思い出せません。
467ご冗談でしょう?名無しさん:04/04/11 21:55 ID:???
それくらい計算すればいいのに。
468ご冗談でしょう?名無しさん:04/04/12 18:15 ID:???
遅レスだが……

>>403
木星の質量が80倍あれば、恒星になれたと言われてる。
ちなみにこれは太陽質量の0.08倍。

>>431
鏡に映っている像は、前後(鏡面に垂直な方向)が反対になるから。

>>450
「上下も反射」の意味が不明だが、仮に「上下も反対」と解釈すると、
回答は「左右も上下も反対にならない。前後が反対になる。」
469教えてチャン:04/04/12 19:22 ID:/oxabT5A
お願いがあります
全然わからないので教えてください
ヒントでもなんでもよいです
よろしくお願いいたします

初速度ゼロの1価イオンを電位差Vボルトのグリッドで
加速した後、真空管を自由に距離Lmだけとばせたところ
t秒かかった。イオンの質量Mを求めよ

なんですけど・・。お願いいたします!!
470ご冗談でしょう?名無しさん:04/04/12 19:41 ID:???
加速後のイオンの運動エネルギーは?
加速後のイオンの速さは?
運動エネルギーと速さと質量の関係は?
471ご冗談でしょう?名無しさん:04/04/13 00:42 ID:CStRGt+b
どうして大気圏突入の際のロケットなどには負担がかかるのでしょうか?
空気との摩擦かとも思ったのですがそれだと「突入の際」にのみ負担がかかることの
説明がつかない様な気がします。
どなたか宜しく御願いします。
472ご冗談でしょう?名無しさん:04/04/13 01:11 ID:???
当然打ち上げの時にも空気摩擦による加熱は起こっているわけだが。

473ご冗談でしょう?名無しさん:04/04/13 01:13 ID:???
送ってしまった

>>471
打ち上げの時は、どうやって上昇している?
再突入の時は、どうやって下降している?
474ご冗談でしょう?名無しさん:04/04/13 01:15 ID:???
>>471
打ち上げ時の速度<<降下時の速度だから。
475ご冗談でしょう?名無しさん:04/04/13 01:16 ID:???
ロケットエンジンでの加速には限界があるけど、
重力による加速は空気抵抗とつりあうまでいくらでも加速できるから。
476ご冗談でしょう?名無しさん:04/04/13 01:18 ID:tmNfwcUT
どなたか単位面積あたり±σの電荷を与えた2枚の金属平板の単位面積あたりに
働く力を求めてください。
477ご冗談でしょう?名無しさん:04/04/13 01:26 ID:tmNfwcUT
http://private.rocketbeach.com/~mms1617/2ch/upload/dat/0060.jpg

幅wの2枚の長い金属平板が、距離d はなれておかれている。
d<<wで、板の暑さは考えなくてよく、長さは無限に長いと考えてよい。
この金属板に向きの異なる電流Iを流したとき、出来る磁場Bを求めなさい。


↑これまったくわかりません
だれか解き方おしえてください
478ご冗談でしょう?名無しさん:04/04/13 01:53 ID:???
>>476
教科書なり参考書なり開こうよ…
479ご冗談でしょう?名無しさん:04/04/13 02:29 ID:???
>>1
宿題を聞くときは、どこまでやってみてどこが分からないのかを書くこと。
丸投げはダメだからね。
(丸投げ君は完全無視。答えるだけ無駄。)
480471:04/04/13 02:42 ID:CStRGt+b
説明不足で済みません
「突入の際」というのは打ち上げ時との比較ではなくて、
大気圏内にすっぽり入ってしまった時との比較なんです
「大気圏外から大気圏内に突入する瞬間」に最も負担がかかると思っていたのですが
違うのですか?大気圏内に入っても
減速するまで同じ位の負担がかかり続けるんですか?
481ご冗談でしょう?名無しさん:04/04/13 02:59 ID:???
時々図をどっかにうぷする努力はするが教科書を開く努力はまったくしないのが表れるな。
前者と後者の努力のオーダーはそんなに桁でちがうものだろうか。
482ご冗談でしょう?名無しさん:04/04/13 05:04 ID:???
>>>>>>>>>>474
483ご冗談でしょう?名無しさん:04/04/13 06:47 ID:???
>>1
宿題を聞くときは、どこまでやってみてどこが分からないのかを書くこと。
丸投げはダメだからね。
(丸投げ君は完全無視。答えるだけ無駄。)
484ご冗談でしょう?名無しさん:04/04/13 08:56 ID:???
>>477
磁場は金属板間にだけできることに注意してアンペ−ルの法則を適用する。
485ご冗談でしょう?名無しさん:04/04/13 11:56 ID:???
>>480
大気圏内外の境目があると思ってる?水面のような。
486ご冗談でしょう?名無しさん:04/04/13 16:30 ID:p5OMmZOy
直列に接続したn個のコンデンサについて、
コンデンサの極板に現れる電荷が全て共通になるには何故ですか
487ご冗談でしょう?名無しさん:04/04/13 16:36 ID:???
電荷量が保存するからだね。
488471:04/04/13 19:08 ID:CStRGt+b
>>485
水面の様なはっきりした境目があるとは思ってませんが、巨視的に見れば境界と
呼んで良い様な領域があるのかと思っていたのですが。
それでロケットの着陸時に「負担が最小になる様、大気圏に丁度良い角度で突入」するのは
その「境界」辺りで最も負担がかかるという事なのかなーと。
489ご冗談でしょう?名無しさん:04/04/13 21:43 ID:???
>>488
気圧は連続的に変化するので明確な境界はない。

負荷は気圧とスピードに概ね比例するので、適度に気圧が上がって来て
なおかつまだそれほどスピードの落ちていないあるところで負荷は最大になる
490ご冗談でしょう?名無しさん:04/04/13 22:46 ID:Vdnsr1TM
物理の世界で、変位xを時間tで微分する時、 「xの上に小さい点を1コつけた表現」がよくでてきます。 これって、「時間で1階微分だよ」と解釈していいのでしょうか? (点の数が階数を表す) 時間以外での微分時は d/d? の表現を使うしかないですよね?
491ご冗談でしょう?名無しさん:04/04/13 22:50 ID:???
>>490
>これって、「時間で1階微分だよ」と解釈していいのでしょうか? (点の数が階数を表す)
そうです。
>時間以外での微分時は d/d? の表現を使うしかないですよね?
d/d? の使用をお勧めします。
t微分はドット、x微分はプライム (') を使う人がまれにいますが、やめといたほうがいいような気がします。
492ご冗談でしょう?名無しさん:04/04/13 23:23 ID:n1CZDNL/
イラクから生還した今井君が純粋なU238を1kg持って6畳部屋に引きこもったとします。
換気は週に一度しかしません。
さて、今井君が癌になる可能性は有意に上がるのでしょうか?

おまけ:
http://sta-atm.jst.go.jp/atomica/15090215_1.html
http://sta-atm.jst.go.jp/atomica/04030202_1.html
http://www.mext.go.jp/b_menu/shingi/housha/ssiryo73/s4.htm
493490:04/04/13 23:27 ID:???
>>491 ありがとうございました〜。
494ご冗談でしょう?名無しさん:04/04/13 23:36 ID:???
>>492
今井君は生還することは出来ません。
よって仮定が間違っています。
495ご冗談でしょう?名無しさん:04/04/13 23:44 ID:6U4Ny/F+
ほんとにチョッとした質問ですけど。鏡に映った映像って目の良し悪し
関係なく同じものですよね?そんで目が悪い自分は、鏡で遠くの物を
写して見たんですがやっぱり見えないんですよ。
もちろん目が悪いと言っても新聞を30センチ離して読めるくらい
の視力はあります。
496ご冗談でしょう?名無しさん:04/04/13 23:46 ID:???
何が疑問なのか分かりません。なぜ自分の視力が弱いか?
497ご冗談でしょう?名無しさん:04/04/13 23:48 ID:???
>>495
鏡で見たって遠いものは見えんよ
絵じゃないんだから
498ご冗談でしょう?名無しさん:04/04/14 00:07 ID:???
>>495
その状況は、鏡を含む平面に関して自分の位置と対称な位置に
いる場合と光学的に同じ。
499ご冗談でしょう?名無しさん:04/04/14 00:26 ID:Xyq9SfhC
>>496文書足らずですねすいません。補足すると目が悪いと言っても
目からの距離30センチ位なら目が良い人と同じく小さい字でも読
めるんですよ、なら鏡で遠くの物を写して近くで見れば見えるので
は無いか?と思ったんですがやはり見えなかった・・・
と言う話なんです。まあ自分でも浅はかな考えかと思うんですけど
理由がわからなかったもんで。
>>497 その通り見えませんでしたw
>>498 つまり鏡で写しても遠いもんは遠いってことですね。
   なんか解ったような気がします。ありがとうございます。
500ご冗談でしょう?名無しさん:04/04/14 01:33 ID:???
スレ読め。まったく同じ質問があるぞ。
実はこのスレ見て読んだことを忘れて
同じ質問を思いついたように錯覚してるんじゃないのか。
501ご冗談でしょう?名無しさん:04/04/14 02:58 ID:IEptqGWJ
「波の位相がπずれる」の意味がよくわかりません
そもそも位相というものがよくわかりません
解説お願いします
502ご冗談でしょう?名無しさん:04/04/14 03:37 ID:???
>>501
y=A sin (ωt-kx)
Aが振幅、(ωt-kx)が位相。
直感的には、振幅は波のマクロな「大きさ」を決定し、位相は波の局所的な状態を決定する。
例えば光も波だが、光の明るさには振幅(振幅の2乗)が寄与し、位相は関与しない。
「単独の」光の性質には位相は関与しない。
複数の光源から出た光が交差するとき、位相の違いによって暗くなったり明るくなったりする。
これは、局所的に明るい部分が強めあったり打ち消しあったりすることで生じる、波の位相から来る現象。
503ご冗談でしょう?名無しさん:04/04/14 07:34 ID:???
波の位相と位相幾何の位相は何か共通項があるの?
504ご冗談でしょう?名無しさん:04/04/14 07:34 ID:???
age
505ご冗談でしょう?名無しさん:04/04/14 13:38 ID:xbHrqbIm
アホ学生ですが,アンペールの法則が本を見たりしてもさっぱり理解できません。
だれかアホにもわかるように教えて下さい。
506ご冗談でしょう?名無しさん:04/04/14 18:35 ID:b18obaCV
2次元平面上を運動する質量mの質点を考える。平面に2次元直交座標
(x,y)を導入する。ポテンシャルエネルギーV(x,y)がA,Bを正の定数として
V(x,y)=-A(x^2+y^2)+b(x^2+y^2)^2であたえられるとき、質点をx軸上の点(x1,0)(x1>0)から
静かに話したところ、質点はx軸上で運動して原点(0,0)で速度が0になった。このことより
座標x1は(?)で与えられる



この(?)内の答えがわかりません
ヒント教えてください
507ご冗談でしょう?名無しさん:04/04/14 18:40 ID:b18obaCV
>>505
Hの周回積分はIなので
2πdH=I
H=I/2πd
B=μ0 I/2πd

また
F=IBLより
=μ0I1 I2 L/2πd
だよ
508ご冗談でしょう?名無しさん:04/04/14 19:13 ID:???
>>506
(0,0),(x1,0)での運動エネルギーはともに0
また原点でのポテンシャルエネルギーもV(0,0)=0

以上のことから、エネルギー保存則が成り立つために
V(x1,0)が満たすべき条件とは?
509ご冗談でしょう?名無しさん:04/04/14 19:41 ID:b18obaCV
>>508

xで微分して0になる点がなんとなく関係してそうな感じがしますが
いまいちよくわかりません。もう少しヒントください。
あと、一つ思ったのですが、(x1,0)の点で静かに離して左にうんどうして
(0、0)の点で速度が0になってるわけだから
(0,0)と(x1,0)点には正のポテンシャルエネルギーがあるように思うんですが
違うんでしょうか?

510ご冗談でしょう?名無しさん:04/04/14 20:21 ID:???
>>509
 問題解くには直接関係ないが、r^2=x^2+y^2を使ってポテンシャルの
概形をつかむとよいかも(VをR2=r^2で表す、とか)。
 後は>>508を読み直せ、それでわからなけりゃポテンシャルの定義を
勉強し直せ、としか・・・。
511ご冗談でしょう?名無しさん:04/04/14 23:29 ID:???
>>507
それは直線電流のつくる磁場をアンペールの法則を用いて求めて
いるんだと思うが、アンペールの法則の説明にはなっていない。
512ご冗談でしょう?名無しさん:04/04/14 23:56 ID:q+Gwr46R
今この世界はどのようにして生まれたのか。地球とはなにか。
物質とは何か。原子とは何か。宇宙とは何か。そして人間とは。
宇宙はいつはじまったの?そもそもはじまりって?そんなもの
が存在するならばはじまりの前はなんだったの?
宇宙の元を「誰」が作った?その「誰」を誰が作った?その前は?
宇宙の果てとは?地球から一定方向に進んでいったら最終的には
どこにたどりつく?今俺たちがこうしていることはなんなんだ?

人生板でコンナ質問があった。
特に宇宙の起源と宇宙の果てについて教えろ
513ご冗談でしょう?名無しさん:04/04/15 00:05 ID:???
>>512
それは物理学の最終問題。
それに答えが出てるなら、もう学問として終了しているはず。
もしそれでも何か答えろっていうなら、妄想詩人達の回答が色々あるから
その中で気に入ったのに飛びついとけ。
514ご冗談でしょう?名無しさん:04/04/15 00:12 ID:vyfFHmlT
>>513
いいね、そういっとくよ。
サンクス
515ご冗談でしょう?名無しさん:04/04/15 03:02 ID:???
>それに答えが出てるなら、もう学問として終了しているはず。

んなわけねえじゃん
516ご冗談でしょう?名無しさん:04/04/15 03:09 ID:???
>>515
つっこみだけじゃなく
反論しろよクズが
517ご冗談でしょう?名無しさん:04/04/15 03:19 ID:???
>>516
統一的な理論の枠組が完成し、いろいろ証拠が見つかったとしても
それは限られた何らかの現象を解析したのみで
一般的に巷に溢れてるさまざまな現象に直接応用できるものではありえない。
したがって、513の言ってることが正しいとするならば、
「宇宙の起源と果て(513のいってる最終問題ってこれだよね?)」
を解明するためにはその他のあらゆる問題をモデル化して
解析し終わっているかまたは最終問題を解明した理論により
近似的モデル化を行うことなく直接的に問題を解くことができることを言わなければならない
これは非自明なことだ
少なくとも、既存の物理体系からの類推では疑わしい将来像と言わざるを得ないな
518ご冗談でしょう?名無しさん:04/04/15 03:23 ID:???
>>516
補足。「学問として終了している」というのが
「もう学問としてつまんねえ」と言う意味ならば、
それは個人の主観的問題ということで。
ま、どっちみち513が512に言いたいことの本質ではないでしょ、この部分は。
基礎物理の話だろうから。
519ご冗談でしょう?名無しさん:04/04/15 03:24 ID:???
>>517
そんな反論かよ。
せこいな
520ご冗談でしょう?名無しさん:04/04/15 03:26 ID:???
>>518
512への答えをどうぞ!
教授
521ご冗談でしょう?名無しさん:04/04/15 09:15 ID:???
>>520
別に513を全否定してるわけではないのに、
なぜそんなに絡まれるのかわかんない。
>>519
せめて自分で言ったことくらい自分で実践して欲しかった。
522ご冗談でしょう?名無しさん:04/04/15 09:24 ID:???
523ご冗談でしょう?名無しさん:04/04/15 09:25 ID:???
524ご冗談でしょう?名無しさん:04/04/15 18:48 ID:mD3XARPj
厨房的な質問で大変恐縮なんですが
力のモーメントと慣性モーメントが物理的にどのような量を表しているのかわかりません
解説お願いします
525ご冗談でしょう?名無しさん:04/04/15 18:51 ID:???
地球の数分の一くらいの、とんでもなくデカイ物体が
地球上にあったらどうなるの?
526ご冗談でしょう?名無しさん:04/04/15 18:52 ID:???
教科書に書いてます
527ご冗談でしょう?名無しさん:04/04/15 18:57 ID:???
>>524
高校で習った、力Fと運動量Pと同様に考えればよい
528ご冗談でしょう?名無しさん:04/04/15 18:59 ID:UMkPooZz
トランジスタの周囲温度が一定であると仮定すると、
トランジスタに消費される電力が一定ならば、 トラン
ジスタ内部の温度も一定になる。

これを熱力学的に説明したいのですが、
電力と熱の関係が発熱量=電力×時間ということしかわからず
電力一定と温度一定を結びつけることができません。
529ご冗談でしょう?名無しさん:04/04/15 18:59 ID:???
>>525
潮汐力が働き、地球は変形する
ただし、そのとんでもなくデカイ物体は
重力以外の力による結合力がそんなに強力でなければ
地球から受ける潮汐力でバラバラに分解されてしまう。

重力のみで形を保っている物体が潮汐力で破壊されてしまわない
ぎりぎりの距離をロッシュ限界と言う
↑は物体の質量と大きさで決まる
530525:04/04/15 19:05 ID:???
へぇ〜、なるほど。
自分は文系人間なんだが、ちょっと思い浮かんだアフォみたいな疑問でも
答えがちゃんとあることに感動しました。
サンクス。
531ご冗談でしょう?名無しさん:04/04/15 19:21 ID:mD3XARPj
>>527
?????
532ご冗談でしょう?名無しさん:04/04/15 19:31 ID:???
>>527
慣性モーメントに対応するのは質量だな。
で、運動量に対応するのは角運動量。
533ご冗談でしょう?名無しさん:04/04/15 20:11 ID:???
>>528
トランジスタ内部の温度が一定なら消費電力も一定だが、逆は言えないのでは?
534528:04/04/15 20:36 ID:IWg06JRb
>>533

伝熱工学の熱抵抗という考えを持ち出せば、
伝熱量[W]=温度差[℃]/熱抵抗[℃/W]
で伝熱量をPとして説明はできるみたいなんですが、
これを熱力学的になんとか説明したいのですが・・・
参考
http://www.necel.com/ja/faq/f_therma.html
ジャンクション温度と熱抵抗
535528:04/04/15 20:39 ID:???
>>534
訂正
伝熱量を電力Pc(コレクタ電力)として説明はできるみたいなんですが、
この熱抵抗を用いないで熱力学的なんとか説明したいのですが
536ご冗談でしょう?名無しさん:04/04/15 20:48 ID:t9pTPXAN
英語で書かれた論文ってどのように勉強していったらいいか分かりません。


訳し方やノートの取り方など具体的に教えて下さい。
537ご冗談でしょう?名無しさん:04/04/15 20:53 ID:???
>>536
ノートなんか取らない。
気になることがあったら論文に書き込む。

英語が読めないのは英語の力が足りないから。
論文用の特別な訳し方なんかない。
538ご冗談でしょう?名無しさん:04/04/15 21:47 ID:???
混じれ酢を入れると
理系の論文はSVOばっかだから
理系単語力の問題
539ご冗談でしょう?名無しさん:04/04/15 21:57 ID:???
英語の論文って一般に単語が難しいだけで英文は平坦なものが多い
540ご冗談でしょう?名無しさん:04/04/15 22:14 ID:xWmt3NY1
何故A/m = N/Wbになるのですか?
541ご冗談でしょう?名無しさん:04/04/15 22:16 ID:???
英語の教科書はたまに複雑な文章になってたりするよな
あ、ジョークかよこれ。みたいのも
542ご冗談でしょう?名無しさん:04/04/15 22:42 ID:???
これって本当なんですか?
http://www.ggm.to/  
543ご冗談でしょう?名無しさん:04/04/15 23:14 ID:???
>>542
有名なとんでもサイトだよ
544ご冗談でしょう?名無しさん:04/04/16 00:26 ID:CJSCvd39
 すいません。明日初めての実験なんですが、レポートの書き方がいまいちわかりません。
 レポートの『装置・方法』を書くところには、教科書に載っているのをほぼ丸写しでよろしいのでしょうか?
 
545ご冗談でしょう?名無しさん:04/04/16 00:38 ID:???
ok
546ご冗談でしょう?名無しさん:04/04/16 00:47 ID:???
>>544
うちも方法はほぼ丸写しだった。多少自分のスタイルに書き換えることはしたけど。
重要なのは考察……よりもレポートの書き方だったかな。
読みにくかったらいくら内容がよくてもだめだから。
547544:04/04/16 01:04 ID:???
>>545>>546
早急なレスありがとうございますm(__)m
書き方勉強します( ..)φ
548ご冗談でしょう?名無しさん:04/04/16 01:41 ID:???
原点に電荷Qがあるとする。
静電界における電場ベクトルをE(r)(ここでrは原点からの距離)とすれば
rot(E(r))=0が常に成り立つことをストークスの定理から示せ。

↑解説きぼんぬ
549ご冗談でしょう?名無しさん:04/04/16 01:48 ID:FWCa86R6
>>548
まずストークスの定理の解説きぽんぬ
550ご冗談でしょう?名無しさん:04/04/16 01:56 ID:???
閉曲線∂Sに囲まれた曲面をSとするならば
∫[S] n・rot(E)dS=∫[∂S]E・ds   (n:長さが1の法面ベクトル)

右辺は閉曲線∂Sに沿う線積分
551ご冗談でしょう?名無しさん:04/04/16 04:57 ID:???
閉曲面では∂S=Φだから
∫[S] n・rot(E)dS=∫[∂S]E・ds=0
となるからrot(E)=0になるのかなあ

物理および数学に詳しい人の書き込みお願いします。
552ご冗談でしょう?名無しさん:04/04/16 15:26 ID:???
量子なんとかってやってるやつ。
それもう未来技術板でやれ。そっちのほうが板と趣旨が合ってるだろ
553552:04/04/16 15:27 ID:CIZmG9BX
554ご冗談でしょう?名無しさん:04/04/16 17:20 ID:???
1000で自爆して終わりか…ある意味壮絶な最期でした。チーン
555ご冗談でしょう?名無しさん:04/04/16 18:19 ID:???
>>553
相対性理論は間違ってる系トンデモは多数あるが、
量子力学系トンデモというのは珍しい。
貴重なサンプルです。
556ご冗談でしょう?名無しさん:04/04/16 19:06 ID:???
××は間違ってる系トンデモは多数あるが
××は正しい、から出発して、他の追随を許さない領域まで
逝ってしまった系トンデモ、という意味でも珍しい
557ご冗談でしょう?名無しさん:04/04/16 19:41 ID:???
「うぇうぇ」なる煽り文句を広めたことがあいつの唯一にして最大の業績だな。うぇうぇうぇ
558ご冗談でしょう?名無しさん:04/04/16 20:23 ID:???
999 名前:ご冗談でしょう?名無しさん :04/04/16 15:21 ID:???

 ここまでのまとめ

 kn2fEhGoは数学を知らない
 kn2fEhGoは工学を知らない
 kn2fEhGoは物理学を知らない
 kn2fEhGoは統計力学を知らない  
 とりわけ熱力学第二法則を知らない
 kn2fEhGoは量子力学を知らない
 kn2fEhGoは人の質問に答え(られ)ない
 kn2fEhGoはYes/Noも知らない
 kn2fEhGoは方程式も知らない
 kn2fEhGoは日本語も知らない
 kn2fEhGoは常識も知らない
 kn2fEhGoは良識も知らない


1000 名前:ご冗談でしょう?名無しさん :04/04/16 15:21 ID:kn2fEhGo
 >>999
 それは「現段階で」だ。
559ご冗談でしょう?名無しさん:04/04/16 21:34 ID:kn2fEhGo
今のぞいてみたが、正直強制IDの未来技術板の方が
レベル高いな。
もう物理板ってばリア厨とかリア工の自動宿題解決機
位にしか使われてないんじゃないか?
だからレベル低いのばっかり。
107の理論を完成させたら未来技術板にたててやろう。
そのときは一レスごとにキルレシオ++という凄まじい戦場になるだろうなwwうぇうぇうぇ
560ご冗談でしょう?名無しさん:04/04/16 21:56 ID:???
>>559
うぇうぇうぇは、物理学で反論されずに済む未来技術板へ逃げることにしました。
561ご冗談でしょう?名無しさん:04/04/16 22:03 ID:???
>>559
未来技術板では架空の技術を使ってもいいから、そもそも議論にならん。
普通のタイムマシーンの方が便利である、で終わり。
562ご冗談でしょう?名無しさん:04/04/16 22:16 ID:???
>>559
ああそうだね。レベルの高い君にはこんな低レベルな過疎板は似合わないよ。
だからレベルの高い未来技術板で頑張ってくださいね。
何があっても戻ってきちゃダメですよ。

永遠にさよなら
563ご冗談でしょう?名無しさん:04/04/17 01:16 ID:???
漏れのところに、相間系トンデモさんから理論の検証(査読)をしてくれと
メールが届きました。

絶対時間の構成を立証した基礎理論だそうです。万有引力定数を理論的に算出できるそうです。
テンソル解析・リーマン幾何は知らないそうです。聞き慣れない新語・造語がたくさん書いてあります。

どう対処したらよいでしょうか?アドバイスお願いします。。。
564秋月:04/04/17 01:19 ID:???
見なかった事にしよう!(ハハハ…)
565563:04/04/17 01:37 ID:???
了解しますた。
566ご冗談でしょう?名無しさん:04/04/17 12:10 ID:x6PuNNk8
炭化ケイ素の電子式ってどう書けばいいか教えて下さい。お願いします
567KingMathematician ◆5lHaaEvFNc :04/04/17 12:25 ID:+p1iJouJ
Re:>>566 炭素の原子番号は6,珪素の原子番号は14。これでよろしいか?
568KingMathematician ◆5lHaaEvFNc :04/04/17 12:27 ID:+p1iJouJ
Re:>>566 常識的に考えると、化学式はCSi_{4}だろう。
569KingMathematician ◆5lHaaEvFNc :04/04/17 12:27 ID:+p1iJouJ
Re:>>566 原子番号と化学式が分かれば、あとは習ったとおりにやればできる。
570566:04/04/17 13:38 ID:x6PuNNk8
>>569 Si::::Cでいいのですか?
571こんな感じ?:04/04/17 15:46 ID:???
>>548
ストークスの定理の証明
http://homepage2.nifty.com/eman/electromag/stokes.html
rot(E(r))=0が常に成り立つことの証明
http://homepage2.nifty.com/eman/electromag/static2.html
これの「エネルギー保存の条件」を参照

補足
エネルギー保存の条件より、
∫E・ds = 0 (閉曲線Cに沿って積分)
ここで左辺にストークスの定理を使うと、
∫E・ds = ∫rotE ・ n dS となる
∫E・ds = 0であるので、
∫rotE ・ n dS=0 (閉曲線Cに囲まれる曲面Sで積分)
静電場のどんなEに対しても上の式を満たすためには、
rotE=0でなければならない。
572KingMathematician ◆5lHaaEvFNc :04/04/17 17:13 ID:+p1iJouJ
よく考えたらCSi_{4}なんて結合は普通は出来ない。
サイト検索もしてみたが、SiCだった。
CとSiが共有結合して巨大な結晶が出来ると考えるのが自然だ。
それじゃあ、電子式はどのように書くべきなのか?
573ご冗談でしょう?名無しさん:04/04/17 18:31 ID:etSnoaix
ファインマンの量子力学と経路積分って本で
SchroedingerがSchrodingerになってる。
ご冗談でしょう、ファインマンさん。
574KingMathematician ◆5lHaaEvFNc :04/04/17 18:55 ID:+p1iJouJ
Schrödinger
575ご冗談でしょう?名無しさん:04/04/17 18:59 ID:???
>>573
oウムラウトの発音は確かにoeに近いが表記はoの方が良さげ。
576ご冗談でしょう?名無しさん:04/04/17 19:23 ID:???
oウムラウトがない場合はoeと表記するのが標準では?
577ご冗談でしょう?名無しさん:04/04/17 19:26 ID:???
576が正しい。
578ご冗談でしょう?名無しさん:04/04/17 19:32 ID:???
ウムラウトってもともとoの真上にeって書いてたのを
省略して二個の点で書くようにしたもんだろ。
579ご冗談でしょう?名無しさん:04/04/17 19:38 ID:2lRq4h1g
燃料電池は、電気を起こすのにエネルギーを使わないのですか?
580ご冗談でしょう?名無しさん:04/04/17 19:55 ID:???
何のための燃料ですか
581ご冗談でしょう?名無しさん:04/04/17 20:26 ID:???
 従来、独語のこのような特殊文字は、
それぞれの言い換え規則によって言い換えられた後の
綴り字で登録されていた。言い換え規則とは、
「"」の代わりに「"」が付くa、o、uの後にeを追加する方法だ。
例えば、株式を表すbo"rse(ここでは便宜的にウムラウトを
「o"」と表記)はboerseと言い換えられ、登録されている。
この言い換えは、歴史的には正しい言い換えなので、
通常の通信文などでの使用は問題がない。

http://internet.watch.impress.co.jp/cda/news/2003/09/25/544.html
582KingMathematician ◆5lHaaEvFNc :04/04/17 21:56 ID:AneetrTI
Re:>>581 htmlのプロに書いてもらえば良さそうなものだが、どうだろう?
583ご冗談でしょう?名無しさん:04/04/17 23:26 ID:x6dLIC1+
ここで良いのかどうか?質問させてください

ジョージ・ガモフの評価は2CHどんなの?
NHKスペシャルをみて、子供の頃に彼の本読んだのを思い出し気になってね。
又厨房がと思ったらスルーしてください。
584ご冗談でしょう?名無しさん:04/04/18 00:21 ID:???
KingMathematicianは数学スレに篭っていて下さい
585ご冗談でしょう?名無しさん:04/04/18 00:24 ID:oiF633hE
>>571
これってエネルギー保存則使わないと駄目なん?
数学的に言えば、この場合、閉曲線Cは空集合だから
ただちに∫E・ds = 0とやってはまずいのでしょうか
586こんな感じ?:04/04/18 01:26 ID:???
>>585
>閉曲線Cは空集合だから

スマソ、この質問の意味がよくわからない。
とりあえず任意にとった閉曲線C上の各点で、Eは値を持つよ。
587ご冗談でしょう?名無しさん:04/04/18 02:41 ID:HxrYLPvG
光よりも速い速度は、その可能性も含めてありえないのでしょうか?
それとも、ただ観測できないということなのでしょうか?
588ご冗談でしょう?名無しさん:04/04/18 03:05 ID:NV6swno2
>587 量子テレポート
検索キーワード
量子テレポート EPR アインシュタイン ボーア 論争
589ご冗談でしょう?名無しさん:04/04/18 04:15 ID:???
電磁気のビオ=サバール則は経験則なんでしょうか
590ご冗談でしょう?名無しさん:04/04/18 04:19 ID:???
物理法則は全て経験則
591ご冗談でしょう?名無しさん:04/04/18 08:02 ID:???
>>585
保存力じゃないと成り立たないでしょ
592ご冗談でしょう?名無しさん:04/04/18 08:37 ID:???
平坦な地面にボールを転がす場合、そのボールを止めようとする力の
最たるものは空気抵抗のような気がするんだけど、もし真空にしたら
ボールを止めようとする一番大きな要因は何になる?
あとは地面との摩擦くらいしか思いつかないけど、これは滑らない限り
ボールの回転にしか寄与しない気がするんだが。
質問です。
ある天体を周回する楕円軌道Aと、円軌道Bがあるとします。
Aをパーキング軌道として、一回目の増(減)速量を最小にしてBに遷移したい時、
その時の位置と、増(減)速度分を求める方法を教えて下さい。
よろしくお願いします。

…やっぱり、増速の場合、近地点から水平に打ち出しだったりしますか?
594ご冗談でしょう?名無しさん:04/04/18 10:32 ID:???
円軌道→円軌道じゃ遷移できねーだろ
595ご冗談でしょう?名無しさん:04/04/18 11:54 ID:???
>>592
世の中には転がり摩擦と言う物が(ry
596ご冗談でしょう?名無しさん:04/04/18 22:51 ID:38h1hONC
質問
自転車の車輪が回転することによって駆動する直流発電機があります。
この発電機に可変式抵抗をつなげます。
抵抗が大きい方が小さい方よりペダルが軽くなりました。
なぜですか?
597DQn:04/04/18 23:01 ID:???
>>596
マニュアルの自動車で
下り坂は何速?
上り坂は何速?
598ご冗談でしょう?名無しさん:04/04/18 23:11 ID:???
>>428
そういう風に作ったから
599ご冗談でしょう?名無しさん:04/04/18 23:22 ID:???
磁界と磁束密度の違いがよくわかりません
600ご冗談でしょう?名無しさん:04/04/19 00:30 ID:???
久々に600ゲトとかやってみるのもいいかも
601ご冗談でしょう?名無しさん:04/04/19 03:35 ID:???

ジョーカーを除いたトランプ52枚の中から1枚のカードを抜き出し、表を見ないで箱の中にしまった。
そして、残りのカードをよく切ってから3枚抜き出したところ、3枚ともダイアであった。

このとき、箱の中のカードがダイヤである確率はいくらか。
 あなたが、3つのドアの1つを選んで賞品を獲得するゲーム番組に出ているとします。
ドアの1つには賞品の車があります。残りの2つのドアはハズレでヤギがいます。
あなたがドアの1つ、たとえば1番ドアを選びます。
司会者は、それらのドアに何があるかを知っており、ヤギのドアを1つ、例えば3番ドアを開けてみせます。
そこで司会者があなたに、「2番ドアに変更してもいいですよ」といいました。2番ドアに変えたほうがいいのでしょうか。
602ご冗談でしょう?名無しさん:04/04/19 05:04 ID:???
>>599
>磁界と磁束密度の違いがよくわかりません

http://nippon.zaidan.info/seikabutsu/1996/00448/contents/022.htm
http://nippon.zaidan.info/seikabutsu/1996/00448/contents/023.htm
http://nippon.zaidan.info/seikabutsu/1996/00448/contents/024.htm
おおざっぱに言うと、磁力線の密度(磁束密度B)と、
磁荷をおいた時に磁荷が受ける力(磁界の強さH)の違い。
ちなみに磁束密度が大きければ、磁界の強さが強い。
電気力線と電界の関係と対応している。
603KingMathematician ◆5lHaaEvFNc :04/04/19 12:54 ID:sfYIX9Ug
ちょっとした疑問:
電界に誘電率をかけると電束(?)になりますが、
電界を電束密度と呼ぶことはないのですか?
604ご冗談でしょう?名無しさん:04/04/19 13:26 ID:???
検索したら一瞬でわかることをいちいち聞くなよ
605ご冗談でしょう?名無しさん:04/04/19 13:48 ID:???
>>603
間違っている。
電界に誘電率をかけると電束密度になる。
606ご冗談でしょう?名無しさん:04/04/19 15:38 ID:oNDV5tPg
質問です。
ヘリウムの分子量は4ですが、これは液体ヘリウムでも4でいいのでしょうか?
ここからの液体ヘリウムの質量計算は、
 8/(6.02^23)=1.33^-23[g]
であっていますか?
607ご冗談でしょう?名無しさん:04/04/19 15:50 ID:???
ヘリウムは単原子分子だから、分子量=原子量=4
質量は 4/(6.02×10^23)
608ご冗談でしょう?名無しさん:04/04/19 17:10 ID:hlH/ywOb
この穴うめ問題わかるかたいますか?写真写り悪いかもしれませんがお願いします。 http://a.pic.to/1j6x3
609ご冗談でしょう?名無しさん:04/04/19 17:16 ID:???
地球はなんで太陽の周りを回っているの?
610ご冗談でしょう?名無しさん:04/04/19 18:46 ID:uK5fN0db
完全な真空って作れるんですか?水銀でやればいいのかな・・・
611ご冗談でしょう?名無しさん:04/04/19 18:54 ID:???
>>610
水銀で真空作ろうとしても
水銀蒸気が入ってしまいます
612ご冗談でしょう?名無しさん:04/04/19 19:55 ID:???
>>610
絶対温度0度にすれば作れるよ。
613ご冗談でしょう?名無しさん:04/04/19 20:11 ID:lRq91NkP
スペインの物理学者でフアン・パロンドという人を知りませんか?
スペイン語か英語での綴りを教えていただきたいのですが。
614ご冗談でしょう?名無しさん:04/04/19 20:21 ID:gt+u3NJE
おしっこをしているとき震えるのは尿が出て膀胱の気圧が下がって体温が下がるからですか?
ヘアスプレーを噴射したときのように。
615ご冗談でしょう?名無しさん:04/04/19 21:06 ID:???
616ご冗談でしょう?名無しさん:04/04/19 22:12 ID:/9jEijxJ
ある物体の重量を低いところと高いとこで量ったら同じ値ですか?という質問で
答えは違うと思うんですが理由は何でしょうか?おしえて下さい
617ご冗談でしょう?名無しさん:04/04/19 22:23 ID:???
>>616
普通に重力が距離の2乗に反比例するからじゃないの?

>>614
俺も気になる。なんで?
618ご冗談でしょう?名無しさん:04/04/19 22:31 ID:???
>>614
気圧とか関係ないだろ。
単に尿が持ち去った熱の分だけ体温が下がる
619ご冗談でしょう?名無しさん:04/04/19 23:29 ID:???
尿の温度は体温とほぼ等しいから、尿を放出しても体温は下がらないだろう。
620ご冗談でしょう?名無しさん:04/04/19 23:56 ID:???
体内の温度は皮膚より1度ほど高いと聞いた事があります。
チンコが外部に飛び出ているのは37度を超えるような温度では精子の活性が
鈍るからだと聞いた事があります。
チンコの温度は体内温度より低い事が予想されます。
勃起時のように血液が活発に流れている時はチンコの温度が一時的に
高くなっているのかもしれませんが、平常時は指先のような状態になっているのでは
ないでしょうか。
そこを体内で十分に温められた尿が通過する事により一時的に温度が上がるでしょう。
普段は衣服の中で保温されていますが、放尿時には外気にもろに触れます。
よって放尿後ぶるっとくるのではないでしょうか。
621ご冗談でしょう?名無しさん:04/04/19 23:59 ID:???
>>614
震いってのは、体温の上昇効果を狙う自然現象
622ご冗談でしょう?名無しさん:04/04/20 00:00 ID:???
>>619
気化熱
実際には下がる前に震えが熱を作り出しているんだけど
623ご冗談でしょう?名無しさん:04/04/20 00:57 ID:ITk9t23Z
>>589
経験則ではありません。
624ご冗談でしょう?名無しさん:04/04/20 02:33 ID:???
>>622
気化するのは体外に出てからなので関係ないのでは?
625ご冗談でしょう?名無しさん:04/04/20 04:49 ID:D4akYs+R
>>510
うーん
まだいまいちよくわかんないです
もうすこしヒントください
626ご冗談でしょう?名無しさん:04/04/20 07:38 ID:FERJAGf+
よく人は、『物理的に無理!』という言葉を使いますが
どこら辺りから無理になってゆくのですか?
627ご冗談でしょう?名無しさん:04/04/20 08:01 ID:6Cive5tS
量子があるなら質子qualitumもあるはずだ
628IT土方:04/04/20 10:04 ID:BAZRhX3P
今日の朝日新聞2面のひと欄で、アンソニー・レゲットさんというノーベル賞
物理学賞受賞者が紹介されているのですが、この人が書いたという「科学英語
についてのノート」という英文9ページの手引きがあるそうですね。紙面によれ
ば日本の物理学者なら、だれでも1回は読んでいるそうですが、物理学とまったく
関係ない人が読んでみたい場合、どうするのが良いでしょうか?

物理学者はインターネットと係わり合いが深いので、てっきりwebで公開されて
いるかと思いましたが、googleでは見つかりませんでした。
なにかポインタになる事を教えていただければ幸いです。
629ご冗談でしょう?名無しさん:04/04/20 11:52 ID:???
日本物理学会誌21巻(1966)p760
ref.「理科系の作文技術」
630ご冗談でしょう?名無しさん:04/04/20 12:53 ID:???
>>624
だから、実際に体温が下がってしまう前に、
震えて熱を作り出しておくんじゃないの?
631ご冗談でしょう?名無しさん:04/04/20 13:21 ID:???
何がだからなのかわかんないですよ
632ご冗談でしょう?名無しさん:04/04/20 13:23 ID:???
>>624
体外に出た瞬間に一部蒸発して周りから熱を奪います
小便をする人の周りでアンモニア臭がするのは一部が蒸発しているからです



633ご冗談でしょう?名無しさん:04/04/20 14:47 ID:???
ということはその瞬間に外に向かって移動している液体を伝って
熱移動が起きるということですか
634ご冗談でしょう?名無しさん:04/04/20 15:15 ID:???
>>630
いや、そもそも体外に出てしまった尿から気化熱が奪われたからって
体温が下がるのかを疑問視してるんだけど

635ご冗談でしょう?名無しさん:04/04/20 15:20 ID:???
>>625
まだ悩んでたのか…というか悩みすぎ
単純にエネルギー保存則を使って解けばいいだけの問題だよ
(0,0)と(x1,0)でVが正じゃなくても、0<x<x1の区間でVが負ならば運動は起こりうるでしょ
636ご冗談でしょう?名無しさん:04/04/20 19:19 ID:???
>>633
そう
人間が近くにいて体温以下の物があれば熱移動が起きる
体温という言い方は違うか体表面から熱を奪う

>>634
系が断熱している状況なら圧力かけていた体温と同じ温度の水が気化すれば
間接的に人体からも熱を奪う
637ご冗談でしょう?名無しさん:04/04/20 19:29 ID:D4akYs+R
>>635
ポテンシャルのxにおける値が0になる点を探せばいいってことですか?


上に流れちゃったんで一応もう一回問題載せときます↓

2次元平面上を運動する質量mの質点を考える。平面に2次元直交座標
(x,y)を導入する。ポテンシャルエネルギーV(x,y)がA,Bを正の定数として
V(x,y)=-A(x^2+y^2)+b(x^2+y^2)^2であたえられるとき、質点をx軸上の点(x1,0)(x1>0)から
静かに話したところ、質点はx軸上で運動して原点(0,0)で速度が0になった。このことより
座標x1は(?)で与えられる

638637:04/04/20 19:31 ID:D4akYs+R
訂正
xにおける値が

(x,y)=(x1,0)における値が
639ご冗談でしょう?名無しさん:04/04/20 19:39 ID:???
あったかい尿が体内にあると、保温器として機能する。それが急になくなったら
もとの状態を保つために筋肉が運動して熱をつくりだすわけです。
640ご冗談でしょう?名無しさん:04/04/20 19:41 ID:???
尿の気化熱がどーのこーのとか言ってるヤツは勉強が足りない。
641ご冗談でしょう?名無しさん:04/04/20 20:02 ID:???
>>636
>人間が近くにいて体温以下の物があれば熱移動が起きる
出たばかりの尿は体表面より高温だってば。
気化で温度が体温より下がるより便器に到達するのが先ちゃうか?

>>639
同じ温度のものがなくなったからといって温度は下がりません。
温度的には元の状態のままです。
642ご冗談でしょう?名無しさん:04/04/20 20:17 ID:???
>>639
人体と尿とは絶えず熱交換をしていることは理解できますか?
643642:04/04/20 20:18 ID:???
間違えた
×>>639
>>641
644ご冗談でしょう?名無しさん:04/04/20 20:20 ID:???
639タンは膀胱内の尿は膀胱での体温より高い温度に保たれていると
思っているのだろうか
645ご冗談でしょう?名無しさん:04/04/20 20:24 ID:???
温度差にだけ思考が固着してる気の毒なヒトがいるみたいですが
そもそも熱とは何かがまったくわかっていない模様
646ご冗談でしょう?名無しさん:04/04/20 20:25 ID:???
>>642-643
だから熱平衡になって人体と尿は同じ温度になってるんでしょうが。
ここで尿が失われたからといって体内に残っている尿の温度が下がる
わけではない。ポットのお湯を注いだらポット内のお湯の温度は
下がりますか? 同様に、尿は体温と同じ温度のまま熱平衡状態。
647ご冗談でしょう?名無しさん:04/04/20 20:27 ID:???
ポットは筋肉で熱平衡を維持しているんですか?
648ご冗談でしょう?名無しさん:04/04/20 20:29 ID:???
>>647
排尿時の熱平衡を維持するのに筋肉はいらない
649ご冗談でしょう?名無しさん:04/04/20 20:31 ID:???
あーうぜーなーw

じゃ、こういいなおそう。
人体の比熱と尿の比熱は完全に等しいのか?
650ご冗談でしょう?名無しさん:04/04/20 20:34 ID:???
温度差0で比熱が関係あるのか?

体温調節ってのは熱量を維持すると勘違いしてませんか?
体温を一定に保つ働きですよ? 温度が変わらないときに
働かせる必要はありません。

651ご冗談でしょう?名無しさん:04/04/20 20:38 ID:???
>>641
液体だけじゃないでしょからだの表面の周りにあるのは
652ご冗談でしょう?名無しさん:04/04/20 20:38 ID:???
尿が体内にある時とない時で熱の流れが完全に一緒とあなたは言ってるわけですか?
653ご冗談でしょう?名無しさん:04/04/20 20:41 ID:???
>>639
尿の代わりに流れ込んでくる気体は温度が低いわけ?
654ご冗談でしょう?名無しさん:04/04/20 20:42 ID:???
尿の代わりに体内に気体が流れこんでくるって、あんたそれ病気だろw
655ご冗談でしょう?名無しさん:04/04/20 20:49 ID:???
>>654
あれ膀胱って絞って尿を出すんだっけ?
656ご冗談でしょう?名無しさん:04/04/20 20:49 ID:???
>>655
お前はもう寝ろ。
657ご冗談でしょう?名無しさん:04/04/20 20:53 ID:???
物理板はバカも歓迎だけど、バカのくせに自分が利口だと勘違いしてるヒトは歓迎しません。
658ご冗談でしょう?名無しさん:04/04/20 20:54 ID:???
>>657
今の流れだと誰の事を指しているのかよくわからん
659ご冗談でしょう?名無しさん:04/04/20 20:54 ID:???
>>658
わからんあんたは歓迎されてないみたいだよw
660658:04/04/20 21:02 ID:???
>>659
いや、俺は何もわからんバカだから
661ご冗談でしょう?名無しさん:04/04/20 21:08 ID:???
あったかいおしっこがからだのなかにあれば、きんにくがあんまりがんばらなくても
おしっことからだのぜんたいのおんどをいっていにたもつことができます。
おしっこがねつをためておけるからですね。
でも、おしっこがからだのそとにでていってしまうと、ねつをためておくものがなくなってしまうので
きんにくががんばってねつをつくりださないと、おしっこがあったときのようなあったかさを
たもてなくなってしまいます。これはわかりますか?
662ご冗談でしょう?名無しさん:04/04/20 21:11 ID:???
>>652
尿を出してる最中(あるいは直後)にブルっと来るほど差があると
あなたは言ってるわけですか?
663ご冗談でしょう?名無しさん:04/04/20 21:13 ID:???
>>661
これだけ言っても熱と温度の区別が付いてないバカがいるらしい。退散するよ
664ご冗談でしょう?名無しさん:04/04/20 21:18 ID:???
>>663
きんにくは、おしっこがあるときとおなじようにやすんでいると
おしっこがなくなったときにおんどをたもつことができません。
ねつとおんどはちゃんとくべつしてかんがえましょう。
665ご冗談でしょう?名無しさん:04/04/20 21:26 ID:???
体温が外界の温度より高いということを忘れてる間抜けがいるみたいだな。
666ご冗談でしょう?名無しさん:04/04/20 21:27 ID:???
あ、退散したのか。
667KingMathematician ◆5lHaaEvFNc :04/04/20 22:10 ID:nDKFX0IT
生理現象にも物理学で考察する余地があるということか。
それじゃあ、人は時間が経つと眠くなるのは何故?
668ご冗談でしょう?名無しさん:04/04/20 22:20 ID:zPSJLVop
漠然とした質問なのですが、
私がいま住んでいる賃貸マンションには、各部屋の天井に
火災報知器が取付けられています。(白い丸型のものです)
この中にはアメリシウムという元素が入っているそうです。

で、この報知器、前に住んでいた人が破損したらしく、押し入れに
取付けられた一つが、カバーが取れてしまっていて、中の回路(?)が
露出していることに気付きました。
放射性元素だということで、なんか急に不安を感じてきたのですが、
カバーが取れて中身が露出してる場合は被曝量がめちゃめちゃ
高くなる、なんてことがあるのでしょうか。
それとも、そんなに心配することでもないんでしょうか。
669ご冗談でしょう?名無しさん:04/04/20 22:28 ID:???
プラスチックカバーの場合、しようがしまいがあまり変わらないと思うがどうか。
670IT土方:04/04/20 23:36 ID:???
>629
ありがとうございます。図書館で探して見ます。
#理科系の作文技術も持っていたはずだが...どこいっちゃったかな。
671ご冗談でしょう?名無しさん:04/04/21 00:22 ID:???
力のモーメントのところで
「棒を回転させる効率は回転軸からの距離に比例」するとありますが
これは経験則なのでしょうか
672ご冗談でしょう?名無しさん:04/04/21 00:26 ID:???
違います
673ご冗談でしょう?名無しさん:04/04/21 00:27 ID:???
説明キボンヌ
674ご冗談でしょう?名無しさん:04/04/21 01:02 ID:???
角運動量が保存するから。
力学の教科書の剛体の章に書いてあるはずだから読んでみ。
675ご冗談でしょう?名無しさん:04/04/21 01:02 ID:???
放尿時における身震い 学説相関関係図

─┬体温低下派─┬→膀胱の気圧が下がる派(気圧低下説)
  │         │
  │         ├→尿が熱を持ち去る派→尿が熱を保持する役割を
  │         │     (熱量減少説)      するよ派(熱保持説)
  │         ├→アンモニアの気化熱派→震えによって筋肉を
  │         │     (気化熱説)     運動させ熱をつくるよ派(熱生成説)
  │         ├→尿を伝わって体内の熱が外に逃げるよ派(尿媒介熱拡散説)
  │         ├→人体の比熱と尿の比熱は違うよ派(比熱要因説)
  │         ├→尿の有無で体内の熱の流れは違うよ派(熱流変化説)
  │         └→尿の代わりに体内に外気が入るからだ派(外気進入説)
  │
  ├体温は同じだよ派┬→尿の温度は体温と同じだよ&
  │            │  温度差0で比熱は無関係派(温度変化否定説)
  │            ├→体外に出た尿の気化熱は無関係派(気化熱説否定)
  │            └→排尿時の熱平衡維持に筋肉は不要派(筋肉熱生成否定)
  │
  ├チンポの一時的な温度変動が原因だよ派(肉棒主因説)
  │
  └物理学で説明は無理派→生理現象にも物理学で考察する余地があのか派
                       (数学原理主義派)
676ご冗談でしょう?名無しさん:04/04/21 04:09 ID:???
日本だとほとんどの場合で気温は体温より低い。
つまり人体は常に外気に冷やされ続けている。
排尿すると人体の熱容量が減る。つまり冷えやすくなる。
比熱の差はあまり関係無い。


>>668
へー、超ウラン元素が日常に使われているのか。知らんかった。

>>671
経験則です。物理学の法則は全て経験則です。
677ご冗談でしょう?名無しさん:04/04/21 04:57 ID:???
プランク定数の定義とは何でしょうか
678ご冗談でしょう?名無しさん:04/04/21 05:11 ID:???
>>677
ミクロの世界における作用量子
679ご冗談でしょう?名無しさん:04/04/21 05:15 ID:???
>>678
意味がわからんがな
680ご冗談でしょう?名無しさん:04/04/21 06:41 ID:???
>>679
それはお前が莫迦だから
681ご冗談でしょう?名無しさん:04/04/21 07:52 ID:???
>>675
外気進入説が一番好きw
682ご冗談でしょう?名無しさん:04/04/21 10:09 ID:???
>>675
俺が新説を出す。よければ、追加してくれ。

膀胱が尿に満たされた状態でその尿を排泄すると、
膀胱の内圧の急激な低下により、断熱冷却に似た
現象が発生し、尿の温度が低下、体から熱を奪う。
683ご冗談でしょう?名無しさん:04/04/21 10:15 ID:???
>>682
定量的に分析して下さい。
684ご冗談でしょう?名無しさん:04/04/21 10:16 ID:???
>>682を改良した。

膀胱の内圧の急激な低下までは同じ。ただし、次に発生するのは
断熱冷却でなく、圧力の低下によるアンモニア等の気化である。
アンモニア等の気化は膀胱内でも発生するので、尿から気化熱を
奪い、さらに体から熱を奪う。
685ご冗談でしょう?名無しさん:04/04/21 10:35 ID:???
膀胱内にアンモニア蒸気がたまるんですか?
686ご冗談でしょう?名無しさん:04/04/21 10:59 ID:???
俺が新説を出す。

排尿後の震えは、排尿の最終段階で尿道口に残った尿を振り払うための不随意運動である。
それが証拠に、体温より高い温度の風呂に浸かりながら放尿しても(アホか俺は)ブルっとくる。
よって、あの震えは熱に関連したものではない。
687686:04/04/21 11:08 ID:???
いや、少なくとも温度低下をトリガーに誘発される震えではない。
688ご冗談でしょう?名無しさん:04/04/21 11:45 ID:???
>>685
殆どは尿とともに排出される。

>>686
不随意運動なのは間違いない。実際に熱が奪われるか否かに関わらず、
排尿後は体温低下に備えて必ずブルっとするようになってるんだろう。
689ご冗談でしょう?名無しさん:04/04/21 13:21 ID:???
>>682>>684
そういうこと言うなら、膀胱周りの筋肉の緊張が解けて
温度低下とかのほうがよくないだろうか
690ご冗談でしょう?名無しさん:04/04/21 14:34 ID:???
arxivってなんてよむの?
691ご冗談でしょう?名無しさん:04/04/21 14:38 ID:???
>>690
あるくしーぶ
692ご冗談でしょう?名無しさん:04/04/21 19:22 ID:???
今朝の排尿はブルッと来なかったぞ。

>>686
>体温より高い温度の風呂に浸かりながら放尿しても(アホか俺は)ブルっとくる。
本当か?検証例を増やす必要があるな。
693ご冗談でしょう?名無しさん:04/04/21 19:43 ID:???
浴槽内での排尿に付随する痙攣は背徳感によるものではないだろうか?w
694ご冗談でしょう?名無しさん:04/04/21 19:48 ID:???
浴槽内での射精に付随する痙攣は背徳感によるものにまちがいないと思う
695ご冗談でしょう?名無しさん:04/04/21 19:52 ID:???
www.asahi-net.or.jp/~dz2s-ymuc/html1005.htm

おおお!
浴槽内での排尿に付随する痙攣は幼児にも見られる現象らしい。
彼らが背徳感を持つことは考えにくいな。
696ご冗談でしょう?名無しさん:04/04/21 20:04 ID:ZrQ8mxnc
>>676 みたいな理屈だと、排尿後ずっとガクブルしてないと
体温が維持できないってことになるんじゃない?

温度低下が理由なら、例えば大ジョッキ一気飲みしたときなんかの方が
温度低下が大きそうだ。
けど、そういうときにはブルっとはしないよね。
697ご冗談でしょう?名無しさん:04/04/21 20:43 ID:???
>>676
>経験則です。物理学の法則は全て経験則です。

字面を厳密に解釈すればそうなるのは当然だが、
実際に聞きたいのは類似の状況固有の現象論か
確立された基本法則から一般的に言えることかということなのでは?
698ご冗談でしょう?名無しさん:04/04/21 21:54 ID:???
>>690
つるぺた
699ご冗談でしょう?名無しさん:04/04/21 23:38 ID:???
ウンコをしても震えないね
700ご冗談でしょう?名無しさん:04/04/21 23:56 ID:ZrQ8mxnc
そういえば、すげー熱い風呂に入ったときにもぶるっとくることがあるな。
おれだけ?
701ご冗談でしょう?名無しさん:04/04/22 00:19 ID:???
いや、それは確かにある。
702ご冗談でしょう?名無しさん:04/04/22 04:02 ID:UmxCfqWx
1/Lの次元をもつ空間を定義し、応用例を一つあげて説明せよ。
という問題なのですが、わかる方おられませんか?
703ご冗談でしょう?名無しさん:04/04/22 04:20 ID:???
地球の真ん中に、半径2メートルくらいの丸い部屋を作って
人間が生きられる環境にして、人を入れると、どういう風に重力がかかるんですか?
壁のほうに行くと、壁からちょっとだけ引っ張られる感じがして
中心に行くと無重力状態になるんですか?
中心に浮いてると、先っぽのほうがむくんだりするんですか?
704ご冗談でしょう?名無しさん:04/04/22 04:22 ID:???
真ん中というのが重心という意味なら、その部屋は無重力空間になるはず。
705ご冗談でしょう?名無しさん:04/04/22 04:27 ID:???
>>704
なるほど、ありがとう
706ご冗談でしょう?名無しさん:04/04/22 06:52 ID:???
北極から南極に向かって直径1kmぐらいの穴を空けたとして、
その穴のど真ん中を人がフリーホールのように落ちていくと、
地球の裏側にたどり着く事は出来るの?
それとも、地球の中心を過ぎてからは、逆向きに重力が働く
ためにブレーキがかかって、結局、中心付近で無重力状態の
ために浮いている状態となってしまうの?
なお、地中のマントルなどの熱の影響は無く、穴の外壁も崩れ
ないと仮定します。コリオリ力の影響も無しです。
707KingMathematician ◆5lHaaEvFNc :04/04/22 07:57 ID:wuu4wMOy
Re:>>702 数学にはフラクタル次元というものがある。
708KingMathematician ◆5lHaaEvFNc :04/04/22 07:58 ID:wuu4wMOy
Re:>>706 振り子のように反対側にいけるだろう。
709ご冗談でしょう?名無しさん:04/04/22 08:34 ID:izXErLqk
>>708
お前馬鹿?
空気抵抗考えたら、落下速度は300キロ前後で頭打ちになる。
これじゃあ、反対側になど到底いけない。
せいぜい、中心から10分の1程度の距離で勢いが無くなり、
中心に向かって逆戻りし、中心付近で漂うのがオチだ。
710ご冗談でしょう?名無しさん:04/04/22 10:36 ID:???
終端速度が300km/h程度なのは1気圧下だな。
穴を塞いで真空にしないのなら、中心付近は超高圧(かつ高温)だね。
中心の遥か以前でほとんど速度を失ってゆっくり沈むようになり、中心にも辿り着けないだろう。
711ご冗談でしょう?名無しさん:04/04/22 20:41 ID:f3r7DQBe
こいのぼり見て思ったんだけど、
風を一切抜かさない布で、
口がはんけい30CMで1Mあたり500gのこいのぼりを作ったとき
長さが100KMのこいのぼりを、
浮かせつづけるためにはどのくらいの風を送ればいいのですか?
712ご冗談でしょう?名無しさん:04/04/22 20:53 ID:???
>>710
空気の上でおひるねできますか?
713ご冗談でしょう?名無しさん:04/04/22 21:42 ID:???
>>712
充分濃い空気の上ならできるよ
耐圧服が要るけどね
714ご冗談でしょう?名無しさん:04/04/22 22:11 ID:???
穴の側面に絶対ぶつからないとかそんな感じの条件が入っていれば
中心にたどり着けないというのはちょっと考えにくいが、
まあかなりきつい減衰振動だろうな。
715ご冗談でしょう?名無しさん:04/04/22 22:21 ID:???
>>714
>中心にたどり着けないというのはちょっと考えにくい
浮力とつりあうところで止まる
716715:04/04/22 22:25 ID:???
…と思ったけど、浮力は密度×重力に比例するから(体積は変わらないとして)、
中心付近で密度が高くなるぶんと重力が小さくなるぶんのどっちが勝つか、
すぐにはわからんな。重力が小さくなるほうが勝ると、中心までつりあわない
717ご冗談でしょう?名無しさん:04/04/22 22:48 ID:???
>>706
地球の空気がどんどんその穴に吸い込まれるって聞いたことあるよ。
中心部は10マソ気圧くらいになるとか。
718ご冗談でしょう?名無しさん:04/04/22 23:09 ID:TjO+PXsg
階段を昇って、一番上まで行って、その後降りてきた時、位置エネルギーの変化はないのに、 
何故疲れるんですか?疲れるってことはエネルギー消耗してるんですよね?
719ご冗談でしょう?名無しさん:04/04/22 23:14 ID:???
力学的エネルギーだけがエネルギーではない
720ご冗談でしょう?名無しさん:04/04/22 23:19 ID:???
>>718
とりあえず、最初の状態から階段上って一番上まで行くのに
エネルギーを使ってるから。
721ご冗談でしょう?名無しさん:04/04/22 23:50 ID:???
v方向のベクトルをB方向に ねじを右にまわすようにまわす時にねじが進む方に力がかかるってのは 
右ねじの法則とは全く関係ないものなんですか?
722ご冗談でしょう?名無しさん:04/04/23 00:32 ID:???
>>716
落下物もべきべきに押しつぶされて密度は増すのだ
723ご冗談でしょう?名無しさん:04/04/23 01:07 ID:JGaHerZV
太さ・強度がともに均一なヒモを片方を固定してもう片方から引っ張ると
どこで切れますか?理由も含めて教えてください。お願いします!
724ご冗談でしょう?名無しさん:04/04/23 01:14 ID:???
>>723
 本当に均一なら切れない。伸びるだけ。なぜなら切れるという特異な場所が存在しないから。
 実際には微小なほころびとかで応力が集中して切れるわけだが。
725ご冗談でしょう?名無しさん:04/04/23 01:44 ID:JGaHerZV
>>724
伸びることによって真ん中が細くなって真ん中から切れるということは
ないんですか?
726ご冗談でしょう?名無しさん:04/04/23 01:53 ID:???
 均一に伸びるから真ん中だけ細くなるなんてことはない。むしろ固定or引っ張っている端のとこに
応力集中することが考えられるな。
727ご冗談でしょう?名無しさん:04/04/23 02:10 ID:hFDDBxib
コイルのエネルギーが1/2*L*I^2になるのは何故ですか?
728KingMathematician ◆5lHaaEvFNc :04/04/23 07:59 ID:Hn4tD5XV
Re:>>727 それぐらいどこかで調べてくれ。吾は高校で習ったはずのLについて今から調べてくる。
729ご冗談でしょう?名無しさん:04/04/23 11:48 ID:???
>>727
∫VdI=∫LIdI
730ご冗談でしょう?名無しさん:04/04/23 13:30 ID:???
>>724
どこか特定はできないけど切れるでしょ
円錐の頂点にボール置いたら、どの方向かは特定できないけど
どちらかの方向には転がるのと一緒
自発的対称性の破れって奴だ
731ご冗談でしょう?名無しさん:04/04/23 15:42 ID:???
ラゲール多項式やエルミート多項式は、それぞれ
ラゲール微分方程式 xe^(-x)y'' + (1-x)e^(-x)y' + ne^(-x)y = 0 や
エルミート微分方程式 e^(-x^2)y'' + 2xe^(-x^2)y' + 2ne^(-x^2)y = 0 の解関数ですよね。
そもそもの微分方程式が2階微分方程式なのだから
その解関数には積分定数が2つ要りそうなものなのに、
ラゲール多項式やエルミート多項式の定義式に積分定数が入っていないのはどうしてですか?
732KingMathematician ◆5lHaaEvFNc :04/04/23 17:03 ID:Hi0v44zv
Re:>>729 果たしてどこから突っ込むべきなのか?
733KingMathematician ◆5lHaaEvFNc :04/04/23 17:06 ID:Hi0v44zv
Re:>>731 ラゲール多項式、エルミート多項式は、ともに方程式を満たすだけであって、
ラゲール多項式、エルミート多項式自身の定義は別にある。
734KingMathematician ◆5lHaaEvFNc :04/04/23 17:07 ID:Hi0v44zv
Re:>>729 やはりIで積分するとはどういうことか、という部分を先に突っ込むのが正しいか。
735ご冗談でしょう?名無しさん:04/04/23 17:37 ID:???
>>733
特解ってことですか?
すると、一般解はどうやって表すのでしょう?
ラゲール多項式を2つ持ってきて積分定数掛けて足したらラゲール微分方程式の一般解になりますか?
736729:04/04/23 17:41 ID:???
ちょっと勘違いしていた。
V=L(dI/dt)だから
IV=LI(dI/dt)=d(1/2*L*I^2)/dtだな。
時間で積分すれば1/2*L*I^2になる。
737729:04/04/23 17:51 ID:???
∫IVdt=∫LI(dI/dt)dt=∫LIdI
としたほうがわかりやすいかな。
738KingMathematician ◆5lHaaEvFNc :04/04/23 17:58 ID:Hi0v44zv
Re:>>735 そうだろう。線型常微分方程式の一般論からいうと、
二階の場合は線型独立な二つの解を使って一般解を求められる。
(判別式が0になる場合は一つの解に(C1x+C2)を掛けることになる。)
739ご冗談でしょう?名無しさん:04/04/23 18:14 ID:???
Re:>>739 果たしてどこから突っ込むべきなのか?

740ご冗談でしょう?名無しさん:04/04/23 19:02 ID:???
>>738
>二階の場合は線型独立な二つの解を使って一般解を求められる。
>(判別式が0になる場合は一つの解に(C1x+C2)を掛けることになる。)

それは*定数係数*2階微分方程式の一般解では。
741ご冗談でしょう?名無しさん:04/04/23 19:02 ID:OMCkdbE4
キッテルの固体物理学入門を勉強した後、ナノテクをもっと勉強するには
どんなテキストブックを使えばいいですか?
742ご冗談でしょう?名無しさん:04/04/23 19:18 ID:???
バネに重りを吊して上下に振動させ、
重りの重量mとバネの周期Tからk=4mπ^2/T^2という計算でバネ定数を算出できますが、
実際にバネ定数を計測するとき、振幅は大きい場合と小さい場合とではどちらがいいのですか?
サインカーブがどうこうという話を耳にしたんですが。
743ご冗談でしょう?名無しさん:04/04/23 19:22 ID:???
>>742
振幅が大きすぎるとフックの法則からずれる可能性がある。
逆に小さすぎると周期を正確に測定するのが難しくなる。
その兼ね合い

744ご冗談でしょう?名無しさん:04/04/23 19:50 ID:???
>>739
そうだな、まずはキミのアンカーの張り方からかな。
745739:04/04/23 19:58 ID:???
>744
テヘ
746KingMathematician ◆5lHaaEvFNc :04/04/23 21:15 ID:Hi0v44zv
Re:>>740
f(x)y''(x)+g(x)y'(x)+h(x)y(x)=0を一階化すると、
(y(x),(y'(x))'=((0,1),(-h(x)/f(x),-g(x)/f(x)))(y(x),y'(x))=A(y(x),y'(x))
これの解は(y(x),y'(x))=exp(Ax)(C1,C2)でいいんじゃないの?
747sage:04/04/23 23:23 ID:mw24TKLw
>>741
CELLでもよめば?
748ご冗談でしょう?名無しさん:04/04/24 01:48 ID:fvqucqQF
測定するしかないと思っていた屈折率を
マクスウェル方程式を使うことによって計算できると聞きました。
正直マクスウェル方程式をあまり理解してないのですが、
どうやって屈折率を計算するのかの流れだけでも教えてもらえないでしょうか?
749ご冗談でしょう?名無しさん:04/04/24 05:58 ID:???
屈折率とは速度の比です。
速度は誘電率と透磁率で決まります。(ここにマクスウェル方程式が出てくる)
何も測定しないというわけにはいきません。
750KingMathematician ◆5lHaaEvFNc :04/04/24 07:04 ID:hAjFYuby
これは奇跡なのか?[>>746]が誰にも突っ込まれてない。
(y(x),y'(x))=exp(∫Adx)(C1,C2)だというのに。
(まぁ、Aが定数ならAxでもいいのだが。)
751ご冗談でしょう?名無しさん:04/04/24 07:06 ID:???
つーか誰も読んでねーし
752ご冗談でしょう?名無しさん:04/04/24 07:42 ID:???
ウザがられてるのに気づかない香具師程困ったものもないな
753ご冗談でしょう?名無しさん:04/04/24 09:13 ID:???
「grad〜」ってありますよね?
この「grad」ってどういう意味なんですか?
754ご冗談でしょう?名無しさん:04/04/24 09:29 ID:UkY2z6Yj
グラディエーター古代ローマ帝国時代の大衆への見世物として、巨大コロシアムで人間同士又は猛獣を相手に死ぬまで戦いを強いられた剣闘士のこと。
755ご冗談でしょう?名無しさん:04/04/24 10:55 ID:???
I'm grad to see you.

私はあなたに会えてナナメってます。
756ご冗談でしょう?名無しさん:04/04/24 13:37 ID:???
ある日、私は森に迷ってしまった。
夜になりお腹も減ってきた。
そんな中、一軒のお店を見つけた。
「ここはとあるレストラン」
変な名前の店だ。
私は人気メニューの「ナポリタン」を注文する。
数分後、ナポリタンがくる。私は食べる。
・・・なんか変だ。しょっぱい。変にしょっぱい。頭が痛い。
私は苦情を言った。
店長:「すいません作り直します。御代も結構です。」
数分後、ナポリタンがくる。私は食べる。今度は平気みたいだ。
私は店をでる。
しばらくして、私は気づいてしまった・・・
ここはとあるレストラン・・・
人気メニューは・・・ナポリタン・・・

757ご冗談でしょう?名無しさん:04/04/24 13:40 ID:???
grad ≡ ∇
gradient:勾配
758ご冗談でしょう?名無しさん:04/04/24 14:35 ID:???
>756
わからん、どーゆーこと?
759ご冗談でしょう?名無しさん:04/04/24 19:27 ID:???
二色の同型・同質量のチョコボールそれぞれ50個程度をボウルに入れ、なるべく少ない労力で
均一に混ざるようにするにはどのように混ぜるのが一番よいか、教えてください。
760ご冗談でしょう?名無しさん:04/04/25 01:18 ID:???
つかまる
761ご冗談でしょう?名無しさん:04/04/25 01:34 ID:???
>>759
ボウルを加熱する
762ご冗談でしょう?名無しさん:04/04/25 10:34 ID:???
今年2年生になったのですが
微分積分を数学科の教科書を買って
厳密に論理を追いかけていくことには
どの程度物理には役に立つでしょうか
ちょっとやってみようと思っています
763ご冗談でしょう?名無しさん:04/04/25 13:34 ID:XuW0cEcU
>>754
映画のせいで英語読みが定着してしまったが、ラテン語なのだから、
グラディアトールと読んで欲しいね。
764ご冗談でしょう?名無しさん:04/04/25 14:04 ID:???
>>762 どの程度かはわからんが、基本的な微積の理解に曖昧な点があるようでは何も出来ない。
765ご冗談でしょう?名無しさん:04/04/25 14:08 ID:???
>>762 ほとんど役に立たない
766ご冗談でしょう?名無しさん:04/04/25 14:17 ID:???
>>762
まったく役に立たんと思うが、
そんな酔狂なことをやってる暇は
この先ないだろうから、やるなら今だ。
767ご冗談でしょう?名無しさん:04/04/25 15:46 ID:eVj0FKLg
前に、空気は何で見えないか、を議論してたスレがあったと思うんだけど、誰か知らない?
結構面白かったからもう一度見たいんだけど。
768ご冗談でしょう?名無しさん:04/04/25 16:35 ID:???
空気で検索すりゃええがな
769ご冗談でしょう?名無しさん:04/04/25 18:47 ID:b42VBtmo
太陽の重力加速度っていくつですか?
770ご冗談でしょう?名無しさん:04/04/25 19:18 ID:???
771ご冗談でしょう?名無しさん:04/04/25 19:21 ID:???
>>769
地球の公転半径:1億5000万km
公転周期:1年

さあーどうなる?
772質問:04/04/26 17:01 ID:7SputznN
宇宙が膨張していると仮定するとして
光の何%かの速度で この地球から遠ざかっている
星か銀河があることになるが

その遠ざかっている星や銀河の時間は
我々の時間より遅く進んでいるのでしょうか
はたまた、それらの星や銀河から我々を見れば
我々が遠ざかっていることになるのだから
その星や銀河から見れば時間が遅く進んでいることになるのでしょうか

773ご冗談でしょう?名無しさん:04/04/26 18:01 ID:???
両方正しいですけど。
774ご冗談でしょう?名無しさん:04/04/26 18:23 ID:26/FHRga
dr/d=√[2/m*{E+mMG/r}-1/(m**2*r**2)]/{n/(m*r**2)}を唐ナ積分する
って箇所で詰まってるんですが、わかります?
775ご冗談でしょう?名無しさん:04/04/26 18:25 ID:???
>>774
わかんねえよ
もう一回問題見直せ
776ご冗談でしょう?名無しさん:04/04/26 18:27 ID:9QNo33OW
あほな質問で申し訳無いんですけど
二酸化炭素って有機物ですか?
777ご冗談でしょう?名無しさん:04/04/26 18:54 ID:???
>>772
結論から言えば、「遅くない」。
宇宙の膨張は重力の効果であり、重力場では特殊相対性理論は局所的にしか成立しない。
遠ざかっている星や銀河のような大局的な範囲に、時間が遅れるといった特殊相対性理論の
結論を単純に適用することはできない。
778ご冗談でしょう?名無しさん:04/04/26 19:02 ID:???
>>776
有機物でぐぐればすぐわかる
779ご冗談でしょう?名無しさん:04/04/26 19:38 ID:26/FHRga
>775
すんません。
今見直してます。おかしいですね…
780ご冗談でしょう?名無しさん:04/04/26 20:56 ID:9QNo33OW
>>778
分かりました。どうも有難う御座います。

もう一つ。

どうして宇宙は膨張しているのでしょうか?
ダークエネルギーの斥力??
781ご冗談でしょう?名無しさん:04/04/26 21:02 ID:DehuYANw
782ご冗談でしょう?名無しさん:04/04/26 21:36 ID:IiD9DDxJ
ラグランジュ方程式で、拘束条件付のとき、λの求め方が分かりません。
783ご冗談でしょう?名無しさん:04/04/26 21:55 ID:???
>>782
 拘束条件を使って変数の数を減らせ。
784ご冗談でしょう?名無しさん:04/04/26 21:57 ID:26/FHRga
dr/d=√[2/m*{E+mMG/r}-1/(m^2*r^2)]/{n/(m*r^2)}を=n/m*r^2で積分する。

785ご冗談でしょう?名無しさん:04/04/26 21:59 ID:26/FHRga
>>784
失敗です…すいません。
あとはただの計算なんですけど…どうすればいいですか?
786ご冗談でしょう?名無しさん:04/04/26 22:05 ID:wNe91ABS
宇宙全部が巨大なブラックホールの中にあるって考えは、無理がありますか?
787限界:04/04/26 22:24 ID:V2tp+q8c
典型的な有機シンチレーター中では100eVで一個の光子が発生する
と公には言われていますが
導出の仕方を教えていただけないでしょうか
788ご冗談でしょう?名無しさん:04/04/26 22:42 ID:YrcqUeka
トリチェリーの水銀柱の実験で
柱の根元だけを太くしたら
水銀は760mm以上高く上がるんでしょうかね?

__
| |
| |
| |
| |
_| |_
| |   

↑こんなん(ワロタ
789ご冗談でしょう?名無しさん:04/04/26 23:03 ID:uzaDC7xY
>>788
割れてるよ!
790ご冗談でしょう?名無しさん:04/04/26 23:08 ID:???
>>787
経験値じゃだめ?

新しい高発光量シンチレータの素材探しをするなら別だけど。
NaI(Tl)より高発光率のプラスチック見付けたら神扱いされるかも。
少なくとも漏れはすげー助かる
791ご冗談でしょう?名無しさん:04/04/26 23:22 ID:???
>>777
なんで一般相対論的な時間の遅れを考えちゃダメなんですか?
792ご冗談でしょう?名無しさん:04/04/27 00:43 ID:GKv4euWP
すみません全くの素人です。

ある人に「時間は何故12進法?時計はなぜ12時まであるの?」と訊かれて、

知らなかったので
『円を12等分するためかなぁ』と答えたのですが…

どなたかご存知の方が居られたら、教えてください。

793ご冗談でしょう?名無しさん:04/04/27 01:49 ID:???
>>792
そこそこ大きくて、いろんな数で割り切れて便利だから
794教えてください:04/04/27 03:53 ID:akGaMzTy
陽子間および陽子→電子間のクーロン力と重力(万有引力)の
比を求めたいのですがどうしたらよいでしょうか
丁寧に教えていただけますと本当に助かります
よろしくおねがいします!!
795教えてください:04/04/27 03:54 ID:akGaMzTy
つけたしです
クーロン力と重力(万有引力)の比です
796ご冗談でしょう?名無しさん:04/04/27 04:03 ID:???
クーロン力と重力をそれぞれ求めて、比をとればよい。
797ご冗談でしょう?名無しさん:04/04/27 04:15 ID:akGaMzTy
その求め方から教えていただけませんでしょうか・・・
すいません本当に、お手数おかけして
798ご冗談でしょう?名無しさん:04/04/27 04:17 ID:???
クーロン力と重力の公式は?
799ご冗談でしょう?名無しさん:04/04/27 04:20 ID:akGaMzTy
F = k ・q1・q2 / r2 [N]
K=9.0×109 [N・m2/C2]

ですか??
800ご冗談でしょう?名無しさん:04/04/27 04:22 ID:???
ニュートンの万有引力(重力)の公式は?
801ご冗談でしょう?名無しさん:04/04/27 04:23 ID:akGaMzTy
F=GMm/r2乗
ですか??
802ご冗談でしょう?名無しさん:04/04/27 04:25 ID:???
あとは割るだけ。おしまい。
803ご冗談でしょう?名無しさん:04/04/27 04:26 ID:akGaMzTy
えぇぇ!!
どう割るんですか!!
なにをわるんですか??
アホな私ごめんなさい。。。
804ご冗談でしょう?名無しさん:04/04/27 04:27 ID:???
F/F
805ご冗談でしょう?名無しさん:04/04/27 04:28 ID:akGaMzTy
よくわからないです。。
陽子間と陽子→電子間
両方出るのでしょうか?
806ご冗談でしょう?名無しさん:04/04/27 04:31 ID:akGaMzTy
教えてくださってる方ほんとにごめんなさい
お手数おかけしてすいません・・
807ご冗談でしょう?名無しさん:04/04/27 04:37 ID:???
必要な数値を入れれば両方でる。
808ご冗談でしょう?名無しさん:04/04/27 05:02 ID:???
物理以前の問題だろ
比の意味分かってたらナンも迷うことないぞ
809ご冗談でしょう?名無しさん:04/04/27 06:05 ID:???
>>808
小学5年生ががんばってニュートン力学を学ぼうとしてるんだよ。
生暖かく見守ってやろう。
810787:04/04/27 07:15 ID:Gwor7LyR
>790
実験値ならその値が知りたいです
溶媒が放出するエネルギーは吸収した内の役3%であるとは書いてありますが
そこから蛍光物質を二種類使って紫の光子になるんですよね
最初に容疑いが吸収したエネルギー100eVがどのように一光子になるか
数式ではどうなるのでしょう?
811ご冗談でしょう?名無しさん:04/04/27 11:29 ID:???
>>810
え、100eV/photon自体が実験から得られた値、という意味だったのですが。

どうも何かの宿題っぽいな。紫の光子1個のエネルギーはいくつですか?
それが吸収エネルギーの3%に当たるとすれば、紫の光子1個作るのに
必要なエネルギーは?
812ご冗談でしょう?名無しさん:04/04/27 11:58 ID:???
>>786
そう考えることによって、従来説では説明できなかった何かが
説明できるならばいいが、そうでないなら無意味だから止めれ。

>>791
考えてもいいが、殆ど差は無いよ。

>>792
最初に暦を作った古代バビロニア人が12進法を使ってたから。
813787:04/04/27 12:43 ID:5hiHsEsr
今さっき気がつきました
最終的に出て来るのが400nmの光子とすると一個辺り3.1eVですね

溶媒から出る光は、さらに溶質で蛍光により波長をずらすのですよね
ならさらにエネルギーは減少するのではないかとずっと考えてましたが
溶媒>溶質の伝達率が〜100%なんてことは、、、
単に3%と言うのは全て経た値だったのでしょうか
もう一度「フィジカル レビュー2002」読み直してみます
814ご冗談でしょう?名無しさん:04/04/27 16:30 ID:q77cjX5A
雨の速度について知りたいのですが、どうして速さに違いがあるのですか?
物理は非履修、ストークスの法則についてはサッパリ分りません。
速い雨と遅い雨があるのはなぜかな?
違いを決定付けているのは何かな?と思ったのです。
おバカな質問ですが宜しくお願いします。
(既出だったらスミマセン)
815ご冗談でしょう?名無しさん:04/04/27 16:34 ID:???
>>814
雨粒の重さと大きさでしょ
816814:04/04/27 16:51 ID:???
>>815
早速お答え頂きありがとうございます。
雨粒が大きくてもシトシトだったり、
小さくても針のようにサーッと降ったりするので不思議だったんですよね。
ありがとうございました。
817ご冗談でしょう?名無しさん:04/04/27 23:14 ID:w5YF4xfp
漏れのノートのHDのプラタが高速で回転してるわけだが
ジャイロ代わりになるのだろうか。
818792:04/04/28 00:23 ID:7yUOOsiA
>>793
>>812

ありがとうございました。
すっきりしました。

お返事遅くなりまして、すみません。

819ご冗談でしょう?名無しさん:04/04/28 00:52 ID:???
劣化ウランを体内に取り込んだ場合の影響が実際どの程度のものなのか、
どなたか試算していただけませんでしょうか。

すでに存在するURL等の紹介でもかまいませんが、政治的思惑を除いた
できるだけ客観的かつ定量的な結果を希望しています。

よろしくお願いします。
820ご冗談でしょう?名無しさん:04/04/28 02:56 ID:???
>>819
試算って、簡単に計算で出せるようなものじゃないでしょ
特にウラン固有の化学的性質に依存する部分は
821ご冗談でしょう?名無しさん:04/04/28 11:22 ID:p+j5tlMC
はじめまして。社会人ですが、なんとか独学でベクトル解析の勉強を
しています。入門書を読んでいて判らない所があるのですがお教えいただけますでしょうか?
1)等位面f(x,y,z)=cにおいてi+z(x)kと、j+z(y)kは接線ベクトルとなる理由
2)ストークスの定理の証明で、
-da(1)/dz*dz/dy*n(3)-da(1)/dy*n(3)=-df/dy*n(3)となる理由
についてです。偏微分記号がうまく書けなくてすみません。ただのdは偏微分記号のつもり、i,j,kは単位ベクトル、*は掛け算、z(x)はzのxによる微分、a(1)はベクトルaのx成分、n(1)は法線ベクトルのx成分です。よろしくお願いします。
822ご冗談でしょう?名無しさん:04/04/28 12:50 ID:yNfVdIc5
電機子の導体を取り外した鉄芯だけのローターが設けられた直流電動機の
補償巻線に電流を流した場合、固定子の界磁により励磁されたローターは
回転するのでしょうか?
つまりローター表面の磁極と電流間に電磁力が働くのでしょうか?
823ご冗談でしょう?名無しさん:04/04/28 18:40 ID:spgWuHFF
にごらせ馬茶の女はどうして腰前後に振らんのだろか?
824ご冗談でしょう?名無しさん:04/04/28 19:35 ID:???
>>821
x,y,zが独立変数なのに「z(x)はzのxによる微分」が出てくるのが意味不明。
等位面に対する接線ベクトルというのも意味不明。
面に対する接線ベクトルは何本でも引けるのでは?
a(1)などが何の関数なのか明示してないのでda(1)/dz*dz/dyとか書かれても意味不明。
いずれにしても説明不足で意味不明。
825ご冗談でしょう?名無しさん:04/04/28 19:35 ID:???
内燃機関で常温核融合をさせる事は可能でしょうか?
826ご冗談でしょう?名無しさん:04/04/28 20:01 ID:???
>>821
1)法線ベクトルとの内積=0を示せばいいよね。
 法線ベクトルは∇fだから、それと内積取ってみたら、
 0になりませんでした・・・。間違ってたらすみませんが、
 i-z(x)k,j-z(y)kだったりしませんか?
2)もう忘れました。a,n,fがよくわかりません。
 
827ご冗談でしょう?名無しさん:04/04/28 20:11 ID:???
>>825
その前に核融合自体が実用化されてないのだが。
828ご冗談でしょう?名無しさん:04/04/28 20:31 ID:3bp28hcW
ところで質問したいんですが
大学院っていうのは一般には今現在自分が通ってる大学の
大学院にしかいけないものなんですか?
たとえば静岡大学から早稲田の大学院に編入することとかってできないんですか?
829ご冗談でしょう?名無しさん:04/04/28 20:35 ID:???
>>828
できるよ。そんなに珍しいことでもないし
830ご冗談でしょう?名無しさん:04/04/28 21:26 ID:???
>>828
学歴ロンダリングという言葉が存在する位だからな…
最終学歴を東大にしたい香具師がほにゃららら
831ご冗談でしょう?名無しさん:04/04/28 22:27 ID:???
>>828
つうか、こんな所で聞かんで知りたい大学院の募集要項を調べろよ
832ご冗談でしょう?名無しさん:04/04/29 05:10 ID:5WakosGm
同じ運動エネルギーで
ドブロイ波が電子と陽子で電子の方が大きいのって
どうしてですか?
ご回答お願いいたします
833ご冗談でしょう?名無しさん:04/04/29 05:35 ID:???
λ=h/mv
だから
834ご冗談でしょう?名無しさん:04/04/29 05:41 ID:5WakosGm
ごめんなさい。
もうちょい詳しく教えてください。
昨日習ってよくわかってないもので・・・
835ご冗談でしょう?名無しさん:04/04/29 06:11 ID:u6le5AXX
ブライトさんという若い婦人がいました。その人の歩く速さは光より速かった。ある日外に出て、戻ると前の晩になってました。?? 誰か教えちくれ
836ご冗談でしょう?名無しさん:04/04/29 06:49 ID:???
若いから
837ご冗談でしょう?名無しさん:04/04/29 10:57 ID:???
>>832
陽子と電子の運動量を考える。
同じ運動エネルギーならおおきな差がでるでしょ。
さてその運動量でそれぞれのドブロイ波を考えてみる。
838 :04/04/29 16:09 ID:4b+Xjr4e
いま、お菓子のカール食べながらおもったんですけど、
カールってひとつひとつ搾り出されるように、うにゅっ、うにゅっ、って出てきますよね。
それで後々は油で揚げられますけど、その時におんなじかたちが出てくる確率ってどれくらいなんでしょう。
また、同じ形にしたらどうしたらいいんだろう、と。
ふと、頭に浮かんだ疑問なんです。すまそ。
839ご冗談でしょう?名無しさん:04/04/29 18:01 ID:???
光より速く歩く事なんて出来るのか?
840ご冗談でしょう?名無しさん:04/04/29 18:05 ID:???
>>838
「同じ」とは?
現在最先端の各種計測器の検出限界以下になる確率はほぼ0だろう。
その意味で「同じ」形に作ることはほぼ不可能だろう。
841ご冗談でしょう?名無しさん:04/04/29 18:47 ID:???
>>838
カールはノンフライ
842ご冗談でしょう?名無しさん:04/04/29 19:13 ID:TGG2TVeZ
>>823
にごらせ馬茶の女は腰前後バージョン収録済みなんだが
なんやプレスリー側の著作権でオンエアできんらしいのw
843ご冗談でしょう?名無しさん:04/04/29 19:37 ID:???
水溶液の混ざるさまを観察して、ミクロの世界で時間が逆行する
というような実験があったと思うのですが
誰のなんと言う実験だか分かる方います?
何とかの悪魔と言ったような気がするのですが・・・
844ご冗談でしょう?名無しさん:04/04/29 22:53 ID:kNDx4g8U
おもりをおとして記録テープで打点をとるあの実験ですが
おもりの重さを変えたときは結果に理論上は違いがでないはずですが
今回実験をしたら微妙に違いました
原因はありますか?
845ご冗談でしょう?名無しさん:04/04/29 22:58 ID:???
>>844

空気抵抗分ではなかろうか
真空で実験するよろし
846ご冗談でしょう?名無しさん:04/04/29 23:36 ID:???
テープと記録タイマーとの間の摩擦も関係するのでは?
847ご冗談でしょう?名無しさん:04/04/30 00:54 ID:???
>>844
おもりを落とすときのテープの角度とかも毎回違うから
そういうことも影響するんじゃない?
848ご冗談でしょう?名無しさん:04/04/30 08:55 ID:???
虹の色の順番ってどうやって決まってるんだっけ?
849ご冗談でしょう?名無しさん:04/04/30 09:07 ID:???
好きな色の順
850ご冗談でしょう?名無しさん:04/04/30 09:18 ID:???
くじ引き
851ご冗談でしょう?名無しさん:04/04/30 09:21 ID:pInnrZE4
次の国会で決まる
852ご冗談でしょう?名無しさん:04/04/30 09:28 ID:???
見る人によって違う
853ご冗談でしょう?名無しさん:04/04/30 09:44 ID:pInnrZE4
虹ってのは心の純粋な奴にしか見えないんだよ
見えない奴はギャルゲーでもやってなさいって
854ご冗談でしょう?名無しさん:04/04/30 10:25 ID:F7FnjqBI
今年理工学部に入学したものです。
下らないかもしれませんがお聞きしたいことがあります。
「宇宙のエントロピーは増大に向かう」のは常識と捉えられていますが
我々生命体は明らかにそのルールに反していますよね。考えてみると、
ある程度以上の知的生命体はそのルールを克服できるような気がします。
これは例外として認めていいのでしょうか、それとも宇宙スケールの時間で考えると
やはり一過性の乱れなのでしょうか?
教えていただけるとありがたいです。またがいしゅつでしたら誘導お願いします。
855ご冗談でしょう?名無しさん:04/04/30 10:28 ID:???
>我々生命体は明らかにそのルールに反していますよね。

反してません。
856ご冗談でしょう?名無しさん:04/04/30 10:36 ID:F7FnjqBI
すいません、マジレスですが自分の考えでは
不安定な(半減期の短い)物質が、単位空間において生命体に
多く存在していると思うんです。
ほっとけば一様な状態になるんじゃないのでしょうか?
857ご冗談でしょう?名無しさん:04/04/30 10:37 ID:???
>>835
光より速く運動すると時間を逆行するってのは嘘だよ。

>>843
何だろう? マクスウェルの悪魔というのはあるが、全然別物だし。

>>848
マジレスすると、屈折率で決まる。波長・周波数の順番と言ってもよい。

>>854
>「宇宙のエントロピーは増大に向かう」のは常識と捉えられていますが
そんな常識は無い。
「閉鎖系のエントロピーは増大に向かう」なら正しいとされてる。

>我々生命体は明らかにそのルールに反していますよね。
「閉鎖系のエントロピーは増大に向かう」というルールに反する現象は
生命体においても確認されてない。
858ご冗談でしょう?名無しさん:04/04/30 10:40 ID:???
>>857
宇宙は閉鎖系ではないという事でよろしいですか?
859ご冗談でしょう?名無しさん:04/04/30 11:22 ID:???
>>858
は? 生物だけ見たらエントロピーが減っているけど、それは生物が
閉鎖系ではないからで、周囲の環境まで含めればエントロピーは
増えてるってこと
860ご冗談でしょう?名無しさん:04/04/30 15:10 ID:F7FnjqBI
みなさまどうも有難うございました。
周囲=閉鎖系=近似すると今のとこ「地球上」
と考えればいいんですね。
物理学というと、何でもミクロレベルの話を還元すれば
説明できるのかな?とか思っていたものですから。
861ご冗談でしょう?名無しさん:04/04/30 15:13 ID:MwFaE+OM
tessu
862ご冗談でしょう?名無しさん:04/04/30 15:19 ID:???
>>860
地球上だって閉鎖系じゃねーよ
863ご冗談でしょう?名無しさん:04/04/30 15:29 ID:F7FnjqBI
そうするとどこまでを考慮して閉鎖系になるのですか?
宇宙全体では間違いなんですよね?
864ご冗談でしょう?名無しさん:04/04/30 15:57 ID:???
>>863
君が何について考えようとしているのか
分からなければ条件のつけようがないよ
865ご冗談でしょう?名無しさん:04/04/30 16:31 ID:???
>>864
「宇宙」
866ご冗談でしょう?名無しさん:04/04/30 17:41 ID:???
つうかエントロピーの定義をちゃんと見なおせ。
そこに閉鎖系の定義も必ず一緒に出てくる。
そうすれば自ずと分かる。
867ご冗談でしょう?名無しさん:04/04/30 17:59 ID:???
>>866
知ってんなら勿体ぶらずに書けよバーカ
どうせ知らないくせに
868ご冗談でしょう?名無しさん:04/04/30 18:02 ID:???
854はスゴ腕の釣り師だな。
869ご冗談でしょう?名無しさん:04/04/30 19:43 ID:F7FnjqBI
自分がアホなのは承知してますが、煽ったりしたい気持ちは
毛頭なく、単に物理学からみた生命はどのように位置づけ出来るのだろうと
考えただけなんです。

この場合閉鎖系は、実際の世界において何に言い換えられるのですか?
870ご冗談でしょう?名無しさん:04/04/30 19:58 ID:???
>>869
ひきこもり
871ご冗談でしょう?名無しさん:04/04/30 20:24 ID:???
エントロピー増大則が成り立つのは閉鎖系ではなく孤立系。
宇宙全体は孤立系だと思うけど、平衡状態ではなさそうなので
エントロピーが増大しているのかは知りません。
872ご冗談でしょう?名無しさん:04/04/30 20:27 ID:???
ひきこもりを準閉鎖系としy(ry
873ご冗談でしょう?名無しさん:04/04/30 20:33 ID:???
>>871
えーと、孤立系=閉鎖系、ではありませんでしたっけ?

>>869
宇宙全体は閉鎖系だと考えられてるけど、常識とまでは
言えないと思う。

余談だが、仮に宇宙が閉鎖系であると仮定した場合、
宇宙は膨張しているので、宇宙のエントロピーの
最大値も増大してることになる。そのため、宇宙の
エントロピーがいつまで増大しても最大値に達せず、
宇宙は常に非平衡状態にあると考えられる。
874ご冗談でしょう?名無しさん:04/04/30 20:47 ID:???
集合と位相、の位相ってなに?
875ご冗談でしょう?名無しさん:04/04/30 21:00 ID:XHTYbX79
劣化ウラン弾は小型核兵器と呼んでおおむね間違いはないのでしょうか?
876ご冗談でしょう?名無しさん:04/04/30 21:06 ID:???
>>875
呼びたきゃ呼べば? 他人との意志の疎通に苦労するだろうけど
877ご冗談でしょう?名無しさん:04/04/30 21:10 ID:F7FnjqBI
>宇宙のエントロピーの最大値も増大してることになる

なるほど、確かにそうですね。その考え方が抜けてましたね。
有難うございます。

>>870
連休家にいて2chじゃ確かにそうですねW
サークルで遊ぶのは自分には向いてないようです。。。。
878ご冗談でしょう?名無しさん:04/04/30 21:36 ID:???
なんだかあれとるの〜
879875:04/04/30 22:13 ID:XHTYbX79
>>876
他の人との意思疎通に苦労する、という事は、
とんでもない拡大解釈をするとそう言えない
事も無い、ぐらいなのでしょうか。

放射線量とかの理論はどうなっているのでしょうか。
880ご冗談でしょう?名無しさん:04/04/30 22:28 ID:???
>>879
すまん。はっきり書くべきだった。
呼びたきゃ呼べば? 他人との意志の疎通は不可能だろうけど
881ご冗談でしょう?名無しさん:04/04/30 22:51 ID:???
>>879
いたずら電話を精神兵器を呼ぶのと同義。
人が死なないものを兵器と呼称して通じる相手は居ないがな。
882ご冗談でしょう?名無しさん:04/04/30 22:54 ID:???
>>881
核兵器じゃないというだけで、兵器には違いないだろ
883ご冗談でしょう?名無しさん:04/04/30 23:48 ID:???
>>882
DUの核の作用で死ぬヤツはいない。
884ご冗談でしょう?名無しさん:04/04/30 23:49 ID:YACRRmgr
垂直落下運動についてなんですが、
何故理論上は、同じ高さから落とした物は全て同時に地面に落ちるのでしょうか
885ご冗談でしょう?名無しさん:04/04/30 23:53 ID:???
>>884
どちらにも同じ加速度がかかるから。
886ご冗談でしょう?名無しさん:04/05/01 00:02 ID:???
>>885
すいません、ちょっとそれがわかりません。
例えば、ソリに体重の重いヤツと軽いヤツが乗ったら
重いヤツのほうが速くうごきますよね。
そしたら重いヤツのほうがより強く加速度がかかっている気がするのですが…
887ご冗談でしょう?名無しさん:04/05/01 00:12 ID:???
馬鹿がいる
888ご冗談でしょう?名無しさん:04/05/01 00:21 ID:???
>>886
>重いヤツのほうが速くうごきますよね。
ダウト
889ご冗談でしょう?名無しさん:04/05/01 00:28 ID:L5oo6glR
摩擦
890886:04/05/01 00:32 ID:???
ということは、ソリに重いヤツが乗っても軽いヤツが乗っても
スピードは変わらないって事でしょうか?
ただの私の勘違いだったようですね。
レスありがとうございました!
891ご冗談でしょう?名無しさん:04/05/01 00:36 ID:???
>>874
噛み砕いていうと,
集合の各要素に対して,その「近傍」(その要素の近く)というものが定義されていれば,
それをもとにして,列(集合の要素が無限に並んでいるもの)の収束を論じることができる。
このような構造を位相といい,そういう集合を位相空間という。

一般に,集合の各要素間の距離が定められているとき,それを距離空間というが,
距離をもとにして近傍を定義できるので,距離空間は位相空間の特殊な例である。
そして,普通のn次元ユークリッド空間は距離空間の特殊な例である。

物理でいうところの「位相」や「位相空間」とは全然関係ないようだ。
892ご冗談でしょう?名無しさん:04/05/01 00:47 ID:???
>>890
いや、晴れた日にソリで雪の上をすべるような話なら重いデブが速いぞ。
893ご冗談でしょう?名無しさん:04/05/01 01:02 ID:iEm0S1XT
電車の中でジャンプしたら落下地点は前に進みますか?
894ご冗談でしょう?名無しさん:04/05/01 01:50 ID:???
>>893
やってみりゃわかる。
895ご冗談でしょう?名無しさん:04/05/01 01:58 ID:???
>>893
ものすごい勢いで回ってる地球の上でジャンプしてみたら?
896ご冗談でしょう?名無しさん:04/05/01 07:00 ID:???
>>892
そうか? 一見軽いヤツの方が速い気がする。
1:デブの方がでかいから空気抵抗大きそう、2:接地面の抵抗も重い方が大きい。
1も2も計ってみると間違ってたりしそうだけど。
897ご冗談でしょう?名無しさん:04/05/01 07:22 ID:???
>>896

デブは位置エネルギーがでかくね?

摩擦とか空気抵抗も大きいけど
デブのエネルギーってやっぱすごくね?
898ご冗談でしょう?名無しさん:04/05/01 09:41 ID:???
>>897
位置エネルギーは関係無い。
抵抗が無いならどんな重さでも同じ加速度。
899ご冗談でしょう?名無しさん:04/05/01 10:03 ID:???
マジレス
感謝する
900ご冗談でしょう?名無しさん:04/05/01 11:04 ID:yGyFKYm0
この板の方々なら簡単だと思うのですが、個人サイトを周っても
何か釈然としないものがあるので、ぜひわかり易く説明していただきたく参りました。

東京タワー(300m)から落とした場合、綿100Kgと鉄99Kgではどちらが先に地面に到達するのでしょうか?
901ご冗談でしょう?名無しさん:04/05/01 11:06 ID:???
空気はあるの?
902ご冗談でしょう?名無しさん:04/05/01 11:12 ID:+ww5yRl+
東京タワーは地球上だから、形状によるんでしょ、思考実験じゃない場合。
鉄だって100km四方に延ばせば綿より遅く落ちるだろうし。
903900:04/05/01 11:17 ID:???
言葉足らずでした。申し訳ないです。

空気がある場合と無い場合で教えていただきたいです。
綿100Kgと鉄99Kgは同じ形状だと仮定してください。
いや、実際ありえないことかもしれませんが個人サイトを周ってみても
ガリレオの答えが正しいのかどうか、いまだに納得できません
904ご冗談でしょう?名無しさん:04/05/01 11:37 ID:0BPbNeSo
特性エックス線は一回の遷移で一個の光子がでてくるのですよね?
エネルギー保存さえ破らなければいくつででも良いような気がするのですがなぜ一個しか出ないのですか?
905ご冗談でしょう?名無しさん:04/05/01 11:40 ID:CNn6B75u
http://ec.uuhp.com/~physics2/cgi-bin/upload250/NeoKiffg/001.GIF

位置ベクトルを使えばスマートに解けるらしいんですが、ちょっとよくわかりません。
よろしくお願いします。
906ご冗談でしょう?名無しさん:04/05/01 11:43 ID:???
>>904 確率の問題
907ご冗談でしょう?名無しさん:04/05/01 11:47 ID:???
>>904
??
準位E2、E1間の遷移一回につきE2-E1のエネルギをもつ光子が出るのが不思議なのか
その光子が一個なのが不思議なのか

どっちだい?
908ご冗談でしょう?名無しさん:04/05/01 11:51 ID:???
>>903
つまり、麺のほうが密度が高いと密度だと仮定すればよいのか?
909ご冗談でしょう?名無しさん:04/05/01 12:00 ID:???
>>903
真空でおとしてみ
910904:04/05/01 12:09 ID:???
>>906
確率の問題とはどういうことですか?

>>907
一個なのが不思議です
全部あわせてE1-E2なら何個でも良いと思うのです
911ご冗談でしょう?名無しさん:04/05/01 12:19 ID:???
>>905
「位置ベクトルを使うと・・・」というのはよく分からないけど、
∇・Eを先に計算してしまってから、それを体積積分するのが楽そう。
その際、極座標系で積分計算します。(dv = dr rdθ rsinθdφ)
912ご冗談でしょう?名無しさん:04/05/01 12:35 ID:???
>>905

r=√(x^2+y^2+z^2)とおくだけでよくね?
913ご冗談でしょう?名無しさん:04/05/01 12:41 ID:???
>>910
>一個なのが不思議です
>全部あわせてE1-E2なら何個でも良いと思うのです
X線放射の原理から言えば
準位間に相当するエネルギーの光子以外はほとんどでないということから

実際の検出では
散乱されたりして準位間のエネルギーより低いエネルギーのX線が連続で検出されます
914ご冗談でしょう?名無しさん:04/05/01 12:42 ID:???
>>910
例えば光子が2個出たとすると、E1とE2の間にもう1つ準位があることになりませんか?
915904:04/05/01 12:57 ID:???
>>909
X線発生の原理がわかりません
後半の連続スペクトルの話は理解しているつもりです

>>910
なぜ間に順位がないと二つ以上の光子にエネルギーを分配できないのかがわからないです
916ご冗談でしょう?名無しさん:04/05/01 12:59 ID:???
>>909
真空だったら重力ないから落ちないでしょ。
917ご冗談でしょう?名無しさん:04/05/01 13:03 ID:???
>>915
>なぜ間に順位がないと二つ以上の光子にエネルギーを分配できないのかがわからないです
それは自然がそうなってるからとしか言いようがないような・・・
918ご冗談でしょう?名無しさん:04/05/01 13:03 ID:???
>>916
真空無重力厨発見!!!
919904:04/05/01 13:07 ID:???
レス番間違えました
>>915>>913,914に対するレスです
920ご冗談でしょう?名無しさん:04/05/01 13:17 ID:???
>>915
>X線発生の原理がわかりません
高いエネルギー準位から低いエネルギー準位への遷移による電磁波の放射

>なぜ間に順位がないと二つ以上の光子にエネルギーを分配できないのかがわからないです
物質ごとに特定のエネルギーが出るという事実はなぜか説明するためにそうなってのです
921ご冗談でしょう?名無しさん:04/05/01 13:25 ID:CNn6B75u
>>911 >>912
レスどうもです。
http://ec.uuhp.com/~physics2/cgi-bin/upload250/NeoKiffg/002.GIF
↑になってしまったんですが、どこか間違ってますでしょうか?

>>912
位置ベクトルを使うというのはそういうことだと思うんですが、
そうすると式がどう簡単になるのかわからないんですが……

この場合、rは積分の外には出せないですよね?
922905:04/05/01 13:26 ID:CNn6B75u
923ご冗談でしょう?名無しさん:04/05/01 13:51 ID:???
>>915
まだ準位間の遷移の仕組みがよく分かってないから、理論的な説明は無理だろうな。
これは不確定性原理みたいなもんで「ねぇ、粒子はどこどこどこぉ??」って聞かれても
答えられないのと一緒。とりあえず実験事実で特性X銭のピークは一つなんだから
そういうものだと納得しとけ。というか光子光子って連発してるから最初「光子っていう
粒子1つが出てるんですかぁ?どうして一個って言い切れるんですかぁ?」みたいな
質問かとおもたw
924904:04/05/01 14:01 ID:???
>>917
そういう物なのでしょうか…

>>920
そういう現象だという事はわかります
ただ>>913はもう少し原理的な事を言っているかと思ったのですが…

後半はボーア模型とかはそうだと思うのですがもっと進んだ理解では他の事から説明出来てはいないのですか?

>>923
やっぱりそういう物なのでしょうか
どこまで勉強すれば理論的な理解ができるのでしょう?

最後の文はどういう意味か良くわかりません
925911:04/05/01 14:11 ID:???
>>905
θの積分範囲は0からπまでです。
(0から2πでは0になりませんか?)

>>904
専門外なのでよく知らないけど、
理論的にっていうと、
量子電磁力学か場の量子論の本で
電磁場(ベクトル場)の量子化について
調べてみるとわかるかもしれません。
9268+7=15って小さくね?:04/05/01 14:36 ID:???

>>921


> >>911 >>912
> レスどうもです。
> http://ec.uuhp.com/~physics2/cgi-bin/upload250/NeoKiffg/002.GIF
> ↑になってしまったんですが、どこか間違ってますでしょうか?
>


いやいやφとかθとか使わなくっていいんだ
rとx、y、zだけでいけまふっていうか
証明なんだからx、y、zは残さなきゃ


> >>912
> 位置ベクトルを使うというのはそういうことだと思うんですが、
> そうすると式がどう簡単になるのかわからないんですが……
>
> この場合、rは積分の外には出せないですよね?

r を外に出す必要はないんだ
そのまま面倒だけど、偏微分すればいいのだよ
そいで、r はほっといて
偏微分で外にでてきたやつらだけ整理すればいいんだ
ほら
面積積分の形になったでしょ
927904:04/05/01 15:54 ID:???
>>925
量子電磁力学or場の量子論ですか
難しそうですが勉強したいと思います
量子力学を先に勉強しないといけないでしょうか

僕の質問に付き合ってくださった皆様どうもありがとうございました
928905:04/05/01 16:35 ID:???
>>925
おっしゃるとおりです……(恥

>>926
なるほど、なんかこっちの方法だと、あっさりしすぎてて
なんとなく騙されたような気持ちがしてしまいますが、
確かにこちらの方がスマートに解けますね。


ご丁寧に教えていただいたお二方に感謝です。
929ご冗談でしょう?名無しさん:04/05/01 18:09 ID:???
>>883
?? だから核兵器じゃないと言ってるのだが?
930ご冗談でしょう?名無しさん:04/05/01 21:26 ID:???
>>918
はい?なんで宇宙は無重力だと思ってるの?
厨房氏ね。
931ご冗談でしょう?名無しさん:04/05/01 21:59 ID:wpQLBxZe
>>916>>930晒しage
932ご冗談でしょう?名無しさん:04/05/01 21:59 ID:???
>930
まさか真空だから無重力だと思ってるのか?
9338+7=15って小さくね?:04/05/01 22:06 ID:???
>>916

ワロタ
934ご冗談でしょう?名無しさん:04/05/01 22:18 ID:LZoz5Ozo
>>932
真空の条件に重力がないとしているだけじゃないの
935ご冗談でしょう?名無しさん:04/05/01 22:19 ID:???
>>930
そうだよね。ヘリは真空だから浮いてるんだよね。

ヘリをホバリングさせながら電車に入れると?
http://science2.2ch.net/test/read.cgi/sci/1063165177/l100
9368+7=15って小さくね?:04/05/01 22:22 ID:???
>>935

つまんね
937ご冗談でしょう?名無しさん:04/05/01 22:31 ID:UhA+A3Zj
ゴムより弾力のある物ってあんの?
938ご冗談でしょう?名無しさん:04/05/01 22:35 ID:???
>>934
意味わかんないんだけど
9398+7=15って小さくね?:04/05/01 22:35 ID:???
>>937

スーパーボールってはずまね?
940ご冗談でしょう?名無しさん:04/05/01 22:42 ID:LZoz5Ozo
>>938
真空では重力場がないみたいな
941ご冗談でしょう?名無しさん:04/05/01 22:47 ID:???
>>940
そんなこと勝手に決められても…。
942ご冗談でしょう?名無しさん:04/05/01 22:49 ID:???
量子論で使う真空のこと言ってんじゃね?
(重力の量子化の問題は置いといてさ)
943ご冗談でしょう?名無しさん:04/05/01 22:56 ID:???
>>942
はい消えたー
944ご冗談でしょう?名無しさん:04/05/01 23:02 ID:???
いい年こいて2chかよ
9458+7=15って小さくね?:04/05/01 23:06 ID:???
>>944
おまいもな
946ご冗談でしょう?名無しさん:04/05/01 23:20 ID:HDuHCfUz
>>904
906さんのいうとおり確率の問題です。
違う言葉でいうとフェイズスペースが0だから。

E=E1+E2として、
エネルギーEの自由な光子がE1とE2の二つの光子に崩壊しないのはなぜか?
ってのと同じこと。
これはエネルギーや運動量保存などからは禁止されていないんだけど、
実際にはおこらない。それは確率が0だから。
なんで0かっていうとフェイズスペースが0だから。

粒子の崩壊確率を自分で計算すると分かりやすいんだが。
947ご冗談でしょう?名無しさん:04/05/01 23:50 ID:???
エネルギーEの自由な光子がE1とE2の二つの光子に崩壊しないのは
スピン角運動量の保存によって禁止されているためじゃないかな。
948906:04/05/01 23:55 ID:???
>>904 >>946
確率0ではないでしょ。一次摂動の近似の範囲内では一光子しかでない。
二次摂動まで考えれば二光子放出もあり得る、ということでしょ。
949ご冗談でしょう?名無しさん:04/05/02 00:52 ID:???
三次摂動まで考えると3光子放出、と。
950ご冗談でしょう?名無しさん:04/05/02 02:37 ID:SQWRUFaX
重力と電気力(っていうのかな?)は同じですか?
951ご冗談でしょう?名無しさん:04/05/02 02:54 ID:???
>>950
違います
952ご冗談でしょう?名無しさん:04/05/02 02:54 ID:???
別物だから別の名前がついてる。
953ご冗談でしょう?名無しさん:04/05/02 02:55 ID:???
>>950
同じです



っていったらそれを信じるの?
954ご冗談でしょう?名無しさん:04/05/02 02:57 ID:SQWRUFaX
>>953

にわかには信じがたいですね。
955ご冗談でしょう?名無しさん:04/05/02 03:17 ID:???
>>954
カルツァー-クライン理論の範囲では同じとみなすこともできます


これだったら?
956ご冗談でしょう?名無しさん:04/05/02 09:58 ID:???
>>950
電気の力はプラスとマイナスがあって
プラスとマイナスは引き合い、同じもの同士は反発し合います。
一方重力の方もプラスとマイナスがあって
プラスとマイナスは引き合い、同じもの同士は反発し合うので同じものです。
957ご冗談でしょう?名無しさん:04/05/02 10:42 ID:???
馬鹿キタ━━━━(゜∀゜≡(゜∀゜≡゜∀゜)≡゜∀゜)━━━━!!!
958ご冗談でしょう?名無しさん:04/05/02 11:00 ID:???
よーし。釣り合戦なら負けないぞー!

一方比熱の方もプラスとマイナスがあって、
プラス同士は温度分布が一様になり、
マイナス同士は温度分布が複雑になっていくので同じ物です。
あ、プラスとマイナスをくっつけちゃ駄目よ。ビックバンが起きるかもw
959ご冗談でしょう?名無しさん:04/05/02 12:42 ID:???
性器にもプラスとマイナスがあり、差し込むとビッグバンがおきます。
このときの摩擦熱が皆さんを構成する分子の元になりました。
たまに同極同士をくっつける香具師がいますが、この正物質で構成される
世界では何も起きません。
960ご冗談でしょう?名無しさん:04/05/02 12:50 ID:???
                   i、              l゙
―――――ーッ            |           ----ふ--、,,_   i、
        /             |          │    ゙i、  ゙l、
     │ /       ゙゙゙゙゙゙゙゙゙゙゙゙厂゙゙゙゙゙゙゛      l゙     │  .゙l
     l゙              |             l゙     /′  │      ヽ   ,i、   丿
    丿              /  |  `-、     l゙    ./    `       ゙i、 丿.゙i、 丿
  _,/            '"   |    ‘'     /   、丿            ゙l,丿  .ヽ/′
                   "                ″             ″   .`
961ご冗談でしょう?名無しさん:04/05/02 14:24 ID:???
assholeは中性でつか?
962ご冗談でしょう?名無しさん:04/05/02 14:42 ID:woWktMLb
blackholeは中性子星でつか?
963ご冗談でしょう?名無しさん:04/05/02 15:38 ID:P7p0piBO
電子レンジのターンテーブルをはずして
チーズとかマシュマロをあっためると
電子レンジのマイクロ波の波長が見えるらしいのですが、
どこかに写真入りで乗ってるサイトないでしょうか?
964ご冗談でしょう?名無しさん:04/05/02 15:58 ID:???
ume
965ご冗談でしょう?名無しさん:04/05/02 15:58 ID:n2NRNmQ7
ume2
966ご冗談でしょう?名無しさん:04/05/02 17:06 ID:???
ちょっと質問させてください。
距離に比例して速度が減速する物体があるとして、t秒後の速度Vを求める式、
または、距離Lを進むのにかかる時間tを求める式を考えているのですが、
答えにたどり着けません。

具体的には、
初速Vm=10m/sとして、1m進むと速度が9m/s、2m進むと速度が8m/s、(3m,7m/s)・・・
(9m,1m/s)、(10m,0m/s)と変化する物体の、

t秒後の速度Vを求める。
距離L進むのにかかる時間tを求める式です。

とりあえず速度が0m/sになることは永遠に無いことはわかるのですが、
よろしくお願いします。
967ご冗談でしょう?名無しさん:04/05/02 17:15 ID:???
t秒後
10*e^(-t)
968ご冗談でしょう?名無しさん:04/05/02 17:21 ID:???
インフレーションの時代(GUT時代)に存在した
真空エネルギーはどうなったんです?
ダンピングと共に光になったんですか?
また、そうだとすると、真空エネルギーが光(ベクトル場)に移行する
根拠はなんですか?
スピノルになってもよさそうだが・・・・
その辺の理屈をキボンヌ!!
969ご冗談でしょう?名無しさん:04/05/02 17:44 ID:???
>>967
レスありがとうございます。
970904:04/05/02 19:36 ID:DedGL0vl
>>946-949
ありがとうございます
でも正直理解できません
結局2つ以上の光子に分配されることもあるけど非常に起こりにくいってことですか?
次の式を導け
En=-C*1/n^2=[-(2π^2*Z^2*e^4m)/(4πε)^2*h^2]*1/n^2

Z=核電化 e=電気素量 m=電子の質量である
また、全エネルギーに対する式
E=運動エネルギー+ポテンシャルエネルギー
E=(1/2mv^2)-Ze^2/4πεr

mvr=n(h/2π) n=1,2,3,4… mvr=角運動量 h=プランクの定数
Ze^2/4πεr^2=mv^2/r
を必要とする

わかりにくくてすいません。どなたかよろしくお願いします。
972ご冗談でしょう?名無しさん:04/05/03 17:54 ID:???
>>971
>1
【質問する前に】
教科書や参考書をよく読もう。
http://www.yahoo.co.jp/ とか http://www.google.com/ を利用し自分で調べること。
宿題を聞くときは、どこまでやってみてどこが分からないのかを書くこと。
丸投げはダメだからね。
(丸投げ君は完全無視。答えるだけ無駄。)
973ご冗談でしょう?名無しさん:04/05/03 20:48 ID:tI40PPp/
松本零士が時計に関するコラムで、私の持ってるセイコーの自動巻の時計はスペースシャトルエンデバーで、宇宙まで行ったことがあるとかいってましてた。
で、中坊な質問なんですが、無重力状態で自動巻ってどんな動作をするもんなんでしょうか?
どこかで実験したとか言う記事をどなたか見たことはないでしょうか?
あれって構造上やっぱ重力が無いと動きようがないですよね?
いつもは時計板なんですが、あちらで聞いたらいろんな意見が出てわからなくなったので、こちらに書きました。
物理なぞ中学以降やってないものですので、よろしくお願いします。
974ご冗談でしょう?名無しさん:04/05/03 22:34 ID:???
なんで重力がないと動かないと動かないと思うの?
975ご冗談でしょう?名無しさん:04/05/03 22:41 ID:???
>>974
うこがないということはなくても実際使い物にならないんじゃないか?>自動巻きの時計
976ご冗談でしょう?名無しさん:04/05/03 22:47 ID:???
>975
腕振ればちゃんと巻いてくれるんじゃないの?
重力なんか関係ないと思うが…。
977973:04/05/03 23:16 ID:tI40PPp/
振り子は重力がないと正常に動かないと認識しておりましたが?
ましてや、自由な円運動の出来ない状態では、それと密閉されたケースの中で一番最初の、力をどう与えてやるか、動いてもゼンマイの抵抗がどう働くか。
でも、物理板の方に言われるとちょっと自信が.....。
978ご冗談でしょう?名無しさん:04/05/03 23:20 ID:???
重力で何か影響あるなら、地球上でも上下逆さに向けたりとか
出来ないジャン。
979ご冗談でしょう?名無しさん:04/05/03 23:21 ID:???
>>977
もし、そういう作りだったとしたら人間の姿勢変化や加速度でまともに動かないぞ
980ご冗談でしょう?名無しさん:04/05/04 00:34 ID:???
>>977
>ましてや、自由な円運動の出来ない状態では、
>それと密閉されたケースの中で一番最初の、力をどう与えてやるか

だから、腕振るんでしょ。
981ご冗談でしょう?名無しさん:04/05/04 00:36 ID:???
右ねじの法則はどうやって証明されてますか?
論理的に説明した本など教えてください
982ご冗談でしょう?名無しさん:04/05/04 00:37 ID:???
>>977
感性のない奴だ・・・
983ご冗談でしょう?名無しさん:04/05/04 00:41 ID:???
>>981
法則じゃないだろ
単なる規格
ねじ回す時にどっちに回せば緩めることができるかがマニュアル見ないと判らないようじゃ不便だろ?
下手に逆に回すとぶっ壊れることもあるし・・・特に相手がプラスチックの場合とか
普通はほとんどが右ねじなので左に回せば緩む
逆の構造をした特殊なねじもある
984ご冗談でしょう?名無しさん:04/05/04 09:31 ID:???
>>983
ワロタ
985ご冗談でしょう?名無しさん:04/05/04 12:35 ID:???
自動巻の時計
重力が関係ないというのは明らかにおかしい
ちょっとした姿勢変化でも巻けるのは重力のおかげ
ただ水平面上の運動だけでも巻けるから十分に腕を動かしていれば無重量状態で使うことはできるだろう
986ご冗談でしょう?名無しさん:04/05/04 13:38 ID:???
>>985
 自動巻きってのは時計に加わる「加速度の変化」で巻くんだよ。
だから重力そのものは直接関係ない。
987ご冗談でしょう?名無しさん:04/05/04 17:26 ID:???
>>986
加速度の変化?
じゃあ重力場中で一定の速度で回転させても巻けないのか?
988ご冗談でしょう?名無しさん:04/05/04 18:02 ID:LF6nfgz/
▼宇宙計画が採用したアナログ時計
 面白い事実がある。アポロ計画(米国が国威をかけて推進させた人類を月に送るという宇宙開発計画)でアメリカ航空宇宙局が採用した公式時計は、機械式の手巻き時計であった。
 1969年7月20日、人類史上初めての有人宇宙船が月面に着陸したとき、アームストロング船長はじめコリンズ、オルドリン宇宙飛行士の腕に巻かれた時計は、スイスOMEGA(オメガ)社の作った手動巻きゼンマイ機械式「スピードマスター」であった。
この宇宙飛行用として公認された時計はクォーツ式でもなければ当時注目を集めていた音叉発振時計でもなかった。
クォーツ時計は電池を使用し長い年月にわたり高温や低温にさらされると電池の性能が低下したり機能しなくなる。
手でゼンマイを巻く機械式時計は精度はクォーツよりも劣るものの宇宙で想定されるさまざまな環境への対応力が大きかった。
また自動巻は重力を利用して内蔵された振り子でゼンマイを巻く方式であるが無重力では機能しないため確実な手動巻きが選定された。
989ご冗談でしょう?名無しさん:04/05/04 18:03 ID:???
>987
そりゃ巻けるだろ。振り子なんだから。
腕振って巻くのに重力はいらんと思うがね。
990ご冗談でしょう?名無しさん:04/05/04 18:35 ID:???
>>989
重力なくても巻けるけど重力があればより効率よく巻けるよ
991ご冗談でしょう?名無しさん:04/05/04 18:45 ID:???
次ぎスレ立てようと思ったが>>2のリンクがおかしい(たとえばpart25)のとscience鯖時代のリンクが確認不能なので断念しますた
誰かそういうことに頓着しない人がいたら立ててくれ
俺は完ぺき主義者なんでな
992ご冗談でしょう?名無しさん:04/05/04 19:27 ID:vKKAxdyp
発光ダイオードって、そのダイオードの光の波長が短くなるほど立ち上がりの電圧が高くなるらしいのですが
この理由わかる方いましたらおしえてください。。
993ご冗談でしょう?名無しさん:04/05/04 20:50 ID:???
>>992
波長が短い=光子一個のエネルギーが大きいからではないかと
994ご冗談でしょう?名無しさん:04/05/04 21:07 ID:OxpVC+L8
弱い相互作用というの引力ですか、それとも斥力ですか?
995ご冗談でしょう?名無しさん:04/05/04 21:10 ID:vKKAxdyp
>993
すみません・・・
もうちょっと詳しくおしえてくれます?
996ご冗談でしょう?名無しさん:04/05/04 21:14 ID:???
>>989
また自動巻は重力を利用して内蔵された振り子でゼンマイを巻く方式であるが無重力では機能しないため確実な手動巻きが選定された。
997ご冗談でしょう?名無しさん:04/05/04 21:23 ID:EXxd3m7J
放射線って原子のどこら辺から出るんですか?
んでまた、放射線って何ですか?
998ご冗談でしょう?名無しさん:04/05/04 21:33 ID:???
>>997
ひみつの場所。
アルファ線はヘリウム原子核そのもの、
ベータ線は電子そのもの、
ガンマ線は光子そのもの
999ご冗談でしょう?名無しさん:04/05/04 21:36 ID:???
>996
腕振れば巻けるってば。
1000ご冗談でしょう?名無しさん:04/05/04 21:36 ID:ScEyRFCG
>>987
>>987
放射線には
あるふぁ船
べーた船
がんま船
の3つがある

あるふぁ船はHeの原子核と同じものが原子核から出港する
べーた船は電子が原子核から出港する
がんま船は電磁波が原子核から出港する

要するに放射線は原子核から出る
出てくる物によって呼び方が違うのだ。
10011001
このスレッドは1000を超えました。
もう書けないので、新しいスレッドを立ててくださいです。。。